M Reasoning en

You might also like

Download as pdf or txt
Download as pdf or txt
You are on page 1of 185

SSC

REASONING
CONTENT

  SL. NO. CHAPTER NAME  PAGE NO.

1. Word Analogy 3 – 9

2. Word Classification 10  –  14

3. Number  Series 15  –  19

4. Letter  Series  &  Grouping 20  –  23

5. Mixed  Series 24  –  25

6. Coding  Decoding 26  –  41

7. Problems  Based  on  Words 42  –  49

8. Direction 50  –  63

9. Clock  &  Calendar 64  –  70

10. Number  Matrix 71  –  75

11. Ranking 76  –  84

12. Cubes  &  Dice 85  –  91

13. Blood  Relation 92  –  99

14. Logical Venn Diagram 100  –  107

15. Syllogism 108  –  125

16. Non-Verbal  Reasoning 126  –  161

17. Puzzle 162  –  169


SSC
SSC
1. WORD ANALOGY
What is analogy Type of relationship: Whole and part relationship.
Literal meaning of analogy is ‘similarity’. But in terms
of reasoning, it is logical similarity in two or more
4. In  the  following  questions,  a  group  of  three
things identified on the basis of shapes, sizes, proper-
interrelated words is given, choose a word from
ties, traits etc.
the given alternatives, that is similar to the given
Word Analogy words and hence belong to the same group.
It establishes relationship of logical similarity in two Caw : Croack : Bleat
or more words.
(a) Cattle (b) Animal
Formats of the questions
(c) Low (d) Camel
1. Doctor : Hospital :: Teacher : ?
Explanation: The given words are the sound of
(a) Education (b) School animals/Birds. ‘Caw’ is the sound produced by
(c) Garden (d) Subject ‘Crow’; ‘Croak’ is the sound produced by
Explanation: ‘Hospital’ is the working place for ‘Frog’and ‘Bleat’ is the sound produced by ‘Goat’.
a ‘Doctor’ and similarly, ‘School’ is the working Clearly, the three words are sounds of animals/birds
place for a ‘Teacher’. Hence correct answer is and hence the word belonging to this group must
option (b). be ‘Low’ which is sound produced by ‘Cattle’.
Therefore, correct option is (c).
Type of relationship: Worker and working place
relationship Type of relationship: Animal and Sound relationship
2. ‘Donkey’ is related to ‘Bray’ in the same way as 5. In the following question, some words are given
‘Horse’ is related to ......... which  are  related  in  some  way.  The  same
relationship obtains among the words in one of
(a) Bark (b) Mew
the alternatives given under it. Find the correct
(c) Roar (d) Neigh alternative.
Explanation: ‘Bray’ is the sound of ‘Donkey’ Cat : Dog : Tail
and similarly, ‘Neigh’ is the sound of ‘Horse’.
(a) Home : Hut : Palace
Hence correct answer is option (d).
(b) Lion : Tiger : Fish
Type of relationship: Animal and Sound relationship
(c) Car : Bike : Gear
3. The  following  question consists  of  two  words
that have a certain relationship to each other, (d) Horse : Cow : Horn
followed by four pairs of words. Select the pair (e) None of these
that has the same relationship as the original Explanation: Both ‘Cat’ and ‘dog’ have ‘tail’and
pair of words. following the similar relationship, option (c) is
Car : Wheel correct as ‘Car’ and ‘Bike’ both have ‘Gear’.
(a) Bicycle : Speed From the question formats, it is clear that questions of
(b) Milk : Butter analogy is based on certain relationships. Some of the
such realtionships are given below: -
(c) Computer : Hard Disk
Important Relationship
(d) Tv : Movie
Antonymous relationship
Explanation: ‘Wheel’ is the part of ‘Car’ and in the
same way ‘Hard Disk’ is the part of ‘Computer’. Example: Import : Export
Hence option (c) is the correct answer. Synonymous relationship
SSC
Example: Victory : Win Example: Cook : Food
Gender relationship Person and Working Place relationship
Example: King : Queen Example: Doctor : Hospital
Country & Continent relationship Worker and Tool relationship
Example: India : Asia Example: Black smith : Hammer
Country & Capital relationship Tool & Work relationship
Example: Srilanka : Colombo Example: Pen : Writing
Country & Currency relationship Individual & Dwelling Place relationship
Example: Japna : Yen Example: King : Palace
Country & Parliament relationship Animals/Birds & Dwelling Place relationship
Example: Afganistan : Shora Example: Lion : Den
Country & National Emblem relationship Animals/ Birds & Sound relationship
Example: Iran : Rose Example: Horse : Neigh
Country & National Animal relationship Animals & Youngones relationship
Example: India : Tiger Example: Cow : Calf
Country & National Flower relationship Religion & Worship Place relationship
Example: India : Lotus Example: Sikh : Gurudwara
Country & News Agencies relationship Religion & religious books relationship
Example: China : Xinua Example: Muslim : Kuran
Country & Intelligence Agencies relationship Flag & Symbol relationship
Example: India : CBI Example: Red Flag : Revolution
Country & National Games relationship Symbol & meaning relationship
Example: Japan : Judo Example: Wheel : Progress
Country & River relationship City & Founder relationship
Example: Italy : Tiber Example: Agra : Sikander Lodi
Country & Tribes relationship Sports & Playing Place relationship
Example: Canada : Eskimos Example: Badminton : Court
Country/City & Sites relationship Sports & Trophies relationship
Example: Australia : Opera House Example: Cricket : Duleep Trophy
States & Capitals relationship Award & Area relationship
Example: UP : Lucknow Example: Oskar : Film
States & High Court relationship Apart from the above mentioned relationships,
Example: UP : Allahabad there may be other kinds of relationships and they may
be based on the facts of History, Politics, Economics,
Product & Raw Materials relationship
Science & Technology, Geography, Culture, Sports,
Example: Curd : Milk Current Affairs etc.
Worker & Product relationship
SSC
Exercise
Directions (Q.  1 - 10): In each of the (c) Usable (d) Radical
f o l l o w i n g   q u e s t i o n s ,   t h e re   i s   a   c e r t a i n 12. ‘Joule’  is  related  to  ‘Energy’  in  the  same
relationship  between  two  given  words  on  the way as ‘Pascal’ is related to ................
one side of :: and one word is given on another (a) Purity (b) Volume
side  of  ::  while  another  word  is  to  be  found
(c) Pressure (d) Density
from  the  given  alternatives,  having  the  same
relation  with  this  word  as  the  words  of  the 13. ‘Umpire’  is  related  to  ‘Pitch’  in  the  same
given pair bear. Choose the correct alternative. way as ‘Engineer’ is related to ................
1. Monkey : Gibber :: Owl : ? (a) Stage (b) Site
(a) Roar (b) Hoot (c) Technology (d) Ramp
(c) Hiss (d) Beat 14. ‘Ecology’ is related to ‘Environment’ in the
s a m e   w a y   a s   ‘ H i s t o l o g y ’  i s   re l a e d   t o
2. Fruit : Orange :: Mammal : ?
..............
(a) Sparrow (b) Fish
(a) Fossiles (b) Tissues
(c) Snake (d) Cow
(c) History (d) Hormones
3. Sheep : Lamb :: Insect : ?
15. ‘Indolence’ is related to ‘Work’ in the same
(a) Cub (b) Larva way as ‘Taciturn’ is related to ...............
(c) Bull (d) Tadpole (a) Observe (b) Speak
4. Monk : Nun :: Bachelor : ? (c) Cheat (d) Acting
(a) Spinster (b) Woman Directions (Q. 16 - 19): The following questions
(c) Lady (d) Man consist of two words each that have a certain
5. Bee : Hive :: Hare : ? relationship  to  each  other,  followed  by  four
(a) Web (b) Burrow pairs of words. Select the pair that has the same
(c) Stable (d) Nest relationship as the original pair of words.
6. Artist : Troupe :: Grapes : ? 16. Food : Hungry
(a) Sweet (b) Bunch (a) Thought : Politics
(c) Garden (d) Fruit (b) Water : River
7. Spree : Germany :: Danube : ? (c) Rest : Weary
(a) Austria (b) USA (d) Wine : Intoseication
(c) Canada (d) UK 17. Chinaman : Googli
8. America : Red Indians :: Newzeland : ? (a) Bahratnatyam : Kathak
(a) Pigmi (b) Nigro (b) Gidda : Peseta
(c) Moori (d) Sherpa (c) Ghumar : Hudson
9. Dark : Fear :: Honesty : ? (d) Apple : Leaf
(a) Personality (b) Money 18. Fan : Sweat
(c) Treachery (d) Trust (a) Fire : Smoke
10. Pigeon : Peace :: White flag : ? (b) Rain : Drought
(a) War (b) Surrender (c) Wind : Evaporation
(c) Victory (d) Friendship (d) Crop : Greenery
11. ‘Impossible’  is  related  to  ‘Feasible  in  the 19. Ampear : Current
same  w ay  as  ‘Theortical’  is  related  to (a) Sound : Waves
................ (b) Newton : Force
(a) Workable (b) Practical (c) Speed : Time
SSC
(d) Distance : Mile relation  with  this  word  as  the  words  of  the
Directions (Q. 20 - 22): In each of the following given pair bear. Choose the correct alternative.
questions,  a  group  of  inter  related  words  is 27. Car : Garage :: Aeroplane : ?
given. Choose a word from given alternatives (a) Post (b) Depot
that  is  similar  to  the  given  words  and  have (c) Hanger (d) Yard
belongs to the same group.
28. Pork : Pig :: Venison : ?
20. Lungs : Kidney : Heart
(a) Deer (b) Cow
(a) Liver (b) Blood
(c) Goat (d) Sheep
(c) Eyes (d) Finger
29. Bear : Lumber :: Horse : ?
21. Radish : Carrot : Potato
(a) Strut (b) Swoop
(a) Tomato (b) Spinach
(c) Gallop (d) Leap
(c) Apple (d) Groundnut
30. Faislabad : Pakistan :: Mirpur : ?
22. Branch : Root : Stem
(a) Afganistan (b) Maldives
(a) Cement (b) Leaf
(c) Bangladesh (d) Malaysia
(c) Tree (d) Fertilizer
31. Scismology : Earthquake :: Entomology : ?
23. Which  of   the  f ollow ing  is  the  same  as
(a) Blood (b) Kidney
Rabbit, Rat, Mole?
(c) Insects (d) Fishes
(a) Ant (b) Squirrel
32. War  is  related  to  Destruction  in  the  same
(c) Cat (d) Lion
way as Fire is related to ..........
24. Which  of   the  f ollow ing  is  the  same  as
(a) Food (b) Water
Norway, Poland, Spain?
(c) Heat (d) Burn
(a) France (b) China
33. Korea is related to Won in the same way as
(c) Kenya (d) Tokyo
Thailand is related to ...........
Directions (Q. 25 - 26): In each of the following
(a) Drachma (b) Baht
questions,  some  words  are  given  which  are
related  in  some  way.  The  same  relationship (c) Guilder (d) Peso
obtains  among  the  words  in  one  of  the  four 34. Chennai is related to Bangalore in the same
alternatives  given  under  it.  Find  the  correct way as Ranchi is related to ..........
alternative (a) Bhagalpur (b) Jharkand
25. Ink : Pen : Pencil (c) Patna (d) Gaya
(a) Juice : Orange : Banana Directions (Q. 35 - 37): The following
(b) Table : Chair : Wood questions consist of two words each that have
(c) Fish : Shark : Water a  certain  relationship  to  each other,  followed
by four pairs of words. Select the pair that has
(d) Cow : Milk : Curd
the  same  relationship  as  the  original  pair  of
26. Picture : Clock : Wall words.
(a) Pen : Pencil : Colour 35. Fish : Shoal
(b) Flowers : Garden : Park (a) Pilgrims : Carvan
(c) Footpath : Road : Highway (b) Cat : Cub
(d) Pillow : Quilt : Bed (c) Swan : Larva
Directions (Q.  27 - 34): In each of the (d) Deer : Child
f o l l o w i n g   q u e s t i o n s ,   t h e re   i s   a   c e r t a i n
36. Spain : Bull Fighting
relationship  between  two  given  words  on  the
one side of :: and one word is given on another (a) India : Cricket
side  of  ::  while  another  word  is  to  be  found (b) China : Hockey
from  the  given  alternatives,  having  the  same (c) Japan : Football
SSC
(d) USA : Baseball 43. DE : 10 :: HI : ?
37. Croak : Froag (a) 17 (b) 20
(a) Patter : Rain (c) 36 (d) 46
(b) Cackle : Sparrow 44. CE : 70 :: DE : ?
(c) Jingle : Monkey (a) 90 (b) 60
(d) Rustle : Drum (c) 120 (d) 210
Directions (Q. 38 - 39) In each of the 45. AG : IO :: EK : ?
following  questions,  a  group  of  inter  related (a) LR (b) MS
words  is  given.  Choose  a  word  from  given (c) PV (d) SY
alternatives that is similar to the given words
46. AKU : ? :: CMW : DNX
and have belongs to the same group.
(a) BGL (b) BLO
38. Eel : Shark : Cod
(c) BGQ (d) BLV
(a) Fishes (b) Dolphin
47. ACE : FHJ :: OQS : ?
(c) Snakes (d) Crocodile
(a) PRT (b) RTU
39. Hectare : Radius : Degree
(c) TVX (d) UWY
(a) Litre (b) Power
48. FHEG : JLIK :: PROQ : ?
(c) Energy (d) Force
(a) TVSU (b) VTUS
Directions (Q. 40 - 41): In each of the
following  questions,  some  words  are  given (c) TVUS (d) SUVT
w hich  are  related  in  some  w ay.  The  same 49. ACEG : NPRT :: ADGJ : ?
relationship  obtains  among  the  words  in  one (a) NRTX (b) ORVX
of the four alternatives given under it. Find the (c) NQSU (d) NQTW
correct alternative 50. ACEG : DFHJ :: QSUW : ?
40. Correspondent : News : Newspaper (a) KMNP (b) MNPR
(a) Barbar : Scissor : Shop (c) TQST (d) TVXZ
(b) Cloud : Water : Pond 51. BOQD : ERTG :: ANPC : ?
(c) Road : Vehicle : Destination (a) DQSF (b) FSHU
(d) Farmer : Crops : Food (c) SHFU (d) DSQF
41. Lizard : Reptile : Insect 52. ACEG : IKMO :: PRTV : ?
(a) Tiger : Mammal : Deer (a) QRUW (b) JLMP
(b) Fly : Insect : Bee (c) WXAC (d) XZBD
(c) Fox : Wolf : Forest 53. BCFE : HILK :: NORQ : ?
(d) Man : Omnivorous : Meat (a) TXWU (b) TXUW
Directions (42-90): In each of the Following (c) TUXW (d) TVWX
questions, there is some relationship between the two 54. BYCX : DWEV :: FUGT : ?
term to the left of (: :) and the same relasionship holds
(a) EHIJ (b) GHIJ
between the two terms to its right. Also in each
question, one term either to the right of (: :) or to the (c) HSIR (d) SRHS
left of it is missing. This term is given as one of the 55. EGIK : FILO :: FHJL : ?
alternatives given below each question. Find out this (a) GJMP (b) GMJP
term. (c) JGMP (d) JGPM
42. C : 16 :: F : ? 56. FILM : ADGH :: MILK : ?
(a) 30 (b) 49 (a) ADGF (b) HDGE
(c) 40 (d) 50 (c) HDGF (d) HEGF
SSC
57. BJNT : CIOS :: DHPV : ? 71. PALE : LEAP :: POSH : ?
(a) EGQU (b) EIQW (a) HSOP (b) POHS
(c) ELPV (d) EIOU (c) SHOP (d) None of these
58. QIOK : MMKO :: YAWC : ? 72. ADHM : ZWSN :: CFJO : ?
(a) SUEG (b) VUES (a) YURM (b) WTPK
(c) USGA (d) UESG (c) XWTP (d) XUQL
59. CAT : DDY :: BIG : ? 73. BVSC : YEHX :: MRCP : ?
(a) CLL (b) CLM (a) NJXK (b) LKXM
(c) CML (d) CEP (c) NIXK (d) OIYM
60. ABCD : OPQR :: WXYZ : ? 74. AFKP : ZUPK :: BGLQ : ?
(a) EFGH (b) KLMN (a) YUQM (b) XURO
(c) QRST (d) STUV (c) YXWV (d) YTOJ
61. KMF : LLH :: RMS : ? 75. DRIVEN : EIDRVN :: BEGUM : ?
(a) SLR (b) SLU (a) UEBGM (b) EUBGM
(c) SSU (d) SUS (c) BGMEU (d) BGMUE
62. CLOSE : DNRWJ :: OPEN : ? 76. Dda : aDD :: Rrb : ?
(a) PRJQ (b) RPJB (a) BBR (b) bRR
(c) PRHR (d) RZWR (c) RRR (d) DDA
63. REASON : SFBTPO :: THINK : ? 77. kcaC : Cack :: XgmF : ?
(a) SGHMJ (b) UIJOL (a) EmgF (b) EgmX
(c) UHNKI (d) UJKPM (c) FmgX (d) GmeF
64. AZBY : DWEV :: HSIR : ? 78. CFED : PSQR :: JMKL : ?
(a) JQKO (b) KPCL (a) YXZW (b) YVZX
(c) KPLO (d) KOLP (c) WZWZ (d) UVXZ
65. AZBY : CXDW :: EVFU : ? 79. DWH : WDS :: FUL : ?
(a) GTHS (b) GHTS (a) UFO (b) OFU
(c) GSTH (d) TGSH (c) FOU (d) ELV
66. SKIP : RIFL :: KYKZ : ? 80. CIRCLE : RICELC :: SQUARE : ?
(a) WJHV (b) WJVH (a) QSUERA (b) QUSERA
(c) JWVH (d) JWHV (c) UQSAER (d) UQSERA
67. HKNQ : GDAX :: SVYB : ? 81. RATIONAL : RATNIOLA :: TRIBAL : ?
(a) TQMK (b) ROLI (a) IRLAB (b) TRIALB
(c) ZVTQ (d) ADGJ (c) TIRLBA (d) TRILBA
68. LOGIC : BHFNK :: CLERK : ? 82. COUNSEL : BITIRAK :: GUIDANCE :?
(a) XVRPA (b) QBKJA (a) EOHYZKBB (b) FOHYZJBB
(c) LBRTU (d) JQDKB (c) FPHZZKAB (d) HOHYBJBA
69. NUMBER : UNBMRE :: GHOST : ? 83. EVTG : HSQJ :: CXVE : ?
(a) HOGST (b) HOGTS (a) EVUF (b) FSUH
(c) HGSOT (d) HGOST (c) FUSH (d) FUTG
70. TALE : LATE :: ? : CAFÉ 84. EGIK : WUSQ :: DFHJ : ?
(a) FACE (b) CAEF (a) BDFH (b) ECGI
(c) CEFA (d) FEAC (c) SQOM (d) XVTR
SSC
85. USPL : KMPT :: LJGC : ? 100 100
(a) BDGK (b) CEHL (a) (b)
64 81
(c) GHIJ (d) QSUW
64 81
86. ACFJ : ZXUQ :: EGJN : ? (c) (d)
120 100
(a) DBYU (b) VTQM
(c) VTRP (d) VUSQ 89. B : 16 :: D :: ?
87. NOLM : IJGH :: STQR : ? (a) 120 (b) 130
(a) MNKL (b) NOLM (c) 200 (d) 256
(c) LMNO (d) OPMN T X
90. : 2 :: : ?
169 J H
88. MK : :: J H : ?
121 (a) 2 (b) 3
23
(c) 4 (d)
7

Answers
1. b 2. d 3. b 4. a 5. b 6. b
7. a 8. c 9. d 10. b 11. b 12. c
13. b 14. c 15. b 16. c 17. a 18. b
19. b 20. a 21. d 22. b 23. b 24. a
25. a 26. d 27. c 28. a 29. c 30. c
31. c 32. d 33. b 34. c 35. a 36. d
37. a 38. b 39. a 40. d 40. 42. b
43. c 44. a 45. b 46. d 47. c 48. a
49. d 50. d 51. a 52. d 53. c 54. c
55. a 56. c 57. a 58. d 59. a 60. b
61. b 62. c 63. b 64. c 65. a 66. d
67. b 68. d 69. a 70. a 71. c 72. b
73. c 74. d 75. a 76. b 77. c 78. c
79. a 80. d 81. d 82. b 83. c 84. d
85. a 86. b 87. b 88. a 89. d 90. 2
SSC
2. Word Classification
What is classification? Choose the word which is least like the other
Classification means finding out different item from a words in the group.
given group of items. In another words, in classification, (a) Pistol (b) Gun
we analyse the items of given group on the basis of (c) Rifle (d) Canon
their shapes, sizes, traits, nature, colour and other (e) Sword
properties and then spot the stranger or odd one out. Explanation:  Here, all except sword are fire arms.
Example : Lion, Tiger, Fox, Crocodile Hence, (e) is the correct option.
Explanation : Clearly, all the animals given above are 2. Word Pair Classification
wild animals. But out of these four animals only Example
crocodile is a water animal as well as a reptile. Hence,
Choose  the  odd  word  pair  one  of  the  given
crocodile will be an odd man.
alternatives.
Word Classificaion
(a) Bird : Chinp (b) Snake : Hiss
Under word classification, one has to take an odd word
(c) Lion : Roar (d) Frog : Bleat
out of the given group of words.
(e) Bees : Hum
Formats of the questions
Explanation: (d) is the correct option as sound of frog
1. One Word Classification
is croak and not bleat. Bleat is the sound of goat.
Example In the other options individual and their sound
relationships are correct.

Exercise
Directions (Q.  1 - 15) Choose the odd man from 7 . (a) Manali (b) Jaipur
the  words  given  below  in  the  question. (c) Mussoorie (d) Darjeeling
1 . (a) Pound (b) Kyat (e) Shimla
(c) Joule (d) Krona 8 . (a) Indigo (b) Green
(e) Yuan (c) Pink (d) Yellow
2 . (a) Canberra (b) Havana (e) Orange
(c) Naiorbi (d) Athens 9 . (a) Krait (b) Phython
(e) Cebu (c) Cobra (d) Viper
3 . (a) panther (b) Cow (e) Mamba
(c) Fox (d) Snake 10. (a) Square (b) Triangle
(e) Dog (c) Circle (d) Rectangle
4 . (a) March (b) May (e) Cube
(c) August (d) November 11. (a) Dog (b) Sheep
(e) July (c) Camel (d) Cow
5 . (a) Guava (b) Litchi (e) Goat
(c) Papaya (d) Watermelon 12. (a) Grapes (b) Pineapple
(e) Jack fruit (c) Banana (d) Orange
6 . (a) Write (b) Read (e) Pomegranate
(c) Knowledge (d) Learn 13. (a) Victoria (b) Gersoppa
(e) Study (c) Angel (d) Nigar
SSC
(e) Marina (a) Asia (b) Australia
14. (a) Haridwar (b) Ahmadabad (c) Antarctica (d) Africa
(c) Varanasi (d) Kanpur (e) Austria
(e) Allahabad 24. Three out of the four options given below
15. (a) Guitar (b) Sitar are  alike  in  some  way  and  hence  form  a
(c) Veena (d) Flute group.  Find  out  the  option  that  does  not
belong to the group.
(e) Violin
(a) Pakistan (b) India
Directions  (Q.    16  -  20)  In  each  questions
given  below, choose the odd  word  pair  out  of (c) Afganistan (d) Srilanka
the  given  alternatives. 25. Three  out of  the four options given below
16. (a) Doctor : Hospital are  alike  in  some  way  and  hence  form  a
group.  Select  the  option  that  is  different
(b) Mason : Wall
from the group.
(c) Actor : Stage
(a) Hare : Burrow
(d) Farmer : Field
(b) Pig : Den
(e) Worker : Factory
(c) Horse : Stable
17. (a) Spain : Madrid (b) China : Bejing
(d) Bee : Hive
(c) Austria : Viena (d) Peru : Lima
26. Three  out of  the four options given below
(e) Finland : Tokyo are alike in a certain way and hence form a
18. (a) Carpenter : Saw group.  Find  out  the  option  that  does  not
(b) Barber : Scissors belong to the group.
(c) Goldsmith : Ornaments (a) Jawahar lal Nehru
(d) Scultor : Chisel (b) Lal Bahadur Shastri
(e) Writer : Pen (c) Dr. Rajendra Prasad
19. (a) Begining : End (d) Rajiv Gandhi
(b) Difficult : Easy 27. Four out of the five options given below are
(c) Coward : Brave alike  in  a  certain  way  and  hence  form  a
(d) Weak : Robust group.  Find  out  the  option  that  does  not
belong to the group.
(e) Dirty : White
(a) Bold (b) Stump
20. (a) Salman Kurshid : Congress
(b) Sushma Swaraj : BJP (c) Cover (d) Diamond
(c) Mulayam Singh Yadav : SP (e) Doosra
Directions  (Q.  28  -  35)  Choose  the  odd  man
(d) Nitish Kumar : NDA
f rom  t he  w ords  given  below   in  t he  every
(e) Laloo Yadav : RJD
question.
Directions  (Q. 21  - 22)  Seperate  the odd man
28. (a) Anger (b) Grief
in the questions given below.
(c) Beauty (d) Joy
21. (a) Knesset (b) Drachama
(e) Humourous
(c) Guilder (d) Rial
29. (a) Iodine (b) Nitrogen
22. (a) Hockey (b) Chess
(c) Oxygen (d) Chloride
(c) Football (d) Cricket
(e) Hydrogen
23. Four out of the five options given below are
alike in some way and hence form a group. 30. (a) Gavaskar (b) Azaharuddin
Find out the option that does not belong to (c) Ravi Shastri (d) Venkat Raghwan
the group. (e) Anil Kumble
SSC
31. (a) Troupe (b) Crowd (e) WY
(c) Singer (d) Chorous 42. (a) LP (b) EV
(e) Gang (c) JQ (d) MN
32. (a) Manipuri (b) Kathaka (e) KP
(c) Bhangra (d) Bharatnatyam 43. (a) DE (b) PQ
(e) Odissi (c) TU (d) MO
33. (a) House (b) Floor (e) FG
(c) Roof (d) Door 44. (a) VT (b) MO
(e) Window (c) PR (d) DF
34. (a) Degree (b) Litre (e) FG
(c) Second (d) Watt 45. (a) TQ (b) MN
(e) Volume (c) HK (d) CF
35. (a) Hygrometer (b) Anemometer (e) WZ
(c) Odometer (d) Perimeter 46. (a) XW (b) FG
(e) Thermometer (c) ML (d) PO
Directions  (Q.    36  -  38)  Choose  the (e) TS
odd  word  pair  out  of  the  given  alternatives. 47. (a) TV (b) FG
36. (a) Chalk : Blackboard (c) PQ (d) QR
(b) Saw : Wood (c) Author : Book (e) BC
(d) Pen : Paper 48. (a) KP (b) MN
37. (a) Venus : Shukra (c) HR (d) GT
(b) Uranus : Indra (e) EV
(c) Mars : Mangal (d) Saturn : Budha 49. (a) TU (b) KJ
38. (a) Home : Room (b) Train : Engine (c) PO (d) YX
(c) Scooter : Gears (d) Curd : Milk (e) CB
(e) Atom : Electron 50. (a) NL (b) VT
Directions  (Q.    39  -  40)  Choose  the (c) RP (d) ZX
odd  word  from  the  options  given  for  each (e) JG
questions.
51. (a) BCD (b) NPR
39. (a) Wife
(c) KLM (d) RQP
(b) Father
(e) HGF
(c) Mother
52. (a) PRT (b) MOQ
(d) Grand father
(c) GEC (d) TVX
40. (a) Rakesh Sharma
(e) SUW
(b) Neil Armstrong
53. (a) DOG (b) DIN
(c) Yuri Gagrin
(c) OUT (d) FED
(d) Edmond Hillary
(e) JOT
Directions (41-75): In each of the following questions, 54. (a) BEH (b) CFI
five groups of letter are given. Four of them are alike (c) DGJ (d) EHL
in a certain way while one is different. Choose the odd (e) FIL
one. 55. (a) VWY (b) QRT
41. (a) BD (b) IK (c) LMO (d) JKL
(c) PN (d) SU
SSC
(e) DEG 74. (a) HSRI (b) FTGS
56. (a) EBA (b) XUT (c) DWEV (d) AZBY
(c) TQP (d) JFE 75. (a) USAGE (b) USUAL
(e) YVU (c) UKASE (d) URANE
57. (a) BHE (b) DJG 76. (a) Nest (b) Whole
(c) SYV (d) JPM (c) Roof (d) Stand/Port
(e) PUS 77. (a) Neptune (b) Uranus
58. (a) Monday (b) Tuesday (c) Pluto (d) Mercury
(c) Saturday (d) Thrusday 78. (a) Post (b) Telegraam
59. (a) IJCD (b) WYTS (c) Telephone (d) Electricity
(c) QRKL (d) PQMN 79.(a) Uncle-Niece (b) Husband-Wife
(e) ABLM
(c) Brother-Sister (d) Nephew-Uncle
60. (a) SUWYA (b) LJNPR
80.(a) Hide (b) Evolve
(c) KMOQS (d) BDFHJ
(c) Cover (d)Secret
(e) ACEGI
81.(a) Pistol (b) Sword
61. (a) jAnUaRy (b) mArCh
(c) Gun (d) Rifle
(c) mAy (d) oCtObEr
82.(a) Aeroplane (b) Bird
(e) dEcEmBeR
(c) Tanker (d) Parachute
62. (a) RNJ (b) XTP
83.(a) Geography (b) Physics
(c) MIE (d) ZWR
63. (a) PUT (b) END
(c) Chemistry (d) Biology
(c) OWL (d) ARM 84.(a) Krukshetra (b) Haldighati
64. (a) EBD (b) IFH (c) Sarnath (d) Panipat
(c) QNO (d) YVX 85.(a) March (b) December
65. (a) AEFJ (b) KOPT (c) July (d) September
(c) UYZD (d) EHIL 86. (a) Platform (b) Park
66. (a) MKGA (b) RPLF
(c) Bus’stand (d) Sea Port
(c) VTPJ (d) PNID
67. (a) JMPS (b) MPSL 87. (a) Yearly (b) Quickly
(c) DGJM (d) UXAD (c) Weekly (d) Monthly
68. (a) BFJQ (b) RUZG 88. (a) RNJH (b) SOKG
(c) GJOV (d) ILQX (c) QMIE (d) MIEA
69. (a) GECA (b) PNLJ
89. (a) DSFU (b) DGRI
(c) VUSQ (d) TRPN
(c) HRGQ (d) BUDW
70. (a) AEGC (b) HLNJ
(c) OSVQ (d) VZBX 90. (a) PXZD (b) QSBR
71. (a) AJKL (b) IBCD (c) RAEG (d) XYZA
(c) ORQP (d) UFGH 91. (a) Tank (b) Ocean
72. (a) DINS (b) CHNR (c) Well (d) Stand/Port
(c) BGLQ (d) AFKP
92. (a) WHEAT (b) PROUD
73. (a) PEAR (b) REAP
(c) TRAIN (d) DRIVER
(c) TORE (d) TEAR
SSC
93. (a) STUTTER (b) SURRENDER 97. (a) 25 (b) 37
(c) RESURRECT (d) SUCCEED (c) 99 (d) 57
94. (a) BCD (b) KLM 98. (a) 72 (b) 42
(c) NPR (d) HGF (c) 152 (d) 110
95. (a) 131 (b) 171 99. (a) 12 (b) 14
(c) 151 (d) 161 (c) 18 (d) 22
96. (a) 11-13 (b) 23-29 100. (a) 16-18 (b) 96-108
(c) 11-17 (d) 31-35 (c) 56-63 (d) 86-99

Answers
1. c 2. e 3. d 4. d 5. d 6. c
7. b 8. c 9. b 10. e 11. a 12. c
13. e 14. b 15. d 16. b 17. e 18. c
19. e 20. d 21. a 22. b 23. e 24. d
25. b 26. c 27. d 28. e 29. a 30. d
31. c 32. c 33. a 34. e 35. d 36. c
37. d 38. d 39. a 40. d 42. c 43. a
44. d 45. e 46. b 47. b 48. a 49. c
50. a 51. e 52. b 53. c 54. c 55. d
56. d 57. d 58. e 59. a 60. b 61. b
62. e 63. d 64. a 65. c 66. d 67. d
68. b 69. a 70. c 71. c 72. c 73. b
74. c 75. b 76. b 77. c 78. c 79. d
80. d 81. b 82. b 83. b 84. a 85. c
86. d 87. b 88. b 89. a 90. c 91. d
92. d 93. d 94. d 95. c 96. d 97. d
98. b 99. c 100. d
SSC
3. Number Series
What is Number Series? Series pattern: 22 - 1, 32 - 1, 42 - 1.....
Number serires is a sequence of digits or numbers or 9. (n2 + n) series
both made on certain pattern. Example: 2, 6, 12, 20, 30
Example: 4, 8, 16, 32, 64 Series pattern: 12 + 1, 22 + 2, 32 + 3.....
Series pattern: Every next number of the series 10. (n2 - n) series
is double the previous number. Example: 120, 143, 168, 195, 224
Formats of the number series Series pattern: 112 - 1, 122 - 1, 133 - 1.....
1. Addition series 11. n3 series
Example: -10, 0, 10, 20, 30, 40 Example: 1, 8, 27, 64, 125
Series pattern: 10 is added to evey next number. Series pattern: 13, 23 , 33, 43...........
2/ Substraction series 12. (n3 + 1) series
Example: 30, 20, 10, 0, -10, -20 Example: 1, 2, 9, 28, 65
Series  pattern: 10 is substracted from every Series pattern: 03 + 1, 13 + 1, 23 + 1.......
element to get the next element. 13. (n3 - 1) series
3. Multiplication series Example: 0, 7, 26, 63, 124, 215
Example: 10, 50, 250, 1250 Series pattern: 13 - 1, 23 - 1, 33 - 1.......
Series pattern: Each number is multiplied by 5 14. (n3 + n) series
to get the next element. Example: 1342, 1740, 2210, 2758
4. Division series Series pattern: 113 + 11, 123 + 12, 133 + 13.......
Example: 250, 50, 10, 2, 0.4 15. (n3 - n) series
Series pattern: Each number is divided by 5 to Example: 120, 210, 336, 504, 720
get the next number. Series pattern: 53 - 5, 63 - 6, 73 - 7.......
5. Prime number series 16. Digits sum series
Example: 3, 5, 7, 11, 13, 17 19, 23 Example: 043, 52, 007, 223, 700
Series pattern: This is consecutive prime num- Series pattern: The sum of digits of each is 7.
ber series. 17. Digits difference series
6. n2 series Example: 41, 85, 52, 96, 74
Example: 4, 16, 36, 64, 81 Series pattern: Digits difference of each number
Series  pattern: Given series is the square of is 3.
consecutive even numbers 18. Digits multiplication series
7. (n2 + 1) series Example: 46, 38, 262, 432, 83
Example: 122, 145, 170, 197, 226 Series pattern: In each number, product of digits
Series pattern: 112 + 1, 122 + 1, 132 + 1.......... in 24.
8. (n2 - 1) series Note
Example: 3, 8, 15, 24, 35 Apart from above mentioned series formats, students
may come across some other miscelleneous patterns.
Exercise
Directions (Q.  1 - 12): Find the missing term/terms (a) -1, 5 (b) 1, 4
in the series given below: (c) 1, -5 (d) -2, -5
1. 14, 15, 19,  ........, 44, 69 4. 388, 373, ........, 298, 238, 163
(a) 20 (b) 28 (a) 299 (b) 341
(c) 35 (d) 32 (c) 295 (d) 343
2. 325, 261, 197, ............., 69 5. 18, 17, 13, .........., -12, -37
(a) 100 (b) 81 (a) 4 (b) -10
(c) 133 (d) 143 (c) 6 (d) 16
3. -5, -3, ......1, 3, ........... 6. 5, 10, 15, ......, 25, 30
SSC
(a) 10 (b) 16 (c) 85 (d) 68
(c) 18 (d) 20 18. 18, 27, 49, 84, 132, ?
7. 7, 11, ...... 17, 19, 23 (a) 196 (b) 183
(a) 13 (b) 14 (c) 193 (d) 178
(c) 15 (d) 16 19. 15, 21, 39, 77, 143, ?
8. 10.4, ......., 11.15, 11.9, 12.9, 14.15 (a) 245 (b) 238
(a) 13.15 (b) 10.65 (c) 181 (d) 253
(c) 5 (d) 11 Directions (Q. 20 - 25): Find the wrong term in the
following number series:
9. ........., 9000, 1794, 352.8, 64.56, 6.912
20. 16, 4, 2, 1.5, 1.75, 1.875
(a) 43030 (b) 42420
(a) 1.875 (b) 1.75
(c) 35350 (d) 45030
(c) 1.5 (d) 2
10. -4, -5, 0, ....., 4, 5, 8, 10, 12, 15
(e) 4
(a) 4 (b) 2
21. 4, 2, 3.5, 7.5, 26.25, 118.125
(c) 3 (d) 0
(a) 118.12 (b) 26.25
11. 6, 13, 25, ......, 101, 203
(c) 3.5 (d) 2
(a) 30 (b) 51
(e) 7.5
(c) 70 (d) 90
22. 7, 9, 16, 25, 41, 68, 107, 173
12. 0, 2, 3, 5, 8, 10, 15, 17, 24, 26, .......
(a) 107 (b) 16
(a) 35 (b) 42
(c) 41 (d) 68
(c) 33 (d) 34
Directions (Q.  13 - 19): Replace the questions marks (e) 25
in the following number series with one of the given 23. 16, 19, 21, 30, 46, 71, 107
options.
(a) 19 (b) 21
(c) 30 (d) 46
13. ,  , ?,  , 
(e) 71
24. 2, 11, 38, 197, 1172, 8227, 65806
(a) (b) (a) 11 (b) 38
(c) 197 (d) 1172
(c) (d) (e) 8227
25. 2, 7, 30, 138, 524, 1557, 3102
14. 20, 20, 19, 16, ?, ?, 14, 11, 10, 10
(a) 138 (b) 529
(a) 17, 13 (b) 10, 11
(c) 3102 (d) 7
(c) 15, 13 (d) 13, 17
(e) 2
15. 44, 182, ?, 82, 28
Directions (Q.  26 - 31) What will come in place of
(a) 36 (b) 61 question mark (?) in the following number series?
(c) 812 (d) 151 26. 512, ?, 1000, 1331, 1728, 2197
16. 16, 24, 60, 210,  ?, 51975, 33783.75 (a) 750 (b) 599
(a) 60 (b) 945 (c) 729 (d) 668
(c) 5197.5 (d) 33783.75 27. 12, 16, 24, 40, ?
17. 19, 26, 40, ?,  124, 236 (a) 72 (b) 78
(a) 65 (b) 78 (c) 76 (d) 79
SSC
28. 8, 17, 42, 91, ? Directions (41-65): In each of the following letter series,
(a) 181 (b) 178 some of the letters are missing which are given in that order
(c) 195 (d) 172 as one of the alternatives below it. Choose the correct
alternative.
29. 9, 19, 39, ?, 159
41. cc – ccbc – accbcc – c – b
(a) 85 (b) 79
(1) acac (2) abac
(c) 91 (d) 78
(3) abab (4) aabc
30. 3840, 960, 240, 60, ?
(a) 11 (b) 18
42. – aa – ba – bb – ab – aab
(c) 25 (d) 15
(1) babab (2) aaabb
31. 7, 8, 18, 57, ?
(3) bbaab (4) bbbaa
(a) 232 (b) 435
(c) 157 (d) 268
Directions (Q. 32 - 35) Find the missing term in the 43. l – n – ml l m – n – l
following series: (1) mnmn (2) mnnm
32. 748, 737, ......, 682, 638, 583 (3) mnmm (4) nmmn
(a) 675 (b) 715
(c) 618 (d) 685 44. – bbm – amb – m – a – bb
33. 3, 23, 43, 63, 83, ........ (1) mbabm (2) abmab
(a) 95 (b) 18 (3) mabam (4) ambbm
(c) 115 (d) 103
34. 50, 26, 14, ........., 5, 3.5 45. – stt – tt – tts –
(a) 6 (b) 5 (1) tsst (2) sstt
(c) 8 (d) 12 (3) ttst (4) tsts
35. 121, 117, 108, 92, 67, ........
(a) 45 (b) 31 46. ac – ga – eg – ce -
(c) 28 (d) 42 (1) dbag (2) ecag
Directions  (Q.  36  -  40)  In  the  following  number (3) deag (4) ebdg
series, a wrong number is given. Find out that wrong
number. 47. a – n – b - - n c b - - n c b
36. 3, 7, 16, 35, 72, 153, 312 (1) bcabab (2) bacbab
(a) 16 (b) 153 (3) abcbcb (4) abbbcc
(c) 35 (d) 72 48. – – aba – – ba – ab
(e) 7 (1) abbba (2) abbab
37. 6, 10, 14, 34, 66, 130, 258 (3) baabb (4) bbaba
(a) 130 (b) 10 49. ab - - - b – bbaa –
(c) 14 (d) 34 (a) abaab (b) abbab
(e) 66 (c) baaab (d) babba
38. 93, 309, 434, 498, 521, 533 50. – baa – aab – a – a
(a) 309 (b) 521 (a) aabb (b) aaba
(c) 93 (d) 533 (c) abab (c) baab
(e) 498 51. - - ba bbba – a - -
39. 2, 6, 24, 96, 285, 568, 567 (a) ababb (b) baaab
(a) 24 (b) 567 (c) bbaba (d) babbb
(c) 96 (d) 568 52. aa – ab - - aaa – a
(e) 2 (a) aaab (b) aabb
40. 6, 15, 35, 77, 165, 221 (c) abab (d) baaa
(a) 77 (b) 221 53. c- bbb - - abbbb – abbb-
(c) 35 (d) 6 (a) aabcb (b) abccb
(e) 165 (c) abacb (d) bacbb
SSC
54. b – abbc – bbca – bcabb – ab (c) 81 (d) 01
(a) acaa (b) acba 70. 1, 6, 13, 22, 33, (...?...)
(c) cabc (d) cacc (a) 44 (b) 45
55. ac – cab – baca – aba – acac (c) 46 (d) 47
(a) aacb (b) acbc 71. 3, 9, 27, 81, (...?...)
(c) babb (d) bcbb (a) 324 (b) 243
56. – acca – ccca – acccc – aaa (c) 210 (d) 162
(a) acca (b) caaa 72. 1, 9, 17, 33, 49, 73,(...?...)
(c) ccaa (d) caac (a) 97 (b) 98
57. – bc - - bb – aabc (c) 99 (d) 100
(a) acac (b) babc 73. 2, 5, 9, (...?...), 20, 27
(c) abab (d) aacc (a) 14 (b) 16
58. a – bbc – aab – cca – bbcc (c) 18 (d) 24
(a) bacb (b) acba 74. 5, 9, 17, 29, 45, (...?...)
(c) abba (d) caba (a) 60 (b) 65
59. ab – aa- bbb – aaa – bbba (c) 68 (d) 70
(a) abba (b) baab 75. 1, 6, 15, (...?...), 45, 66, 91
(c) aaab (d) abab (a) 25 (b) 26
60. bc – b – c – b – ccb (c) 27 (d) 28
(a) cbcb (b) bbcb 76. 3, 7, 15, 31, 63, (...?...)
(c) cbbc (d) bcbc (a) 92 (b) 115
61. abb – baa – a – bab – aba (c) 127 (d) 131
(a) abba (b) abab 77. 1, 2, 3, 5, 8, (...?...)
(c) ccac (d) aabb (a) 9 (b) 11
62. abca – bcaab – ca – bbc – a (c) 13 (d) 15
(a) ccaa (b) bbaa
1 2 1
(c) abac (d) abba 78. 33 , 20 ,14 , ?,9
63. c – bba – cab – ac – ab – ac
3 7 11
(a) abcbc (b) acbcb 5
(c) babcc (d) bcacb a. 14 b. 5
17
64. a – bc - c – abb – bca –
(a) cccbc (b) cbbac 5 1
(c) accba (d) abbba c. 5 d. 11
19 9
65. – c – bd – cbcda – a – db – a
(a) adabcd (b) cdbbca 79. 8, 15, 28, 53, (...?...)
(c) daabbc (d) bdbcba (a) 106 (b) 98
Directions: In Questions No. 1 to 50, find the missing (c) 100 (d) 102
number from the given responses. 80. 1, 0, 5, 8, 17, 24, 37, (...?...)
66. 1, 4, 9, 16, 25, (...?...)
(a) 40 (b) 43
(a) 35 (b) 36
(c) 46 (d) 48
(c) 48 (d) 49
81. 6000, 5940, 5881, ?
67. 20, 19, 17, (...?...), 10, 5
(a) 12 (b) 13 (a) 5823 (b) 5746
(c) 14 (c) 15 (c) 5854 (d) 5788
68. 2, 3, 5, 7, 11, (...?...), 17 82. 87, 90, 84, 88, 81, (...?...), (...?...)
(a) 12 (b) 13 (a) 85, 93 (b) 86, 98
(c) 14 (d) 15
(c) 86, 78 (d) 86, 68
69. 6, 11, 21, 36, 56 (...?...)
(a) 42 (b) 51 83. 2, 12, 36, 80, 150, (...?...)
SSC
(a) 194 (b) 210 (a) 8311 (b) 8312
(c) 252 (d) 258 (c) 8509 (d) 8515
94. 1, 9, 25, 49, ?, 121 92. 22, 24, 28, ?, 52, 84
(a) 81 (b) 91 (a) 46 (b) 36
(c) 64 (d) 100 (c) 3 (d) 42
93. 3, 7, 23, 95, ?
2 3 4 5
85. , , , , (.....? ....) (a) 575 (b) 479
5 5 5 5 25 (c) 128 (d) 62
6 6 94. 3, 15, ?, 63, 99, 143
(a) (b) (a) 27 (b) 45
5 5 25 5
(c) 35 (d) 56
6 7 95. 3, 28, 4, 65, 5, 126, 6, ?
(c) (d)
125 25 (a) 215 (b) 216
86. 2, 3, 6, 15, 42, ? (c) 217 (d) 218
(a) 84 (b) 123 96. 3691, 6931, 9361, 3691, ?
(c) 94 (d) 60 (a) 1369 (b) 6931
87. 9, 17, 33, 65, ? (c) 1963 (d) 3961
(a) 99 (b) 117 97. 19, 23, 26, 30, 33, ?
(c) 119 (d) 129 (a) 31 (b) 35
88. 6, 17, 39, 72, ? (c) 37 (d) 39
(a) 94 (b) 127 98. 240, 120, 60, ?, 15
(c) 8 (d) 116 (a) 20 (b) 29
89. 138, 161, 185, 210, ? (c) 30 (d) 10
(a) 240 (b) 236 99. 1, 3, 7, 13, 21, ?
(c) 272 (d) 229 (a) 27 (b) 29
90. 2460, 3570, 4680, ? (c) 31 (d) 33
(a) 8640 (b) 5670 100.133, 119, 105, 91, ?
(c) 5970 (d) 5790 (a) 87 (b) 77
91. 7714, 7916, 8109, ? (c) 85 (d) 79
Answers
1. b 2. c 3. a 4. d 5. a 6. d
7. a 8. b 9. d 10. d 11. b 12. a
13. c 14. a 15. c 16. b 17. d 18. c
19. a 20. b 21. c 22. d 23. a 24. d
25. a 26. c 27. a 28. d 29. b 30. d
31. a 32. b 33. d 34. c 35. b 36. d
37. c 38. b 39. a 40. e 41. a 42. c
43. b 44. c 45. a 46. b 47. a 48. b
49. c 50. c 51. d 52. a 53. b 54. c
55. a 56. b 57. a 58. b 59. b 60. a
61. a 62. c 63. b 64. c 65. a 66. b
67. c 68. b 69. c 70. c 71. b 72. a
73. a 74. b 75. d 76. c 77. c 78. d
79. d 80. d 81. a 82. c 83. c 84. a
85. b 86. b 87. d 88. d 89. b 90. d
91. a 92. b 93. b 94. c 95. c 96. b
97. c 98. c 99. c 100. b
SSC
4. Letter Series & Grouping
What is Letter Series? ter positions but it may also be based on back-
Letter series is a sequence of elements made of letters ward order letter positions.
from English alphabet. Such series follow a certain (ii) Apart from addition of positions of letters, series
pattern can be based on substraction, multiplication and
Example: P, R, T, V, X division of letter positions.
Series  pattern: Every next letter in the series (vi) Small letter based series
takes place two steps forward. Example: Complete the following series with one
Keep in Mind (From chapter 3, coding-decoding) of the given options.
(i) Positions of letters (Left to right) ab_d_aaba_na_badna_b
(ii) Positions of letters (Right to left) (a) andaa (b) babda
(iii) Opposite letters (c) badna (d) dbanb
Formats of the series (e) None of these
(i) Forward order series Series pattern:
Example: D, G, J, M, P, S, V ab/adna/ab/adna/ab/adna/ab
Series pattern: Every next letter takes place 3 Clearly, option (a) is the correct answer.
steps forward. Note
(ii) Backward order series Apart from above mentioned patterns of letter
Example: U, R, O, L, I series, students must be prepare to face some sur-
prising patterns.
Series pattern: Every next letter takes place 3
steps backward. What is Letter       Grouping?
(iii) Multiple series Grouping of letters means classification of letters in
which a single or more letter are given in each of four/
Example: C, V, D, W, E, X, F, Y
five options. All there given options except one are
Series pattern: somehow similar on the basis of some common prop-
erty and hence they form a group. But the option which
is exception is called the odd man and this is the odd
man which is our answer.
Example: L, B, A, C, X
Explanation: Clearly, all the given letters except ‘A’
(iv) Opposite letter series
are consonants & hence they form a group. But
Example: AZ, BY, CX, DW, EV ‘A’ which is a vowels does not belong to the group
Series pattern: Each element of the series is made of consonants and therefore, it will be considered
of two opposite letters. as an odd man in the group.
(v) Series based on letter position Formats of the questions
Example: EF, CH, AJ, DG, BI (i) Grouping based on vowels and consonants
Series pattern: In each term, the sum of letter Example: Choose the odd man
positions is equal to 11. (a) E (b) D
Let us see: (c) M (d) P
(e) X
Explanation: Here, all except ‘E’ are consonants.
Note: Hence, option (a) is correct.
(i) The above example is based on forward order let- (ii) Grouping based on shapes
SSC
Example: Find out the odd letter Explanation: Option (a) is correct as except ‘PQRS’
(a) J (b) R all others have a difference of one letter between
(c) D (d) T first and second letter.
(e) P (v) Grouping based on capital and small letters
Explanation: Here, all except ‘T’ are made with the Example (A) : Take out the odd man.
combination of straight and curved lines. Hence, (a) PqrS (b) IJKL
option (d) is correct. (c) Tuvl (d) EfgH
(iii) Opposite letters based grouping (e) lmNO
Example: Which of the following is the odd Explanation: Option (b) is correct as except ‘IJKL’
man? all others have two small and two capital letters.
(a) PR (b) GT Example (B) : Select an odd man out of the following
(c) VE (d) MN options.
(e) HS (a) PSqr (b) IjkL
Explanation: Here, except ‘PR’ all others are groups (c) EFgH (d) TuvW
of opposite letters. Hence, option (a) is correct. (e) XgzA
(iv) Letter difference based gruoping Explanation: Option (a) is correct as except ‘PSqr’
Example (A) : Find the odd letter group all others have capital letters at ends.
(a) EF (b) HJ (vi) Grouping  based  on  meaningful  and  non
(c) KM (d) XZ meaningful words
(e) AC Example : Find the odd man.
Explanation: Option (a) is correct as except ‘EF’ all (a) RUN (b) AUN
others have a difference of one letter between (c) SUN (d) GUN
them. (e) PUN
Example (B) : Which of the following is an odd man? Explanation: Option (b) is correct as except ‘AUN’
(a) PQRS (b) KMNO all others are meaningful words.
(c) FHIJ (d) ACDE
(e) VXYZ

Exercise
Directions (Q.  1 - 6): Fill the blank space/spaces to (a) U (b) T
complete the letter series.
(c) V (d) W
1. J, M, ......., S, ........, Y
¼e½ Z
(a) N, V (b) P, V
5. O, ......, P, J, ....., I, R, H, ......, G
(c) V, P (d) P, R
(a) K, Q, T (b) K, Q, S
¼e½ P, X
(c) K, Q, U (d) K, Q, V
2. P, M, J, G, D, ......
(e) None of these
(a) B (b) C
6. BZA, DYC, FXE, ......, JVI
(c) E (d) A
(a) HWG (b) HUG
(e) None of these
(c) WHG (d) UHG
3. D, P, F, R, H, ......, J, V
(e) None of these
(a) S (b) I
Directions (Q.  7 - 10): Replace the question mark
(c) K (d) T (s) in the given series.
(e) None of these 7. A, C, F, J, ?, ?
4. D, G, J, M, P, S, ......... (a) O, U (b) R, V
SSC
(c) M, U (d) L, P Directions  (Q.    21  -  25):  In  each  of  the  following
(e) None of these letter series, some of the letters are missing which
8. BMO, EOQ, HQS, ? are  given  in  that  order  as  one  of  the  alternatives
below it. Choose the correct alternative.
(a) SOW (b) SOV
21. a_ba_b_b_a_b
(c) LMN (d) KSU
(a) abaab (b) abbab
(e) None of these
(c) aabba (d) bbabb
9. AB, DEF, HIJK, ?, STUVWX
(a) LMNOP (b) QRSTU (e) None of these
(c) MNOPQ (d) LMNO 22. _c_bd_cbcda_a_db_a
(e) None of these (a) adabcd (b) bdbcba
10. ?, D, G, ? P, V (c) cdbbca (d) daabcc
(a) B, V (b) B, K (e) None of these
(c) K, L (d) B, M 23. a_c_abb_a_bc_bc_ab
(e) None of these (a) cbcaaa (b) bcccab
Directions (Q.  11 - 20): Find the odd man. (c) bccaac (d) acbabc
11. (a) L (b) Q (e) None of these
(c) O (d) S 24. c_ac_aa_aa_bc_bcc
(e) U (a) cabba (b) ccbbb
12. (a) HS (b) BY (c) bbbbb (d) cbacb
(c) MN (d) IP (e) None of these
(e) PK 25. a_ca_bc_bcc_bca
13. (a) TUV (b) LMN (a) bbaa (b) bbab
(c) BCD (d) PRS (c) aabb (d) baba
(e) XYZ (e) None of these
14. (a) VRT (b) MIK Directions (Q.  26 - 29): Fill the blank space/spaces
(c) FBD (d) YUW in the following letter series.
(e) RMP 26. Z, ......., U, Q, L, F
15. (a) QN (b) WU (a) V (b) Y
(c) MK (d) GE (c) W (d) X
(e) XV (e) None of these
16. (a) ZGPKU (b) FRGSP 27. C, F, ......, L, O, R, U
(c) NEXFL (d) LANCP (a) K (b) G
(e) MTIXZ (c) H (d) I
17. (a) AUgPZ (b) MXiDV (e) None of these
(c) KFeCO (d) yGLhT 28. L, LMN, LMNOP, ................, LMNOPQRST
(e) UHmQY (a) LMNOPRQ (b) LMNOPQR
18. (a) DnUZ (b) LPuB (c) LMONPQR (d) LNMOPQR
(c) FiMy (d) UXeN (e) None of these
(e) WaQS 29. WFB, TGD, QHG, ?
19. (a) OTP (b) MBM (a) NIJ (b) NIK
(c) SZX (d) UVB (c) NJK (d) OIK
(e) YQR ¼e½ PJK
20. (a) CFIL (b) QTWZ Directions  (Q.    30  -  32):  Replace  the  question
(c) ORUX (d) JMPS mark(s) in the following number series.
(e) PSVX 30. C, Z, F, X, I, V, L, T, O , ?, ?
(a) O, P (b) P, Q
SSC
(c) R,R (d) S, R 35. c_baa_aca_cacab_acac_bca
(e) None of these (a) acbaa (b) bbcaa
31. UPI, ?, ODP, MBQ, IAW (c) bccab (d) cbaac
(a) RHJ (b) SHJ (e) None of these
(c) SIJ (d) THK Directions  (Q.  36  -  40):  Find  the  odd  man  in  the
(e) None of these question given below.
32. O, J, M, L, ?, N, I, P, G, R, E, T 36. (a) WY (b) SU
(a) K (b) M (c) PN (d) BD
(c) T (d) O (e) IK
(e) None of these 37. (a) GJMQVX (b) NWMBHJ
Directions  (Q.  33  -  35):  In  each  of  the  following (c) XGKNTY (d) DXCLQZ
letter series, some of the letters are missing which (e) PFZUBM
are  given  in  that  order  as  one  of  the  alternatives 38. (a) IR (b) XC
given below it. Choose the correct alternative. (c) HP (d) EV
33. _ _aba_ _ba_ab (e) LO
(a) abbba (b) abbab 39. (a) BDW (b) DFU
(c) baabb (d) bbaba (c) FHS (d) GIQ
(e) None of these (e) IKP
34. ba_cb_b_bab_ 40. (a) HOT (b) LOT
(a) acbb (b) bacc (c) NOT (d) FOT
(c) bcaa (d) cabb (e) POT
(e) None of these

Answers
1. b 2. d 3. d 4. c 5. b 6. a
7. a 8. d 9. c 10. b 11. a 12. d
13. d 14. e 15. a 16. b 17. d 18. a
19. b 20. e 21. d 22. a 23. c 24. b
25. a 26. d 27. d 28. b 29. b 30. c
31. b 32. a 33. b 34. b 35. a 36. c
37. e 38. c 39. d 40. d
SSC
5. MIXED SERIES
What is Mixed Series? (iii) Mixed Sequence (Forward & backward both)
Such series is a sequence of many elements made of pattern
numbers and letters arranged from left to right. In some Example: E, 45, I, 37, M, 29
cases some symbols may take place in such series. The Series pattern: Letters and numbers take place
symbols may be '+', '-', 'x', '÷', ?. '>', '<', = etc. In other alternately. Every next letter is four letters for-
words we can say that a mixed series is sequence of ward from the previous letter and every next num-
diverse elements. ber decreases by 8.
Formats of the mixed series (iv) Miscellaneous pattern
(i) Forward sequence pattern Example: 25, M, 16, O, 25, R, 16, V
Example: E, -3, G, 1, I, 5 Series pattern: Letters and numbers take place
Series pattern: Letters and numbers take place alternately. Every next letter takes place by
alternately. Every next letter is two letters ahead skipping 1, 2 and 3 letters respectively as diagram
from the previous letter and every next number given below.
increases by 4.
(ii) Backward sequence pattern
Example: T, 3, R, 2, P, 1, N, 0, L, -1, J, -2
Series pattern: Letters and numbers take place
If we see the numbers, we find that they decreases
alternately. Every next letter is two letters
by 9 and increase by 9 alternately.
backward from the previous letter and every next
number decreases by 1.
Exercise
Directions (Q.  1 - 20): What comes in place of 7. 5DE2S,  5MNP2,  5TXY9,  ..............
blank space/? (a) VQ6KR (b) 5HG7M
1. 117, J, 123, M, 129, P, 135, ......, 141 (c) 8RTN5 (d) 4BT6K
(a) 138 (b) 136 8. 19MSP,   5CDIT,   NP6QE,   ?,   4BZAD
(c) R (d) S (a) 3LXYN (b) CDITF
2. 8, S, 3, R, -2, Q, -7, P, ........... (c) KPGMS (d) MPQRT
(a) -14 (b) 12 9. 156, X, 160, U, 164, R  ?
(c) 14 (d) -12 (a) 170 (b) 165
3. ........  Q, 10, L, 6, H, 3, E, 1, C, 0, B (c) 175 (d) 168
(a) 15 (b) 18 10. -4, P, ......, R, 2, T, 5
(c) 16 (d) 22 (a) -3 (b) -1
4. 12, E, 8, G, 4, J, 0,  N, ......., S, -8 (c) 2 (d) 5
(a) -3 (b) -5 11. 1CV, 5FU, 9IT, ?, 17OR
(c) -4 (d) -2 (a) 5FU (b) 13LS
5. 225, T, 234, R, ...... , P, 252 (c) 9IT (d) 17OR
(a) 250 (b) 243 12. E, 211, G, 212, I, 214, K, 217, M, .......
(c) 241 (d) 238 (a) 218 (b) 223
6. D, -2, E, -1, G, 1, J, 4, N,  ........ (c) 220 (d) 221
(a) 9 (b) 8 13. 172 , V, 179, X, ........ Z, 193
(c) 10 (d) 11 (a) 189 (b) W
SSC
(c) Y (d) 186 23. .......  F, 1, J, -6, N, -13, R, -20
14. D, ?, G, 2, J, 6, M,  10, P (a) 5 (b) 9
(a) -2 (b) 1 (c) 8 (d) 7
(c) -5 (d) 2 24. L, 2, ..... , 8, Q, 32, U  ....
15. 2÷P,  2+L,   2Bx,  2<9,  ? (a) M, 120 (b) N, 125
(a) 4÷P (b) 5+N (c) M, 130 (d) N, 128
(c) 2T= (d) E<3 25. O  -5, K, -1, .......,  3, F, 7, E
16. W-144, ?, S-100, Q-81, O-64 (a) H (b) G
(a) U-121 (b) U-122 (c) 1 (d) 2
(c) V-121 (d) V-128 26. E, -6, I, .......,  M, 2, Q, 6
17. 98, J, 89, H, ........,  F, 71, ....... (a) 2 (b) -2
(a) 85, G (b) 80, D (c) -4 (d) 4
(c) G, 85 (d) D, 80 27. R, -8, O, 0, K, 8, H,  ...... , D, 24
18. B, 8, D, 9, G, 17, K, 44, P, 108, ....... (a) 22 (b) 16
(a) 220 (b) 110 (c) 19 (d) 18
(c) 225 (d) 233 28. 2B, 4C, 8E, 14H
19. 101, O, 100, N, 96, M, 87, L, ....... (a) 16K (b) 20I
(a) 72 (b) 81 (c) 20L (d) 22L
(c) 99 (d) 71 29. 2, A, 9, B, 6, C, 13, D, ?
20. .......  115, U, 110, R, 105, O (a) 9 (b) 10
(a) Y (b) W (c) 12 (d) 19
(c) X (d) V 30. Q1F, S2E, U6D, W21C, ?
Directions (Q.  21 - 32): What comes in place (a) Y44B (b) Y66B
of blank space/? (c) Y88B (d) Z88B
21. E, 25, I, 16 ....... 7, Q 31. N5V, K7T, ? E14P, B19N
(a) P (b) M (a) H9R (b) H10Q
(c) N (d) O (c) H10R (d) I10R
22. T÷6P, 5BA+, .........,  3Y-H,  Dx82 32. -5, S, 0, P, 5, ?, 10
(a) 4TB (b) CN÷ (a) T (b) M
(c) +EL6 (d) V+7 (c) Q (d) L

Answers
1. d 2. d 3. a 4. c 5. b 6. b
7. b 8. a 9. d 10. b 11. b 12. d
13. d 14. a 15. c 16. a 17. b 18. d
19. d 20. c 21. b 22. c 23. c 24. d
25. a 26. b 27. b 28. d 29. b 30. c
31. c 32. b
SSC
6. CODING DECODING
Coding GT G T  Road (Built by Shershah)
Coding means hiding the actual meaning of letter’s/ FU U F (mQ in hindi)
word(s) in a particular way. EV Remember  EV  of EVM (Electronic Voting
Decoding Machine)
It means making out the actual message that is dis- DW Remember D W of DEW
guised in a particular method of coding. CX Remember it as it is
Important for Coding-Decoding B Y Remember the word ‘By’
1.  Position of letters (Left to Right) AZ Remember it as it is
Formats of the questions
1. Coding by Addition
Example : If ‘EDEFF’ is coded as ‘GGHIH’ then
find the code for ‘NLMNT’.
(a) VQPOP (b) POPQV
(c) POPVQ (d) OPPVQ
2.  Position of letters (Right to Left)
(e) None of these
Explanation: Option (b) is correct. Let us see:-

Remember
Backward order (Right to Left) position of a particular 2. Coding by Substraction
letter Example : If ‘TTMLL’ is coded as ‘OSLKG’ then
find the code for ‘RPEDP’.
= 27 - forward order position of that letter
Backward order position of L (a) MODCK (b) MDOCK
= 27 - forward order position of L (c) MODKC (d) MOCKD
= 27 - 12 = 15 (e) None of these
Similarly, Explanation: Option (a) is correct. Let us see:-
Backward order position of E = 27 - 5 = 22
Backward order position of R = 27 - 18 = 9
3. Opposite letters

3. Coding by Addition and Substraciton both
Trick to Remember Example : If a certain code language  ‘FPLL’ is
MN Remeber M N of MAN coded as ‘AUGQ’ then how will ‘HJNN’ be coded.
LO Remeber L O of LOVE (a) CISO (b) CIOS
KP Kevin Piterson (English cricketer) (c) COIS (d) CSIO
JQ Jack & Queen (in the game of cards) (e) None of these
IR International Relation Explanation: Option (c) is correct. Let us see:-
HS Higher Secondary
SSC

Note:
(i) In place of letter positions in forward order,
4. Coding by Forward Order Positions of Letters
backward order position may be put for such
Example : If code for ‘LEFN’ = 12 5 6 14 then operations.
code for ‘HLGQP’ = ‘?’.
(ii) The above code can be in a single digit also if code
(a) 8 7 12 17 61 (b) 8 7 12 17 16 for PSOD = 9
(c) 8 2 17 17 16 (d) 8 12 7 17 16 Let us see:-
(e) None of these
Explanation: Option (d) is correct. Let us see:-

5. Coding by Backward Order Positions of Letters 8.  Opposite Letters Coding
Example  :  If    code for ‘DEF  =  23  22  21’  then Example : If  in a certain code language ‘TSZX’
‘JZMA = ?’ is written as ‘GHAC’ then how will ‘PTMB’ be
(a) 17 1 14 26 (b) 17 1 14 62 written in that language?
(c) 17 11 62 4 (d) 17 14 12 6 (a) KYGN (b) KGYN
(e) None of these (c) KNGY (d) KGNY
Explanation: Option (a) is correct. Let us see:- (e) None of these
Explanation: Option (d) is correct as every letter
is replaced by their opposite letters.
9. Fictious Language Coding
Example : If  in a certain code language ‘India is
winner’ is written as ‘ba ga ra’ and ‘beautiful
6.  Coding by Reversing Letters
India’ is written as ‘ba ka’ then find the code for
Example : If  code for ‘NATURE’ is ‘ERUTAN’ ‘India’?
then find the code for ‘REASON’
(a) ka (b) ga
(a) NSOAER (b) NOSEAR
(c) ra (d) ba
(c) NOSAER (d) NOASER
(e) None of these
(e) None of these
Explanation: Option (d) is correct. Let us see:-
Explanation: Option (c) is correct as coding has
been done by writing the words ‘NATURE’ and is winner = ga ra
‘REASON’ is reverse order. beautiful = ka
7. Coding by Positions of Letters and arithmetical
clearly, code for ‘India’ = ba
operations (+, -,  ,  )
Note: You may see digits/letters/symbols in place of
Example : If  code for ‘PSOD = 54’ then code for
fictious language.
‘NQMBG = ?’
10. Coding by Words in Chain
(a) 47 (b) 53
Example : If  ‘English’ is called ‘Maths’; ‘Maths’
(c) 43 (d) 66
is called ‘History’; ‘History’ is called ‘Hindi’;
(e) None of these ‘Hindi’ is called ‘Political Science’ then which
Explanation: Option (b) is correct. Let us see : subject contains story of mughal king Akbar?
SSC
(a) Maths (b) History
(c) English (d) Hindi
(e) None of these
Explanation: Option (d) is correct as Akbar’s story Clearly,
is the story of ‘History’ but here ‘History’ is called
‘Hindi’. Example 4: If A = 2; N = 28; Y = 50 and ANY =
Some other formats of the questions 80 then BET = ?
Example 1: If in certain language, ‘POPULAR’ (a) 45 (b) 54
is coded as ‘MBSVQPQ’ then what will be the (c) 65 (d) 38
code for ‘TORTURE’ in that language. (e) None of these
(a) VSFUUPS (b) VFSUUPS Explanation: option (b) is correct as each letter
(c) VSFUUSP (d) VSUFUPS has been coded with 2 (the forward order letter
(e) None of these positions).
Explanation: option (a) is correct. Let us see:- Let us see:-
Code for B = 2 2 = 4
Code for E = 5 2 = 10
Code for T = 20 2 = 40
ANY = 2 + 28 + 50 = 80
BET = 4 + 10 + 40 = 54
Note: (i) Such coding can also be done backwardly. Example 5: Directions (Q i - iv): These questions
(ii) You may see other patterns of such format. are based on a certain code language utilises letters
Example  2:  If  in certain code language, ‘A’  is in English alphabet. In each question a word is
coded as ‘4’; ‘B’ is coded as ‘5’; ‘C’ is coded as written in capital letters with one letter underlined.
‘6’; and so on, then code for ‘BCCFABF = ?’ For each letter of that word there is a code written
(a) 5 6 9 6 4 5 (b) 5 6 6 9 4 9 5 in small letters. That code is denoted by (a), (b),
(c) 5 6 6 9 9 5 4 (d) 5 6 6 9 4 5 9 (c), (d) or (e), though not in the same order. You
have to find the exact code for the underline letter
(e) None of these
of word. Please note that same letters appearing
Explanation: option (d) is correct. Let us see:- iin different words may be coded differently.
According to the question (i) V O W E L
(a) t (b) s
(c) r (d) h
(e) j
(ii) C A R R Y
(a) t (b) m
Note: In such question the letters may also be coded
(c) y (d) w
by letters/symbols.
(e) f
Example 3: In a certain code, ‘DESK’ is written
as ‘# 52’; ‘RIDE’ is written as ‘%7# ’. How is (iii) R E A D Y
‘RISK’ written in that code? (a) d (b) d
(a) %725 (b) %752 (c) g (d) h
(c) %7#2 (d) %7 # (e) s
(e) None of these (iv) R I G H T
Explanation: option (b) is correct. Let us see:- (a) fk (b) ot
(c) qv (d) ej
SSC
(e) di (B) If the first as well as the last digit is odd, both are
Explanation: to be coded by ‘X’.
(i) option (e) is correct as vowels are coded five places (C) If the first as well as the last digit is even, both are
forward while consonants are coded four places to be coded by ‘*’.
backward. (i) 4 5 0 3 2 6
(ii) option (d) is correct as the letters shift 5 positions (a) XR %J (b) LR %J@
backward and forward alternately. (c) *R %J* (d) R %JL
(iii) option (c) is correct as the letters shift +1, +2, +3, (e) None of these
+4, +5 places forward respectively from left.
(ii) 3 4 1 7 5 8
(iv) option (b) is correct as for one letter we write two
(a) #LT$R% (b) %R$TL#
letters; 1st letter 3 places backward and 2nd letter
(c) %LT$R# (d) #L$TR%
2 places forward.
(e) None of these
Example  6:  Directions (Q i - iii): Read the
following information carefully and answer the (iii) 7 0 4 8 2 9
questions given below it : - (a) * L#J* (b) X L#JX
Coding of digits 1 to 9 is as follows:- (c) *@L%* (d) $ L#J$
(e) None of these
Explanation:
(i) option (c) is correct as condition (C) is applied here.
Conditions :
(ii) option (a) is correct as condition (A) is applied here.
(A) If the first digit is odd and the last digit is even, the
codes for the first and the last digit are to be (iii) option (b) is correct as condition (B) is applied here.
interchanged.

Exercise

1. In a certain code ‘HEALING’ is written as (c) HNPG (d) HPGN


‘BFIKHOJ’. How is ‘BEDTIME’ is written in (e) None of these
that code? 5. If ‘PROSET’ is coded as ‘NPMQCR’ then find
(a) EFCSJNF (b) EFCSFNJ the code for ‘KNOSEZ’ ?
(c) EFCUFNS (d) CFESFNJ (a) ILMQCX (b) IMLQCX
(e) None of these (c) IMLCQX (d) ILMQXC
2. In a certain code language ‘MODERN’ is written (e) None of these
as ‘5%38#@’ and ‘WIN’ is written as ‘6©@’. 6. If ‘OPEF’ is coded as ‘PQJK’, then find the code
How is RIDE written in that code? of ‘TPDL’ ?
(a) #©38 (b) #@38 (a) IQQU (b) QIQU
(c) ©@38 (d) #C83 (c) QUIQ (d) UQIQ
(e) None of these (e) None of these
3. If the code for ‘PENT = 16 5 14 20’ then what is 7. If code for ‘TORTURE = ERUTROT’ then code
the code for ‘DEAR’ ? for ‘NATURAL’ ?
(a) 4 15 18 (b) 4 8 11 8 (a) LARUTNA (b) LRAUTAN
(c) 4 5 1 18 (d) 4 5 18 1 (c) LARTUAN (d) LARUTAN
(e) None of these (e) None of these
4. If code for ‘DWOS = EYQT’ then find the code 8. If ‘OTLG = LGOT’ then ‘PAEMH = ?’
for ‘GNEM’ ?
(a) KZVNS (b) KVZNS
(a) HGNP (b) HNGP
SSC
(c) KVZSN (d) KSVZN (c) jo (d) ka
(e) None of these (e) None of these
9. If ‘PMQE’ is coded as ‘51’ then how will ‘NRSZ’ 17. In a certain language ‘nik ka pa’ means ‘who are
be coded? you’; ‘ka na ta da’ means ‘you may come here’;
(a) 105 (b) 85 and ‘ho ta sa’ means ‘come and go’. What does
(c) 77 (d) 98 ‘nik’ means in that code?
(e) None of these (a) who (b) are
10. If ‘Fox’ is called ‘Tiger’; ‘Tiger’ is called ‘Cow’; (c) who or are (d) Data inadequate
‘Cow’ is called ‘Lion’; ‘Lion’ is called ‘Panther’; (e) None of these
‘Panther’ is called ‘Goat’, then which of the 18. If ‘tree’ means ‘mountain’; ‘mountain’ means
following is a domestic animal? ‘water’; ‘water’ means ‘jungle’; ‘jungle’ means
(a) Lion (b) Cow ‘bus’; ‘bus’ means ‘truck’; and ‘truck’ means
(c) Fox (d) Goat ‘house’ then where does the fish live?
(e) None of these (a) water (b) jungle
11. If ‘MACHINE’ is coded as ‘19 7 9 14 15 20 11’ (c) mountains (d) bus
then how will ‘TOTAL’ be coded? (e) truck
(a) 26 12 26 7 19 (b) 26 21 26 7 91 19. In a certain code language the word ‘FUTILE’ is
(c) 26 21 26 7 18 (d) 26 21 62 7 19 written as ‘HYVMNI’. How will the word
‘PENCIL’ be written in that language?
(e) None of these
(a) OIFRLT (b) OIFRLS
12. If code for ‘NOTE  =54’, then find the code for
‘TALENT’. (c) OLFRIT (d) OIRFLT
(a) 82 (b) 72 (e) None of these
(c) 85 (d) 68 20. In a certain code, ‘POETRY’ is written as
‘QONDSQX’ and ‘OVER’ is written as ‘PNUDQ’.
(e) None of these
How is ‘MORE’ is written in that code language?
13. If code for ‘ETLM = 5’ , then find the code for
(a) LNNQD (b) NNNQD
‘YZNP’.
(c) NLNQD (d) NLPQD
(a) 6 (b) 4
(e) None of these
(c) 8 (d) 9
Directions  (Q  21  -  25):  Read  the  following
(e) None of these
informations to find the code for the number given
14. If code for ‘PICTURE = 11 18 24 7 6  9 22’ , then below it:
find the code for ‘BEST’.
The coding of numbers from 0 to 9 is as follows:-
(a) 25 22 8 7 (b) 25 22 7 8
(c) 22 5 27 8 (d) 25 7 22 8
(e) None of these
15. Find the code for ‘ACTORS’; if code for Conditions
‘NATURE = UHWXQD’. (i) If the first as well as the last digit is odd, then codes
(a) UVWRDF (b) UVRWDF are to be interchanged.
(c) UVWRFD (d) UWVRDF (ii) If the first digit is even and the last digit is odd,
(e) None of these both are to be coded by the code for odd digit.
16. In a certain code ‘open the door’ is written as ‘ka (iii) If the last digit is ‘0’ it is to be coded by ‘X’.
te jo’; ‘door is closed’ is written as ‘jo pa ma’ (iv) If the first as well as the last digit is even both are
and ‘this is good’ is written as ‘la ra pa’. Find the to be coded by ‘*’
code for ‘closed’. 21. 7 6 4 1 3 8
(a) ma (b) pa (a) EJQA E (b) RJOA E
SSC
(c) RJQA R (d) EJQA * given in each question.
(e) None of these The code of some letters are given below in the form
of digit and symbols:
22. 3 8 9 1 6 0
(a) R$AJH (b) R$AJX
(c) XR$AJX (d) R$AJ
Conditions
(e) None of these
(i) If the first letter is a consonant and the last letter is
23. 8 0 1 2 3 4 a vowel both are to be coded as the code for the
(a) *HA@ * (b) RHA@ * consonant.
(c) *H @A* (d) RHA@ Q (ii) If the first as well as the last letter is a vowel, there
(e) None of these codes are to be interchanged.
(iii) If the first letter is a vowel and the last letter is a
24. 4 3 9 1 5 7
consonant both are to be coded as .
(a) Q $AKE2* (b) E $AKQ 30. JUKSTD
(c) Q $AKO@A*(d) E $AKE (a) 6 7 2 4 3 9 (b) 9 7 2 4 3 6
(e) None of these (c) 9 7 2 4 3 9 (d) 6 7 2 4 3 6
25. 5 8 6 4 0 3 (e) None of these
(a) KRJQH (b) RJQHK 31. EIHLFB
(c) HJQRK (d) KHJQR (a) $ @#% (b) *@#%
(e) None of these (c) $*@#%* (d) $*@#%1
26. If ‘8’ is written as ‘B’; ‘1’ as ‘R’; ‘6’ as ‘K’; ‘9’ as (e) None of these
‘0’; ‘4’ as ‘M’; ‘7’ as ‘W’ and ‘3’ as ‘T’, then how 32. In a certain code  ‘SOLDIER’  is written as
would ‘WROMBT’ be written in the numeric ‘JFSCRNK’. How is ‘GENIOUS’ written in that
form? code?
(a) 7 1 4 9 8 3 (b) 7 1 9 4 8 3 (a) PVTHHFO (b) PVTHFDM
(c) 7 6 9 4 8 3 (d) 7 1 9 4 8 6 (c) PVTHMDF (d) TVPHFDM
(e) None of these (e) None of these
27. In a certain code ‘MIGHT’ is written as ‘LHFGS’. 33. If ‘blue’ is called ‘green’; ‘green is called ‘black’;
How is ‘BELOW’ written in that code? ‘black’ is called ‘white’; ‘white’ is called ‘pink’;
(a) CFMPX (b) ADJNV ‘pink’ is called ‘red’ and ‘red’ is called ‘orange’,
(c) ADKMV (d) ADKNV then what is the colour of ‘blood’.
(e) None of these (a) Orange (b) Red
28. In a certain code ‘good’ and ‘bad’ is written as (c) White (d) Pink
‘725’; ‘One’ and ‘all’ is written as ‘932’ and ‘this (e) None of these
is good’ is written as ‘154’. How is ‘one’ written 34. If the code for ‘HZ = 18’, then code for ‘DX = ?
in that code? (a) 55 (b) 28
(a) 9 (b) 3 (c) 14 (d) 20
(c) 2 (d) Data inadequate (e) None of these
(e) None of these 35. In a certain code language ‘food is good’ is written
29. If code for ‘APQR = 52’, then how will ‘RZXB’ as ‘ho na ta’, ‘eat food regularly’ is written as
be coded? ‘sa ta la’ and ‘keep good health’ is written as ‘da
(a) 45 (b) 70 na ja’. How is ‘eat written in that code?
(c) 95 (d) 120 (a) sa (b) la
(e) None of these (c) sa or la (d) Data inadequate
Direction  (Q.  30  -  31)  Read  the  following (e) None of these
information to find the code for the group of letters 36. If code for ‘RABBIT = IZYYRG’, then what is
SSC
the code for ‘TRUE’? (A) SATETS (B) STAETS
(a) GIFV (b) GFJV (C) SATTES (D) SAETTS
(c) GIVF (d) GVFI 46. In a certain code ‘MILLION’ is written as ‘IMLLOIN’
(e) None of these then code for HILTON would be ?
37. In a certain code ‘DOES’ is written as ‘5$3%’ and
(A) IHTLON (B) IHLOTN
‘SITE’  is written as ‘%4#3’.  How is ‘EDIT’
written in that code. (C) IHTLNO (D) HILTNO
(a) 354# (b) 3#54 47. If MONUMENT is coded as ONPTODPS, then how
(c) 3$4# (d) 35$# is BEAR coded as :
(e) None of these (A) ADBQ (B) DGCT
38. If code for  ‘TAMILNADU  =  UDANLIMAT’, (C) CDCD (D) DDCQ
then code for ‘KARNATAKA’?
48. In a certain code CHAPTER is written as DGBOUDS,
(a) AKATANRKA (b) AKATANRAK
then BROWN is coded as :
(c) AKAATNRKA (d) AKANATRAK
(A) CQPVO (B) EONXM
(e) None of these
(C) EQPVO (D) ESQVO
39. Find the code for ‘LDP’, if the code for ‘IBH’ is
‘PIO’. 49. Code for BEAT is GIDV, how is SOUP coded as:
(a) SKW (b) SWK (A) XSYS (B) YSXR
(c) WKS (d) WSK (C) YXSR (D) XSXR
(e) None of these 50. If the word CLERK is codes as EOIWQ then how is
40. If the code for ‘LMAP = 12 13 1 16’, then code TABLE coded as :
for ‘TSB = ?’
(A) VCDNG (B) VCDGN
(a) 21 9 20 (b) 22 1 90
(C) VDFQK (D) VDFOK
(c) 20 19 2 (d) 2 10 9 2
51. If UNITED is coded as SLGRCB, how is DISOWN
(e) None of these
41. If CONSTITUTION is written as CONSTIUTTINO coded as:
then DISTRIBUTION would be (A) BGQMUL (B) CGRLTK
(A) DISTRIBUTION (B) DISTRIUBTINO (C) CGRTLK (D) BGQLUM
(C) DISTIRUBTINO (D) DTISRIUBITON 52. If EASY is coded as BBDDPPBB, then HINT will be

42. If TELEPHONE is written as ENOHPELET then coded as :

ALIGATOR will be written as : (A) KKWWLLEE (B) EELLKKWW


(A) ROTAGILE (B) ROTAGILA (C) AAWWLLKK (D) BBLLDDKK
(C) ROTEGILA (D) ROTAGAIL 53. If FOX is coded as GHPQYZ, how would BOAT be
43. If MUNICIPALITY is written as INMUAPCIYTLI written as :

then the code for JUDICIAL will be : (A) SVUVSRHJ (B) CTUAREKL
(A) UJDILACI (B) IDUJACI (C) XMKLBAI (D) CDPQBCUV
(C) IDJULAIC (D) IDJULACI 54. Code for LONDON is HPOEPO, then word for

44. If GIGANTIC is written as GIGTANCI, then how is DVOHSZ is :


MIRACLES coded as : (A) MEXICO (B) ISLANDS
(A) MIRLCAES (B) MIRLACSE (C) HOLAND (D) HUNGRY
(C) RIMCALSE (D) RIMLCAES 55. If VCDNG stands for TABLE, then MPKHG stands
45. In a certain code ‘DIVISION’ is written as DVISIOIN, for :
how is STATES written as : (A) SPORT (B) KNIFE
SSC
(C) PLACE (D) CHAIR 66. If SINCERE is coded as HOHZQFV, then MANAGER
56. If CENPNPJL stands for DOOK, then FHNPNPCE is coded as :
stands for : (a) PXQXJBU (b) OPUTHGY
(A) FOOD (B) ROOT (c) UBJXQXP (d) OPIRS
(C) GOOD (D) FOUR 67. If CUPBOARD is written as CXPBXXRD then code
57. If CENPBDSUNPQS stands for DOCTOR, then for AMAZEMENT is ?
MOTVQSRTDF stands for : (a) XMXZXMXNT (b) XMXZXXMNT
(A) PURSE (B) NURSE (c) XMXZXMXTT (d) XMXZXMMXT
(C) HORSE (D) FORCE 68. If MANAGEMENT is written as MSNSGSMSNT then
58. If JOEJB means INDIA, then last letter of the word code for BEHAVIOUR is:
got by decoding BSNZ is : (A) BSHSVSSOUR (B) BSHSVSSSR
(A) W (B) X (C) BEHSVSSUR (D) BNHAVISSR
(C) Y (D) Z 69. If ACHIEVEMENT is written as CCHCCVCMCNT,
59. If QMBO is the code for PLAN then NJOJTUFS is what is the code for EFFECTIVE ?
the code for ? (A) CEFCCTHVC (B) CVTHCCFFC
(A) SINISTER (B) SEMESTER (C) CFFCCTCVC (D) ECCECTCVC
(C) MINISTER (D) DISEASTER 70. If ACTOR is written as ECTUR, then what is the code
60. If BVZHG means YEAST, what does DVZI means? for OPERA ?
(A) BEAR (B) WEAR (a) UPIRE (b) UPIER
(C) DARK (D) BARK (c) EPORU (d) OPIRS
61. If QBSJT stands for PARIS, then MPOEPO stands for 71. If BRASS is coded as YIZHH then what is the code
? for COPPER ?
(A) QUEBEC (B) LARGER (a) XLWKZVI (b) IVKKLX
(C) LONDON (D) MISTER (c) XLKKVI (d) None of these
62. If MONKEY is coded as XDJMNL then TIGER is 72. If VILLAGE is coded as EROOZTV, then FOREIGN
coded as : is coded as ?
(a) SHFDQ (b) QDFHS (A) ULIVRTM (b) Cannot be determined
(c) SDFGS (d) QDHJS (C) UIIVRTM (D) ULIVTO
63. If PARIS is coded as VLUDS, then what is the code 73. If code for MATCH is NZGXS, then code for SELECT
for TOKYO : ?
(A) WRNBW (B) RBNSW (A) HWOVXG (B) HVOVXG
(C) RBNRW (D) NMRWD (C) HUOVXGY (D) None of these
64. If NAMITA is coded s XVFOXP then what is the code 74. If PRACTICE is coded as KIZXGRXV, then code for
for MADHULIKA : ENERGY is ?
(A) JCAKSOFXM (B) CHKIWEFXO (A) VMVITB (B) VMVIYC
(C) XMFOSPPKA (D) JCAKOSCJ (C) VMBIYD (A) None of these
65. The code for ADVENTURES is WIVYXJARZW, what 75. IF (M=26) AND (DOG=52) THEN (BAT = ?)
is the code for SYSTEMATIC : (A) 46 (B) 41
(a) GMXEQAPOUO (b) GMXEXJARZE (C) 39 (D) 44
(c) GMEPAGHYAK (d) THKORPEHGHX
SSC
76. IF (REASON=5) AND (BELIEVED=7) THEN ?)
(GOVERNMENT =) ? (A) 9 (B) 10
(A) 6 (B) 9 (C) 23 (D) 43
(C) 10 (D) 8 88. IF (ARID=80), ( RAID=63) , (DEAR=89)
77. IF (BAT=40) AND (AT=20) THEN (CAT = ?) THEN READ = ?
(A) 70 (B) 50 (A) 47 (B) 53
(C) 60 (D) 30 (C) 74 (D) 59
78. IF (HE=41) AND (SHE=49) THEN (THE = ?) 89. IF (CARUSO=5), ( GIGLI=4) THEN (CROSBY= ?)
(A) 33 (B) 36
(a) 4 (b) 5
(C) 48 (D) 40
(C) 6 (D) 7
79. IF (Z=52) AND (ACT=48) THEN (BAT = ?)
90. IF (C=3) AND (CEP=24) THEN (HUX = ?)
(A) 39 (B) 41
(A) 47 (B) 49
(C) 44 (4 46
(C) 51 (D) 53
80. IF (GO=32) AND (SHE=49) THEN (SOME=?)
91. IF (HEMA=81) AND (VELU= ?)
(A)56 (B) 58
(C) 62 (D) 70 (A) 48 (B) 54
81. IF (DEMOCRATIC=9.1) THEN (AGITATION= ?) (C) 56 (D) 60
(A) 11.50 (B) 10.66 92. WHAT IS THE VALUE OF (H+B+F)
(C) 10.60 (D) 11.8 (A) 20 (B) 16
82. IF (PAPER=11.20), AND (PENCIL=9.83) THEN (C) 17 (D) 15
(PEN = ?) 93. J-B+D = ?
(A) 12.80 (B) 11.60 (A) 9 (B) 21
(C) 11.66 (D) 13.8 (C) 16 (D) 12
83. IF (AROMA=24) AND (GRAND=22) THEN 94. IF (TOM=48) AND (DICK=27) THEN ( HARRY = ?)
(KWALITY = ?) (a) 46 (b) 50
(A) 40 (B) 62
(C) 70 (D) 67
(C) 55.5 (D) 50.5
95. If the code for RADHA = 181481, then the code for
84. IF (SUPER=79) AND (SUPREME=97) THEN SHYAM is ?
(LABOUR = ?) (A) 18825112 (B) 19824133
(A) 79 (B) 69 (C) 19825114 (D) NONE OF THESE
(C) 89 (D) 59 96. If the code for SULLEN = 8615152213, then code for
85. IF (ARM=6)AND (MAN=0) THEN ( SIN = ?) HAPPY is ?
(A) 14 (B) 16 (A) 1962611115 (B) 1952671356
(C) 19 (D) 20 (C) 192412103 (D) 192611112
86. IF (CAR=54) AND (DIM=468) THEN (EAT =?) 97. IF (RATE=6), (FATE=18), (MATE=11) THEN (CEAT
(A) 109 (B) 100 = ?)
(C) 26 (D) 167 (A) 12 (B) 13
87. IF (HOTEL=12) AND ( SIN=14) THEN (ARMID = (C) 14 (D) 15
SSC
98. IF (PRESS=77), then (MEDIA=?) (a) HBNZV (b) GBNZV
(A) 32 (B) 23 (c) GBMYV (d) GBMYV
(C) 56 (D) 72 108. If in any code language ‘PLANET’ is written as
‘QMBOFU’ then how to write ‘ORANGE’ in the
99. IF (ENTER=62), then (PREFER=?)
same language?
(A) 94 (B) 89
(a) PTBOHF (b) PSBMHF
(C) 68 (D) 57 (c) PSBOIF (d) PSBOHF
100. IF (MOON=12), then (NOON=?) 109. If ‘SPARK’ in any code language is written as
(A) 12 (B) 15 ‘TQBSL’ then how to write ‘NURSE’ in the same
(C) 13 (D) 14 language?
101. If in any code language ‘FIELD’ is written as ‘GJFME’, (a) OWSTF (b) OBSTF
then how will ‘NORMAL’ bo written? (c) OVSUF (d) OVSTF
(A) LAMRON (B) MANQLK 110. If in a code language ‘GOAT’ is written as ‘CKWP’
(C) OPSNBM (D) PQTOCN then how to write ‘ROPE’ in the same language?
102. If ‘CLERK’ in any code language is written as (a) NKAL (b) NKZL
‘EOIWQ’, then how will ‘TABLE’ be coded in the that (c) NKLA (d) NJLA
language? 111. If ‘PENCIL’ in any code language is written as
(A) VCDNG (B) VCDGN ‘TIRGMP’ then how to write ‘BOARD’ in the same
(D) VDFQK (D) VDFOK language?
103. If in any code language ‘BUTTER’ is written as (a) FSEUH (b) FSEVH
‘CVUUFS’ and ‘BREAD’ as ‘CSFBE’, then how to (c) FTEVH (d) FSEVG
write ‘COFFEE’ in the same language? 112. If in any code language ‘PINK’ is written as ‘NGLI’
(a) DPGGFF (b) GGDPEE then how to write 'IRON' in the same language?
(c) GDPGFF (d) FFDPGG (a) GMPE (b) KMPL
104. If in any code language ‘STAMP’ is written as (c) GPML (d) KPML
‘TVBOQ’ then how to write ‘BRAIN’ in same 113. If in any code language ‘NUMBER’ is written as
language? ‘OTNAFQ’ then how will ‘CHANGE’ be written in
(a) DSCJO (b) CTBKO the same language?
(c) CSBJO (d) CTCKO (a) DGBMHF (b) BGBMHF
105. If ‘BLOOD’ in any code language is written as (c) DIBMHD (d) DGBMHD
‘EIRLG’, then how to write ‘PERIOD’ in the same 114. If ‘ONIDA’ in any code language is written as
language? ‘USMGC’ then how to write ‘FRAUD’ in the same
(a) SBURAF (b) SBUFRA language?
(c) SUFBAR (d) RBUFSA (a) LWEYF (b) LWEXF
106. If in any code language ‘ROAD’ is written as (c) LWEZF (d) LXEXF
‘URDG’ then how will ‘SWAN’ be written in the 115. If in any code language ‘COBRA’ is written as
same language? ‘GTFWE’ then how to write ‘FRAUD’ in the same
(a) VXDQ (b) VZCQ language?
(c) UXDQ (d) VZDQ (a) JVEZH (b) ZWEJH
107. If 'EARTH' is written as 'GZTSJ', then how will (c) JWDZH (d) JWEZH
‘ECLAT’ be written in same language? 116. If in any code language ‘BLAME’ is written as
SSC
‘AKZLD’ then how to write ‘SPACE’ in the same (c) RJZZ (d) RKZW
language? 125. If in any code language ‘QUESTION’ is written as
(a) ROZDD (b) RNZBD ‘NXBVQLLQ’ then how will ‘REPLY’ be written in
(c) ROZBD (d) ROZBE the same language?
117. If ‘DIPLOMA’ in any code language is written as (a) OBMIV (b) UHSOB
‘FERHQIC’ then how will ‘FOREIGN’ be written in (c) OHMOV (d) OFMMV
that code? 126. If in any code language ‘RAMA’ is written as
(a) HJTAKCP (b) HKTALCP ‘OWHX’ then how to write ‘ALKA’ in the same
(c) HKTAKCP (d) HKTAKBP language?
118. If in any code language ‘BINA’ is written as ‘ZFLX’ (a) XHFY (b) XHGX
then how will ‘JONA’ be written in the same (c) XHFX (d) XFHX
language? 127. If in any code language ‘DISPLAY’ is written as
(a) HLMX (b) HLLY ‘BLQSJDW’ then how to write ‘PROJECT’ in the
(c) HXLL (d) HLLX same language?
119. If in any code language ‘BIRD’ is written as ‘YGOB’ (a) NUMMCFR (b) NUNMCFR
then how will ‘AVIS’ be written in the same (c) NTNMCFR (d) NTMMCFR
language? 128. If any code language ‘SURBHI’ is written as
(a) XTGQ (b) XSFQ ‘QSPZFG’ then how to write ‘ANJANA’ in the same
(c) XTFQ (d) XUFQ language?
120. If in any code language ‘BLADE’ is written as (a) YLHZLY (b) YLHYLX
‘AJZBD’ then how to write ‘GLOBE’ in the same (c) YLHYLY (d) YLHYMY
language? 129. If ‘GOLD’ in any language is written as ‘DLIA’ then
(a) FJNYD (b) FJNZC how will word ‘HOPE’ be written in same
(c) FJNZE (d) FJNZD language?
121. If in any code language ‘CHAIR’ is written as (a) FMNC (b) DKLA
‘EGCHT’ then how will ‘AUDIT’ be written in the (c) DGKM (d) ELMB
same language? 130. If any code language ‘MOON’ is written as ‘KMML’
(a) CTFHV (b) CSFHV then how to write ‘EARTH’ in same language?
(c) BTFHV (d) CTEHV (a) CYPOF (b) CYPRF
122. If in any code language ‘OPTION’ is written as (c) DZQSG (d) BZQRF
‘UKXFQM’ then how will ‘CHOICE’ be written in 131. If in any code language ‘DRINK’ is written as
the same language? ‘AOFKH’, then how will ‘MILK’ be written in the
(a) ICSFED (b) ICKFED same language?
(c) IMRLEF (d) HLRKDF (a) HKMO (b) JFML
123. If ‘RURAL’ in any language is written as ‘SRWTU’ (c) JFIH (d) IEHJ
then how to write ‘URBAN’ in the same language? 132. If ‘TABLE’ in any code language is written as
(a VOGSW (b) VOHTW ‘UZCKF’ then how will ‘CHAIR’ be written in the
(c) VOGTW (d) VOGTX same language?
124. If any code language ‘DARK’ is written as ‘HVXD’ (a) EGBHQ (b) DIBJS
then how to write ‘NOTE’ in the same language? (c) DGBHS (d) BGZHQ
(a) RKZX (b) RJZX 133. If in any code language ‘TAP’ is written as ‘SZO’
SSC
then how will ‘FREEZE’ be written in the same ‘FULCBPO’ then how will ‘MONSOON’ be written
language? in the same language?
(a) ESDFYEF (b) GQFDYF (a) OQRSKML (b) NQOSKML
(c) EQDFYG (d) EQDDYD (c) OPQSKML (d) OQRSNQQ
134. If in any code language ‘NOBLE’ is written as 142. If word ‘LISTEN’ in any code language is written
‘QREOH’ then how will ‘PLATE’ be written in the as ‘KHRUFO’ then how will ‘KEEPER’ be written in
same language? the same language?
(a) SMDWH (b) SODWH (a) JDDQFS (b) JDCQFS
(c) SODVH (d) RODWG (c) JBBQFS (d) JDDPQS
135. If ‘LPPHGLDWH’ means ‘IMMED-IATE’ then what 143. If word ‘COUNCIL’ in any code language is written
does ‘WRSVHFOHW’ mean? as ‘DPVNDJM’ then how will ‘PROMISE’ be written
(a) TOPSECLET (b) CRICKETER in the same language?
(c) STOPTRACK (d) TEARAPART (a) QSPBJTE (b) PSQBJTE
136. If in any code language ‘COMPUTER’ is written (c) QSPMJTF (d) QSPMJUF
as ‘RFUVQNPC’ then how will ‘MEDICINE’ be 144. If in any code language ‘COMPANY’ is written as
written in the same language? ‘BNLQBOZ’ then how will ‘HAUNTED’ be written
(a) EOJDJEFM (b) EOJDEJFM in the same language?
(c) MEFJDJOE (d) MFEDJJOE (a) ZGTOUFE (b) GZTOUFE
137. If ‘DISTANCE’ in any code language is written as (c) GZUOTFF (d) ZGTUOFE
‘FLUWCQEH’ then how will ‘NUMERALS’ be 145. If in any code language ‘COURSE’ is written as
written in the same language? ‘DPVQRD’ then how will ‘SCHOOL’ be written in
(a) PXNHTDNV the same language?
(b) PXOITDNV (a) TDINNK (b) TEKNNK
(c) PWOHTDNV (c) TDJMNK (d) TDIMNK
(d) PXOHTDNV 146. If in any code language ‘ORNAMENT’ is written
138. If any certain code language word ‘HANDSOME’ as ‘UOFNNQMZ’ then how will ‘MEDICINE’ be
is written as ‘HLRRGMDG’ then how will written in the same language?
‘OVERLAND’ be written in the same language? (a) FOJDMECH (b) FQJDLDCH
(a) GMDKUDYN (b) GMCJUDXN (c) FQJDLECH (d) FOJDLDCH
(c) FMJKUDYN (d) POCKUDYN 147. If in any code language ‘GRACE’ is written as
139. If in any code language ‘CREMETORIUM’ is written ‘FQBDF’ then how will ‘CLAIM’ be written in the
as ‘NVJSPTDSFNF’ then how will ‘PRACTIONERS’ same language?
be written in the same language? (a) BKZJN (b) BKBJN
(a) UDBSQITSFOP (b) TSFOPIQSBDU (c) BMBJN (d) BOCJN
(c) TSFPOIQSBDU (d) TSFOPIUDBSQ 148. If in any code language ‘MERCURY’ is written as
140. If in any code language ‘CYLINDER’ is written as ‘NFSBTQX’ then how will ‘STEEL’ be written in the
‘UHGQGJWA’ then how will ‘HYDROGEN’ be same language?
written in the same language? (a) TSFDK (b) TUFDM
(a) QHJROBWF (b) QHJRPBWF (c) TUDDK (d) TUDEJ
(c) QJHRPBWF (d) QHRJOBWF 149. If in any code language ‘SURGEON’ is written as
141. If in any code language ‘PRECISE’ is written as ‘TVSHDNM’ then how will ‘DENTIST’ be written
SSC
in the same language? (a) BSHSFHKM (b) BHSSMHHF
(a) EFUUJST (b) EFPTHRS (c) BSSHFMKH (d) BHSSFKHM
(c) EFOUJRS (d) EFOUHRS 158. If in any code language ‘PARENTS’ is written as
150. If in any code language ‘PERSONAL’ is written as ‘RCTGPVU’ then how will code ‘EJKNFTGP’ be
‘FQTSOPMB’ then how will ‘REPUBLIC’ be written written in the same language?
in the same language? (a) CHILDERN (b) CHLIDREN
(a) FSVQAMDJ (b) FSVQMCDJ (c) CHILDREN (d) CIHLDREN
(c) FSVQNCDJ (d) FSVQMDCJ 159. If word ‘JAPAN’ in any code language is written as
151. If in any code language word ‘TIME’ and ‘KCSES’ then how will code ‘JPGMF’ be written?
‘MOTHER’ are coded as ‘JUFN’ and ‘UPNSFI’ (a) INIDA (b) INDAI
respectively then how will ‘BOTH’ be written in (c) INDIA (d) NIDIA
the same language? 160. If in any code language ‘BAND’ is written as ‘CBOE’
(a) ANUI (b) PCIU then how will code ‘OPCF’ be written?
(c) PCGS (d) NAIU (a) NOEB (b) NOBE
152. If in any code language ‘TEACHER’ is written as (c) NBOE (d) NOBD
‘BFUDSFI’ then how will word ‘STUDENT’ be 161. If ‘VICTORY’ is code as ‘YLFWRUB’ then will
written in the same language? ‘FAILURE’ be coded?
(a) VUTEUOF (b) VUTDUOF (a) JELOXUH (b) IDLOXUH
(c) VUTDUFO (d) VUTEUPF (c) JDLKWUH (d) IDOLKUH
153. If in any code language ‘JUNCTION’ is written as 162. If in any code language ‘PAPER’ is written as
‘MTISBMNH’ then how will word ‘DECEMBER’ be ‘WISCQ’ then which word will be written for
written in the same language? ‘DTRE’?
(a) BDCTDQDA (b) BDCLETDA (a) COYP (b) COPY
(c) BDCLDQPA (d) BDCLDQDA (c) CORY (d) COPZ
154. If in any code language ‘ALARM’ is written as 163. If ‘KCIQ’ is the code for ‘PICK’ then code ‘HSINUQ’
‘MZANQ’ then how will word ‘CLASH’ be written stands for-
in the same language?
(a) PUNISH (b) HSINUP
(a) MABQI (b) MBAIR
(c) PUNISHI (d) RUNSHI
(c) MBACI (d) MABDQ
164. If in any code language ‘RANGER’ is written as
155. If in any code language ‘FLIGHT’ is written as ‘REGNAR’ then how will ‘TABLE’ be written in the
‘GMJSGF’ then how will ‘TICKET’ be written? same language?
(a) UJTDSD (b) UJDSOJ (a) TABEL (b) ELBAT
(c) UJDSDJ (d) UTDSDJ (c) TELBAT (d) TLBAT
156. If ‘AUTOMATIC’ in any code language is written 165. If in any code language ‘CLOCK’ is written as
as ‘PUVBMBUJD’ then how will word ‘BUILDINGS’ ‘KCOLC’ then how will ‘STEPS’ be written in the
be written? same language?
(a) MJVCDJOHT (b) CVJMDJOHT (a) SPEST (b) SPSET
(c) MKVCDTHOJ (d) CVJMDTHOJ (c) SEPST (d) SPETS
157. If in any code language ‘CREATIVE’ is written as 166. If in any code language ‘ADJUST’ is written as
‘BDSBFUJS’ then how will ‘TRIANGLE’ be written ‘JDATSU’ then how will ‘VERIFY’ be written in the
in the same language? same language?
SSC
(a) REVFYI (b) REVYFI how to write ‘COPY’ in that code?
(c) ERVYFI (d) VRFYIF (a) XOKB (b) XLKZ
167. If in any code language ‘PUNISHED’ is written as (c) XLKB (d) XLKC
‘UPNISHDE’ then how will ‘CREATION’ be written 176. If in any code language ‘BANK’ is written as ‘YZMP’
in the same language? then how to write ‘STATE’ in the same language?
(a) CRAETION (b) REAETINO (a) HGZGV (b) HZGZV
(c) RCEATINO (d) CRAETINO (c) HGAGV (d) HGZRV
168. ‘DISPEL’ in a code language is written as ‘IDPSLE’ 177. If ‘IZQZHGSZM’ is code for ‘RAJASHTAN’, how will
then how will ‘EFFECT’ be written in that code? word ‘COMMISSION’ be written?
(a) FEEFTC (b) CTFEEF (a) XYZNNILTNM (b) XLNNRHRLRM
(c) EFFETC (d) EEFFCT (c) XSNNJTORRM (d) XLNNRHHRLM
169. If in a certain code ‘AUTHORITY’ is written as 178. If in any code language ‘BOY’ is written as
‘YTUROHTIA’ then how will ‘DESIGNATE’ be ‘CDPQZA’ then to write ‘CAT’ in the same
written in that code? language?
(a) ESENGATD (b) ESEGNITAD (a) DEYZUV (b) DEBCUV
(c) ESENGITAD (d) ESNEGTAD (c) DEBCSV (d) DEBCVU
170. If in a certain code ‘STANDING’ is written as 179. If in any code language ‘GAMBLE’ is written as
‘NATSGNID’ then how will ‘PRODUCES’ be written ‘HOQ’ how will ‘FRIEND’ be written in that code?
in that code? (a) XNP (b) NXR
(a) DOPRSECU (b) DORPSCEU (c) XNR (d) NNQ
(c) DORPSECU (d) DORPESCU 180. If in any code language ‘RUN’ is written as
171. If in any code language ‘ACTIVE’ is written as ‘QSTVMO’ how to write ‘VAN’ in same code?
‘AVITCE’ how will ‘DELIMIT’ be written in the same (a) UWZBMN (b) UWZBNM
language?
(c) UWBZMO (d) UWZBMO
(a) TIMILED (b) DTIMILE
181. If word ‘MENTAL’ is written as ‘LNDFMOSUZBKM’,
(c) DIMIELT (d) DIMILET how to write ‘TEST’ in that code?
172. If in any code language ‘GAME’ is written as ‘TZNV’ (a) UVFGTUUV (b) RSCDQRRS
how to write ‘BOSS’ in the same language?
(c) SUDFQRSU (d) SUDFRTSU
(a) YLII (b) LYHH
182. If in a code language ‘SIR’ is written as ‘PSPIPR’,
(c) YLSH (d) YLHH how will 'MAN' be written in same language?
173. If in a code language ‘TRAIN’ is written as ‘GIZRM’ (a) PMANP (b) PMPAPN
how will ‘FIGURE’ be written in the same
(c) NANP (d) MPANP
language?
183. If in a code language ‘PEN’ is written as ‘NOUALM’,
(a) USTGKV (b) VTYXTC
how to write ‘COD’ in that code?
(c) URTFIV (d) VKGTSV
(a) ABIEBC (b) ABEICB
174. If in any code language ‘MANISH’ is written as
(c) ABEIBC (d) ABIECB
‘NZMRHS’ how will ‘MOHANI’ be written in that
184. If a certain code language ‘PLATE’ is written as
code?
'PQMBUF' and 'TOWN' as 'TUPXO', how will word
(a) NLSZMT (b) NLZSMR
‘DEAR' be written?
(c) NLSMZR (d) NLSZMR
(a) DEFBS (b) DDFBS
175. If in any code language ‘BOOK’ is written as ‘YLLP’,
(c) EEFBS (d) DEEBS
SSC
185. If in any code language ‘SUN’ is written as ‘JCE’ (c) 64 (d) 72
then how to write 'LET' in same code? 196. If ARID = 32 and DEAR = 28 then what is value of
(a) CEB (b) CBD ABNP?
(c) CEK (d) CBE (a) 18 (b) 33
186. If in a code language ‘SLOP’ is written as ‘JCFC’, (c) 42 (d) 51
how to write ‘WALK’ in that language? 197. If FOX is coded as 45, what would be the code for
(a) EACB (b) EADC ‘BOX’?
(c) FACD (d) FCBD (a) 41 (b) 49
Direction: Word ‘SCENES’ has been written using (c) 55 (d) 60
the following four code:- 198. If ‘SINGLE’ is coded 66, what would be the code
(a) U E G P G U for 'WINKLE'?
(b) T E H R J Y (a) 76 (b) 66
(c) H X V M V H (c) 78 (d) 74
(d) R A B J Z M 199. If in a code language BAD is valued as 8 and CAT
Question Nos. 87 to 90 are based on the above as 60, what would be the value of DOG in that
mentioned rule (a, b, c, d). You have to tell which rule code language?
has been used in the following questions:- (a) 105 (b) 28
Word Code (c) 420 (d) 26
187. TITAN SGQWI 200. If ST – OP = 8 then SP – OT = ?
(a) (b) (c) (d) (a) 0 (b) 2
188. STATE HGZGV (c) 3 (d) 11
(a) (b) (c) (d) 201. If E = 5 and HOTEL = 12, what is code for LAMB?
189. ROBUST SQFYXZ (a) 28 (b) 7
(a) (b) (c) (d) (c) 10 (d) 26
190. CHECK EJGEM 202. If POLICE = 10 and MEND = 9, what is code for
(a) (b) (c) (d) ‘INSPECTORS’?
191. If C = 3 and POLISH = 79 then POINTER = ? (a) 13.8 (b) 19
(a) 95 (b) 96 (c) 16 (d) 21
(c) 97 (d) 98 203. If ‘MOON’ is coded is 12, what is code for ‘NOON’?
192. If A = 1 and VAN = 37 then FAT = ? (a) 12 (b) 15
(a) 21 (b) 20 (c) 13 (d) 14
(c) 26 (d) 27 204. If LOOK = 2 then POOK = ?
193. If A = 26 and SUN = 27 then CAT = ? (a) 2 (b) –2
(a) 24 (b) 57 (c) 0 (d) 6
(c) 58 (d) 27 205. If HAZI = 81269 then STRATA = ?
194. If C = 3 and FEAR = 30 then HAIR = ? (a) 1920182201 (b) 1820181201
(a) 35 (b) 36 (c) 1920181201 (d) 19432176
(c) 30 (d) 33 206. If PACKET = 11262416227 then NOISES = ?
195. If If A = 2, M = 26 and Z = 52 then BET = ? (a) 131218229 (b) 1312188228
(a) 44 (b) 54 (c) 1415919519 (d) 13132919519
SSC
Answers
1. b 2. a 3. c 4. d 5. a 6. d
7. d 8. a 9. c 10. a 11. c 12. b
13. d 14. a 15. a 16. a 17. c 18. b
19. e 20. c 21. e 22. b 23. a 24. d
25. b 26. b 27. d 28. d 29. b 30. b
31. b 32. b 33. a 34. d 35. c 36. a
37. a 38. b 39. a 40. c 41. b 42. b
43. d 44. b 45. a 46. c 47. d 48. a
49. d 50. c 51. a 52. b 53. d 54. d
55. b 56. c 57. b 58. c 59. c 60. b
61. 3 62. b 63. c 64. b 65. a 66. c
67. a 68. 2 69. c 70. a 71. c 72. a
73. b 74. a 75. 1 76. b 77. c 78. c
79. d 80. a 81. b 82. 3 83. d 84. b
85. a 86. b 87. a 88. a 89. 2 90. d
91. a 92. b 93. d 94. c 95. d 96. 4
97. b 98. a 99. c 100. c 101. c 102. c
103. a 104. b 105. b 106. d 107. b 108. d
109. d 110. c 111. b 112. c 113. d 114. b
115. d 116. c 117. c 118. d 119. c 120. d
121. a 122. a 123. c 124. b 125. c 126. c
127. a 128. c 129. d 130. b 131. c 132. c
133. d 134. b 135. a 136. a 137. d 138. a
139. b 140. b 141. a 142. a 143. c 144. b
145. a 146. d 147. b 148. c 149. d 150. b
151. b 152. a 153. d 154. b 155. c 156. a
157. d 158. c 159. c 160. b 161. b 162. b
163. a 164. b 165. d 166. b 167. c 168. a
169. c 170. c 171. d 172. d 173. c 174. d
175. c 176. a 177. d 178. b 179. c 180. d
181. d 182. b 183. c 184. a 185. a 186. a
187. d 188. c 189. b 190. a 191. a 192. d
193. b 194. b 195. b 196. b 197. a 198. d
199. c 200. a 201. b 202. a 203. c 204. b
205. c 206. b
SSC
7. PROBLEMS BASED ON WORDS

Alphabetical/Dictionary Arrangement (a) 3, 1, 5, 2, 4 (b) 3, 5, 1, 2, 4


Example 1: Which word comes third if they are (c) 5, 1, 4, 3, 2 (d) 5, 4, 1, 3, 2
arranged  alphabeticaly  as  in  the Explanation: Option (a) is the correct answer.
dictionary? Logical Arrangement of Words
(a) Robber (b) Rocket E xa m p l e   3 :   A r r a n g e   t h e   f o l l o w i n g   i n   a
(c) Random (d) Restrict meaningful sequence.
Explanation: Option (a) is correct. Let us see the 1. Phrase 2. Letter
arrangement: 3. Word 4. Sentence
Random, Restrict, Robber, Rocket (a) 1, 2, 3, 4 (b) 1, 3, 2, 4
Example  2:  Arrange  the  given  words  in  the (c) 2, 3, 1, 4 (d) 2, 3, 4, 1
sequence  in  which  they  occur  in  the
Explanation: Option (c) is the correct because a
dictionary and choose the correct sequence.
group of letters makes a word. A group of
1. Repoint 2. Reptile words makes a phrase. A group of phrase
3. Repent 4. Repute makes a sentence. Thus, the correct order is
5. Report 2, 3, 1, 4

Exercise

Directions (1-120): Which one of the given re- C. Adult D. Adolescent


sponses would be a meaningful order of the fol- E. Child
lowing words?
(a) E, D, C, B, A (b) C, D, B, A, E
1. A. Honey B. Flower
(c) A, E, D, C, B (d) B, C, D, E, A
C. Bee D. Wax.
6. A. Sound B. Socks
(a) B, A, D, C (b) B, C, A, D
C. Shock D. Sharp
(c) D, C, B, A (d) A, C, D, B
E. Snooker
2. A. Plant B. Food
(a) E, D, C, A, B (b) D, C, E, B, A
C. Seed D. Leaf
(c) C, D, E, A, B (d) D, C, B, E, A
E. Flower
7. A. Seed B. Flower
(a) A, C, D, E, B (b) C, B, D, E, A
C. Soll D. Plant
(c) C, A, D, E, B (d) C, C, D, E, B
E. Fruit
3. A. Wound B. Writer
(a) D, B, E, A, C (b) B, E, D, A, C
C. Whiter D. Worst
(c) C, B, A, E, D (d) C, A, D, B, E
E. Worked
8. A. Live B. Litter
(a) E, C, B, A, D (b) A, D, C, E, B
C. Little D. Literary
(c) C, E, D, A, D (d) B, A, C, D, E
E. Living
4. A. Eagle B. Earth
(a) D, C, E, B, A (b) D, B, C, A, E
C. Eager D. Early
(c) C, D, B, A, E (d) C, B, D, E, A
D. Each
9. A. Conference B. Registration
(a) B, A, D, C, E (b) A, E, B, D, E
C. Participate D. Participate
(c) B, C, E, D, A (d) E, C, A, D, B
E. Representatives
5. A. Infant B. Old
SSC
(a) A, B, D, E, C (b) A, D, E, B, C 18. A. Fruit B. Flower
(c) D, A, E, B, C (d) D, E, A, C, B C. Seed D. Pollination
10. A. Stem B. Flower E. Bud
C. Root D. Leaves (a) C, E, B, D, A (b) D, B, E, C, A
E. Fruit (c) E, B, E, C, A (d) E, B, D, A, C
(a) C, A, D, B, E (b) C, A, E, B, D 19. A. Tree B. Seed
(c) E, B, A, D, C (d) D, C, A, E, B C. Flower D. Fruit
11. A. Doctor B. Fever E. Plant
C. Prescribe D. Diagnose (a) D, B, C, E, A (b) A, D, B, C, E
E. Medicine (c) B, E, A, C, D (d) B, A, C, D, E
(a) B, A, C, D, E (b) A, D, C, B, E 20. A. Butterfly B. Cocoon
(c) B, A, D, C, E (d) B, D, C, E, A E. Egg D. Worm
12. A. House B. Road (a) A, D, C, B (b) A, E, D, B
C. Room D. Hemlet (c) B, D, A, C (d) C, D, B, A
D. District 21. A. Yarn B. Plant
(a) C, B, A, D, E (b) C, A, D, B, E C. Saree D. Cotton
(c) C, A, B, D, E (d) C, A, B, E, D E. Cloth
13. A. Pulp B. Print (a) B, D, E, A, C (b) B, D, A, E, C
C. Paper D. Purchase (c) B, D, C, E, A (d) B, D, E, C, A
E. Publish 22. A. Elephant B. Cat
(a) A, C, B, E, D (b) A, D, E, B, C C. Mosquito D. Tiger
(c) A, B, C, E, D (d) A, E, D, B, C E. Whale
14. A. Book B. Words (a) E, C , A, B, D (b) A, C, E, D, B
C. Letters D. Sentences (c) C, B, D, A, E (d) B, E, A, D, C
E. Chapters F. Pages 23. A. Adult B. Child
(a) C, B, F, E, D, A (b) C, B, E, D, F, A C. Infant D. Boy
(c) C, B, D, E, F, A (d) C, B, D, F, E, A E. Adolescent
15. A. ROCK B. HILL (a) A, C, D, E, B (b) C, B, D, E, A
C. MOUNTAIN D. RANGE (c) B, C, E, D, A (d) B, C, D, A, E
E. STONE 24. A. Wall B. Clay
(a) A, C, D, B, E (b) E, A, B, C, D C. House D. Room
(c) D, C, B, E, A (d) E, B, C, D, A E. Bricks
16. A. Rain B. Monsoon (a) E, B, A, D, C (b) B, E, D, A, C
C. Rescue D. Flood (c) B, E, A, D, C (d) A, B, C, D, E
E. Shelter F. Relife 25. A. Letter B. Phrase
(a) A, B, D, E, C, F (b) A, B, C, D, E, F C. Word D. Sentence
(c) B, A, D, C, E, F (d) D, A, B, C, F, E (a) A, C, D, B (b) A, C, B, D
17. A. Accident B. Judge (c) B, C, A, D (d) D, C, A, B
C. Doctor D. Lawyer 26. Doctor, Fever, Medicine, Medical, Shop
E. Police (a) Medical Shop, Medicine, Fever, Doctor
(a) A, C, D, B, E (b) A, C, E, D, B (b) Fever, Doctor, Medical shop, Medicine
(c) A, B, C, D, E (d) A, B, E, D, C (c) Doctor, Medical Shop, Medicine, Fever
SSC
(d) Medicine, Doctor, Medical, Shop, Fever (c) C, D, B, A (d) D, C, B, A
27. A. Grasp B. Granite 36. A. Inventory B. Involuntary
C. Grass D. Graph C. Invisible D. iNVARIABLE
E. Grape E. Investigate
(a) E, A, B, C, D (b) E, A, C, B, D (a) D, B, E, C, A (b) D, E, A, C, B
(c) B, E, D, A, C (d) B, E, A, D, C (c) B, E, D, A, C (d) D, A, E, C, B
28. A. Electricity B. Dam 37. A. Exhaust B. Night
C. Lights D. River C. Day D. Sleep
E. Power House E. Work
(a) D, B, A, C, E (b) D, B, E, C, A (a) A, C, E, B, D (b) C, E, A, D, B
(c) D, B, C, A, E (d) D, B, E, A, C (c) C, E, A, B, D (d) C, E, B,A, D
29 A. Rain B. Vaporisation 38. A. Family B. Community
C. Water D. Condensation C. Member D. Locality
E. Cloud E. Country
(a) A, C, B, D, E (b) E, C, D, A, B (a) C, A, D, B, E (b) C, A, B, D, E
(c) C, B, E, D, A (d) B, C, E, D, A (c) C, A, B, E, D (d) C, A, D, E, B
30. A. Travel B. Destination 39. A. TORTOISE B. TORONTO
C. Payment D. Reservation C. TORPED D. TORUS
E. Availability of berth/Seat for reservation E. TORSEL
(a) A, B, C, D, E (b) B, C, E, D, A (a) B, E, C, A, D (b) B, E, C, D, A
(c) B, A, E, C, D (d) B, E, C, D, A (c) B, C, E, A, D (d) B, C, E, D, A
31. A. Study B. Job 40. A. Type B. Print
C. Examination D. Earn C. Open D. Save
E. Appointment E. Close
(a) A, C, E, B, D (b) A, B, C, D, E (a) C, D, A, B, E (b) C, E, D, B, A
(c) A, C, B, E, D (d) A, C, E, D, B (c) C, A, D, B, E (d) C, B, A, D, E
32. A. Cold B. Iron 41. A. Exploit B. Explosive
C. Sand D. Diamond C. Exponent D. Exposition
(a) D, C, B, A (b) A, B, C, D E. Explore
(c) C, B, A, D (d) A, B, D, C (a) A, C, D, E, B (b) A, E, B, C, D
33. A. Bus B. Bullock cart (c) A, E, C, B, D (d) A, B, E, C, D
C. Aeroplane D. Horse 42. A. Foundation B. Plastering
E. Rocket C. Building D. Painting
(a) D, B, A, C, E (b) C, D, A, B, E (a) A, B, C, D (b) A, C, B, D
(c) B, D, A, C, E (d) A, D, B, C, E (c) C, A, B, D (d) C, A, D, B
34. A. Income B. Fame 43. A. 15.04.1950 B. 14.05.1960
C. Education D. Employment C. 14.06.1960 D. 20.04.1950
(a) A, B, C, D (b) C, D, A, B E. 25.05.1960
(c) C, D, B, A (d) D, C, B, A (a) A, E, B, C, D (b) A, C, B, D, E
35. A. Plant B. Tree (c) A, D, B, E, C (d) A, B, D, E, C
C. Soil D. Employment 44. A. Amoeba B. Oyster
(a) A, B, C, D (b) C, D, A, B C. Worm D. Cow
SSC
(a) A, C, B, D (b) A, B, C, D (a) A, D, C, B, E (b) C, D, A, B, E
(c) D, C, B, A (d) C, B, D, A (c) E, B, D, A, C (d) B, D,C, E, A
45. A. Intricate B. Interview 53. A. Noble B. Nobilitary
C. Intransigent D. Interrogation C. Noblesse D. Nobility
E. Intravenous E. Nobble
(a) B, D, E, C, A (b) E, C, A, B, D (a) A, D, C, B, E (b) C, D, A, B, E
(c) D, B, C, E, A (d) C, E, B, A, D (c) E, B, D, A, C (d) B, D, C, E, A
46. A. Ocean B. Rivulet 54. A. Birth B. Death
C. Sea D. Glacier C. Childhood D. Infancy
E. River E. Adolescence F. Adulthood
(a) E, B, C, A, D (b) D, B, E, C, A G. Old age
(c) E, B, C, D, A (d) D, B, A, C, E (a) B, F, G, E, D, C, A (b) A,
47. A. Preposition D, C, E, F, G, B
B. Preparativiely (c) A, D, C, F, E, G, B (d) B,
C. Preponderate G, F, D, E, C, A
D. Prepossess 55. (a) Frankenstein (b) Frankincese
E. Prepossess (c) Frankalmoign (d) Frauendienst
(a) B, D, A, E, C (b) A, E, B, D, C 56. A. Cut B. Put on
(c) E, D, B, C, A (d) D, B, E, A, C C. Sew D. Measure
48. A. Sentence B. Word E. Chalk mark
C. Chapter D. Phrase (a) D, E, A, C, B (b) E, D, A, B, C
E. Paragraph (c) A, B, C, D, E (d) D, E, C, A, B
(a) D, C, A, B, E (b) B, A, D, E, C 57. A. Chest B. Forehead
(c) C, E, A, D, B (d) A, C, B, D, E C. Stomach D. Ears
49. A. Inhabit B. Ingentious E. Legs F. Ears
C. Inherit D. Influence (a) A, B, D, F, C, E (b) B, D, F, A, C, E
E. Infatuation (c) E, D, B, F, A, C (d) A, D, B, F, C, E
(a) A, B, C, D, E (b) E, D, A, B, C 58. A. Tune B. Forehead
(c) D, E, B, A, C (d) E, D, B, A, C C. Taste D. Truth
50. A. Major B. Captain E. Task
C. Colonel D. Brigadier (a) E, C, B, A, D (b) B, E, C, D, A
E. Lt. General (c) E, C, B, D, A (d) E, C, D, B, A
(a) E, D, C, A , B (b) E, A, D, B, C 59. A. Fruits B. Roots
(c) D, E, A, C, B (d) C, D, B, E, A C. Trunks D. Leaves
51. A. Dissident B. Dissolve E. Flowers F. Branches
C. Dissent D. Dissolute (a) B, C, F, D, E, A (b) C, B, E, A , F, D
E. Dissolution (c) A, C, F, D, B, E (d) B, C, F, A, D, E
(a) C, A, D, E, B (b) C, B, A, D, E 60. A. Paper B. Library.
(c) C, A, D, B, E (d) C, B, D, E, A C. Author D. Book
52. A. Noble B. Nobilitary E. Pen
C. Noblesse D. Nobility (a) B, D, C, E, A (b) C, B, A, E, D
E. Nobble (c) C, E, A, D, B (d) E, A, C, D, B
61. A. Grapes B. Wineyard
SSC
C. Factory D. Paper 71. A. Police B. Punishment
E. Distillation C. Crime D. Justice
(a) A, B, E, D, C (b) C, E, D, B, A E. Judgement
(c) C, A, D, E, B (d) D, B, C, A, E (a) C, A, B, D, E (b) A, B, D, C, E
62. A. Seed B. Fruit (c) E, D, C, B, A (d) C, A, D, E, B
C. Plant D. Stem 72. Which word will appear third in the dictionary
E. Flower order?
(a) A, C, D, E, B (b) A, C, E, D, B (a) Sentimentalize (b) Sententious
(c) A, E, C, D, B (d) A, C, E, B, D (c) Sentimentally (d) Sentinel
63. A. Foetus B. Child 73. Which word will come third in the dictionary or-
C. Baby D. Adult der?
E. Youth Rumbustious, Rumanian, Rumour, Ruminate,
Rumple
(a) E, D, B, C, A (b) A, B, D, C, E
(a) Rumbustious (b) Ruminate
(c) B, C, E, D, A (d) A, C, B, E, D
(c) Rumour (d) Rumple
64. A. Windows B. Walls
74. Which of the following words will appear 4th
C. Floor D. Foundation
when arranged in the order of dictionary?
E. Roof F. Room
(a) Carpet (b) Cart
(a) D, E, C, B, A, F (b) D, B, A, E, C, F
(c) Carry (d) Carpus
(c) D, A, E, F, B, C (d) D, C, E, F, B, A
75. A. Seed B. Plant
65. A. Leaf B. Root
C. Flower D. Fruit
C. Branch D. Stem
E. Tree
E. Flower
(a) A, B, C, D, E (b) B, C, D, E, A
(a) C, D, E, A, B (b) B, A, E, C, D
(c) A, E, D, C, B (d) A, B, E, C, D
(c) A, D, C, A, E (d) E, A, D, C, B
76. A. Leaf B. Stem
66. Which of the given words will come fourth in the
C. Root D. Flower
dictionary?
(a) D C A D (b) B D C A
Minister, Mineralogy, Minnow, Miniature, Mink
(c) C B A D (d) B A D C
(a) Mink (b) Miniature
77. A. Ocean B. Sea
(c) Minnow (d) Mineralogy
C. Pond D. River
67. Which of the given words will come third in the
dictionary? (a) C, A, C, D (b) B, C, D, A
Charm, Chasm, Chase, Chart (c) A, D, B, C (d) B, D, C, A
(a) Charm (b) Chasm 78. A. Centi B. Deca
(c) Chase (d) Chart C. Kilo D. Deci
68. (a) Gastrutus (b) Gatecrash (a) D, A, C, B (b) B, C, D, A
(c) Gauntlet (d) Garrusib (c) A, D, B, C (d) D, B, C, A
69. (a) Hick (b) Hide 79. A. Neonate B. Child
(c) High (d) Hay C. Infant D. Embryo
70. A. Presentation B. Recommenation (a) A, C, B, D (b) D, A, C, B
C. Arrival D. Discussion (c) D, C, A, B (d) C, A, D, B
E. Introduction 80. A. Plastering B. Painting
(a) C, E, B, D, A (b) A, B, C, D, E C. Foundation D. Walls
(c) E, D, C, B, A (d) C, E, A, D, B E. Ceiling
SSC
(a) C, D, A, B, E (b) A, B, C, D, E 89. A. Slunk B. Slung
(c) E, D, C, B, A (d) C, D, E, A, B C. Slump D. Slum
81. A. Grandfather B. Great grandfather (a) D, C, B, A (b) C, B, D, A
C. Grandson D. Son (c) C, A, D, B (d) A, B, C, D,
E. Father 90. A. Plant B. Food
(a) C, D, E, A, B (b) D, A, C, E, B C. Seed D. Leaf
(c) A, B, C, D, E (d) B, C, A, E, D E. Fruit
82. A. Book B. Words (a) E, B, C, B, A (b) C, B, D, E, A
C. Letters D. Sentences (c) C, A, D, E, B (d) A, C, E, D, B
E. Chapter 91. A. Gold B. Silver
(a) C, B, F, E, D, A (b) C, B, E, D, F, A C. Platinum D. Diamond
(c) C, B, D, E, F, A (d) C, B, E, F, E, A (a) D, A, B, C (b) C, D, A, B
83. Arrange the following in a logical order. (c) B, A, C, D (d) A, B, C, D
A. Open Text book 92. Amongst the following words which word appears
B. Attend Assembly second in order in the English dictionary?
C. Catch Bus (a) Complaint (b) Complication
D. Wear Uniform (c) Complement (d) Compose
E. Get into classroom 93. A. Line B. Angle
(a) D, B, C, E, A (b) D, C, E, A, B C. Square D. Triangle
(c) D, C, B, E, A (d) D, C, A, E, B (a) B, A, D, C (b) C, D, A, B
84. A. Collect B. Collinear (c) D, B, A, C (d) A, B, D, C
C. Collection D. Column 94. A. Childhood B. Adulthood
E. Collapse C. Infancy D. Adolescence
(a) E, A, C, B, D (b) E, A, B, C, D E. Bobyhood
(c) E, A, D, C, B (d) E, A, B, D, C (a) D, A, C, B, E (b) C, E, A, D, B
85. A. Evaporation B. Rain (c) B, E, A, D, C, (d) E, D, B, C, A
C. Sea D. Clourds 95. A. Advertise B. Admit
(a) A, B, C, D (b) D, A, B, C C. Addition D. Adhesive
(c) C, A, D, B (d) B, D, A, C (a) A, B, C, D (b) C, D, B, A
86. A. Critical B. Criterion (c) C, B, D, A (d) D, A, B, C
C. Crisis D. Crisp 96. A. Seed B. Fruit
(a) C, D, B, A (b) A, C, D, B C. Plant D. Food
(c) C, D, A, B, (d) D, C, A, B (a) BADC (b) ADBC
87. A. Lung B. Nostrils (c) ACBD (d) CABD
C. Windpipe D. Blood 97. A. Yearly B. Fortnightly
(a) A, B, C, D (b) B, C, A, D C. Monthly D. Daily
(c) A, C, D, B (d) D, C, B, A E. Weekly
88. A. House B. Street (a) E, B, C, D, A (b) D, E, B, C, A
C. Room D. Town (c) B, E, D, C, A (d) A, D, E, C, B
E. District 98. A. Apple B. Appreciate
(a) C, B, A, D, E (b) C, A, D, B, E C. Apply D. Application
(c) C, A, B, D, E (d) C, A, B, E, D E. Apartment
(a) 1 (b) 2
SSC
(c) 3 (d) 4 108. A. Destination B. Booking
99. A. Hut B. House C. Boarding D. Travel
C. Palace D. Bungalow E. Planning
(a) D, C, A, B (b) B, A, D, C (a) D, C, A, B, E (b) E, B, C, D, A
(c) A, D, C, B (d) A, B, D, C (c) A, B, C, D, E (d) C, D, E, A, B
100. A. Banal B. Banana 109. A. Genuine B. Gensesis
C. Banish D. Bandage C. Gender D. Gentle
E. Bandit E. General
(a) A, C, B, D, E (b) A, B, D, C, E (a) C, E, B, D, A (b) B, E, C, A, D
(c) A, B, D, E, C (d) A, C, B, E, D (c) D, E, C, B, A (d) A, E, D, C, B
101. A. Atomic Age B. Metallic Age 110. A. Police B. Punishment
C. Stone Age D. Alloy Age C. Crime D. Judge
(a) A, C, D, B (b) C, B, D, A E. Judgement
(c) B, C, A, D (d) D, C, B, A (a) C, A, D, E, B (b) C, A, B, D, E
102. A. PREMONITION B. PRELUDE (c) A, B, D, C, E (d) E, D C, B, A
C. PREMICE D. PRELIMINARY 111. A. Feedback B. Teach
E. PREMIUM C. Reteach D. Plan
(a) DBAEC (b) BDCEA E. Replan
(c) DBCEA (d) BDACE (a) D, B, A, E, C (b) A, D, B, E, C
103. A. Word B. Paragraph (c) C, E, B, D, A (d) B, D, A, E, C
C. Sentence D. Letter 112. A. Infancy B. Childhood
(a) A, C, D, B (b) A, D, C, D C. Adulthood D. Babyhood
(c) C, D, A, B (d) D, A, C, B E. Adolescence
104. A. Stem B. Flower (a) A, D, B, E, C (b) A, C, E, B, D
C. Root D. Leaves (c) A, B, C, D, E (d) A, D, E, B, C
E. Fruit 113. A. Spokesperson B. Sportsman
(a) C, A, D, B, E (b) C, A, E, B, D C. Spreadsheet D. Specification
(c) D, C, A, E, B (d) E, B, A, D, C E. Sophisticated
105. A. Particle B. Pardon (a) E, D, A, B, C (b) E, D, A, C, B
C. Parchment D. Parliament (c) D, E, A, B, C (d) E, D, B, A, C
E. Party 114. A. Sowing B. Manuring
(a) B, D, A, C, E (b) C, B, D, A, E C. Ploughing D. Harvesting
(c) C, D, B, E, A (d) B, C, D, A, E E. Watering
106. A. Furniture B. Forest (a) C, A, E, B, D (b) A, C, B, D, E
C. Wood D. Tree (c) A, B, D, C, E (d) A, E, B, D, C
(a) D, A, B, C (b) C, B, A, D 115. A. Study B. Employment
(c) B, D, C, A (d) B, C, B, D C. Pass D. Admission
107. A. Diagnosis B. Doctor E. Examination
C. Sick D. Treatment (a) D, A, E, C, B (b) A, C, D, E, B
E. Recovery (c) D, A, B, E, C (d) D, E, C, A, B
(a) B, A, C, D, E (b) D, E, A, C, B 116. A. Leaf B. Fruit
(c) A, B, C, D, E (d) C, B, A, D, E C. Stem D. Roots
SSC
E. Flower (a) D, A, E, C, B (b) A, D, E, C, B
(a) D, C, A, E, B (b) D, A, C, E, B (c) A, B, C, D, E (d) E, D, A, C, B
(c) D, C, A, B, E (d) C, D, E, A, B 119. A. Metal B. Ally
117. Which will appear fourth in the dictionary? C. Ore D. Purification
(a) Minortiy (b) Mistake E. Spare parts
(c) Mission (d) Miracle (a) C, A, B, D, E (b) C, A, B, E, D
118. A. Members B. City (c) C, D, A B, E (d) C, E, A, B, D
C. Village D. Community 120. Which will appear 3rd in the dictionary?
E. Community (a) colloquy (b) collinear
(c) collegiate (d) collision

Answer
1. (b) 2. (c) 3. (c) 4. (d) 5. (c) 6. (b)
7. (d) 8. (a) 9. (d) 10. (a) 11. (c) 12. (d)
13. (a) 14. (d) 15. (b) 16. (c) 17. (b) 18. (a)
19. (c) 20. (b) 21. (b) 22. (c) 23. (b) 24. (c)
25. (a) 26. (b) 27. (c) 28. (d) 29. (c) 30. (d)
31. (a) 32. (c) 33. (c) 34. (b) 35. (b) 36. (d)
37. (c) 38. (b) 39. (c) 40. (d) 41. (b) 42. (b)
43. (c) 44. (c) 45. (a) 46. (b) 47. (a) 48. (b)
49. (d) 50. (b) 51. (a) 52. (c) 53. (c) 54. (b)
55. (d) 56. (a) 57. (b) 58. (b) 59. (a) 60. (d)
61. (a) 62. (a) 63. (d) 64. (b) 65. (b) 66. (a)
67. (c) 68. (b) 69. (b) 70. (c) 71. (a) 72. (a)
73. (c) 74. (d) 75. (a) 76. (c) 77. (a) 78. (c)
79. (c) 80. (d) 81. (b) 82. (c) 83. (c) 84. (a)
85. (b) 86. (a) 87. (b) 88. (c) 89. (a) 90. (c)
91. (c) 92. (c) 93. (d) 94. (a) 95. (b) 96. (c)
97. (b) 98. (a) 99. (d) 100. (c) 101. (a) 102. (c)
103. (d) 104. (a) 105. (b) 106. (a) 107. (d) 108. (b)
109. (a) 110. (b) 111. (a) 112. (b) 113. (c) 114. (a)
115. (a) 116. (a) 117. (b) 118. (b) 119. (a) 120. (d)
SSC
8. DIRECTION
What is direction?
Generally we make our concept of direction from
the position of the sun. In fact, this is truth that sun
rises in the East and goes down in the West. Thus,
when we stand facing sunrise then our front is called
East while our back is called West. At this position
our left hand is in the northward and the right hand
is in the southward. Let us see the following direction
(i) (ii)
diagaram that will make your concept more clear.
Direction Diagram:

(i) (ii)

Concept of degree
Above diagram makes the following things clear:-
 There are four main directions and they are EAST,
WEST, NORTH and SOUTH.
 There are also four corner directions and they are
'NORTH- EAST', 'NORTH-WEST', 'SOUTH-
EAST' and 'SOUTH-WEST'.
 EAST and WEST are exactly opposite directions.
 NORTH and SOUTH are exactly opposite Points to remember
directions. (1) If our face is towards North, then after left turn
 North-East is exactly opposite to 'South-West'. our face will be towards West while after right
 North-West is exactly opposite to 'South-East'. turn it will be towards East.
 On paper, North is always on the top and South is (2) If our face is towards South, then after left turn
always at the bottom while East is towards our right our face will be towards East and after right
hand and West is towards our left hand. turn it will be towards West.
Concept of turn (3) If our face is towards East, then after left turn
Left turn = Anti clockwise turn our face will be towards North and after right
turn it will be towards South.
Right turn = Clockwise turn
(4) If our face is towards West, then after left turn
Let us understand it through pictorial presentation:-
our face will be towards South and after right
turn it will be towards North.
(5) If our face is towards North-West, then after
left turn our face will be towards South-West
and after right turn it will be towards North-
East.
(6) If our face is towards South-West, then after
SSC
left turn our face will be towards South-East
and after right turn it will be towards North-
West.
(7) If our face is towards South-East, then after
left turn our face will be towards North-East
and after right turn it will be towards South-
West.
(8) If our face is towards North East, then after
left turn our face will be towards North West
and after right turn it will be towards South Example 2: Rashmi's house is just opposite
East. to Mansi's house. Rashmi goes towards North-
Minimum Distance East and reaches Mansi's house. If Mansi has
to go Rashmi's house, then in which direction
should Mansi go?
(a) South-West (b) North-West
(c) South-East (d) North-East
h2 = b2 + p2, where (e) None of these
h = Hypotenuse Explanation: Option (a) is correct because
b = Base Rashmi's and Mansi's house are opposite to each
p = Perpendicular other. To reach Mansi's house, Rashmi has to go
straight towards North-East. Therefore, clearly,
AB = BA is the minimum or shortest distance to
to go Rashmi's house Mansi will have to go
reach A from B or to reach B from A.
exactly opposite to North-East and that direction
Remember this important rule is known as will be South-West. Let us see below:
'Pythagoras Theorem'.
Formats of the questions
Example  1:  Kamal  walks  from his  house T
towards East and after walking some distance
he takes left turn and reaches his office O. If
he has to return home driving exactly opposite
to office, which direction will he drive?
(a) North-West (b) North-East
(c) South-West (d) South-East
(e) None of these
Explanation: Option (c) is correct. Let us see:
M = Mansi's House
R = Rashmi's House.
SSC
Exercise
1. 'A' man starts from a point and walks 2km towards walks for 8 kms to E. How far is she from A to E?
North, turns towards the right and walks 2km, turns (a) 2 km (b) 3 km
right again and walks. What is the direction now
(c) 5 km (d) 8 km
he is facing?
8. Janaki started from her house and walked 2 kms
(a) South (b) South-East
towards North. Then she took a right turn and cov-
(c) North (d) West ered one kilometre. Then she took again a right
2. Mamatha walks 14 metres towards west, then turn and walked for 2 kms. In what direction is
thuns to her right and walks 14 metres and then she going?
turns to her left and walks 10 metres. Again turn- (a) North (b) East
ing to her left she walks 14 metres. What is the
(c) South (d) West
shortest distance (in metres) between her starting
9. Mohan starts from point A and walks 1 km to-
point and her present position?
wards south, turns left and walks 1 km. Then he
(a) 38 m (b) 28 m
turns left again and walks 1 km. Now he is facing.
(c) 24 m (d) 10 m
(a) East (b) West
3. Sohan started from point X and travelled forward
(c) North (d) South-west
8 km up to point Y, then tumed towards right and
10. Mira starts at point T, walks straight to point U
travelled 5 km up to point Z then turned right and
which is 4 ft. away. She turns left, at 90° and walks
travelled 7 km up to point A and then turned to-
to W which is 4 ft. away, turns 90° right and goes
wards right and travelled 5 km up to B. What is
3 ft. to P, turns 90° right and walks 2 ft. to Q, turns
the distance between point B and X?
left at 90° and goes to V, which is 1 ft. away and
(a) 1 km (b) 2 km
once again turns 90° right and goes to O, 3 ft,
(c) 3 km (d) 4 km away.; What is the distance between T and O?
4. Harit travelled 17 km to the east, he turned left (a) 4 ft. (b) 5 ft.
and went 15 km, he again turned left and went 17
(c) 7 ft. (d) 8 ft.
km. How far is the from the starting point?
11. Suresh starts from point, walks 2 miles towards
(a) 17 km (b) 2 km
1
(c) 15 km (d) 32 km south, turns right and walks 1 miles, turns left
2
5. Rama travels 10 km towards the north, turns left
and travels 4 km and then again turns right and 1
and walks miles and then he turns back. What
covers another 5 km and then turns right and trav- 2
els another 4 km. How far is he from the starting is the direction he is facing now?
point? (a) East (b) West
(a) 5 km (b) 10 km (c) South (d) North
(c) 15 km (d) 19 km 12. Ashok wants to book a ticket to Madurai. He starts
6. A man starts from a point, walks 2 km towards and covers 5 kms. to reach the Booking Office
north, turns towards his right and walks 2 km, turns which is in the East of his house. From there he
right again and walks. What is the direction now turns to the North towards the market by travel-
he is facing? ling 3 kms. From there he hurns left to his friend
(a) South (b) East Sandeep's house which is 5 kms away. Now he
(c) North (d) West has to get back to his house. How many kms. he
has to cover to reach his house?
7. Sandhya walks straight from point A to B which
is 2 kms away. She turns left, at 90° and walks 8 (a) 8 kms (b) 3 kms
kms to C, where she turns left again of 90° and (c) 5 kms (d) 6 kms
walks 5 kms to D. At D she turns left at 90° and
SSC
13. Arun started from point A and walked 10 kms East (a) EAST (b) NORTH
to point B, then turned to North and walked 3 kms (c) WEST (d) SOUTH
to point C and then turned West and walked 12 20. Rohan walks a distance of 3 kms towards North,
kms to point D, then again turned South and then turns to his left and walks for 2 kms. He again
walked 3 kms to point E. In which direction is he turns left and walks for 3 kms. At this point he
from his starting point? turns to his left and walks for 3 kms. How many
(a) East (b) South kms is he from the starting point?
(c) West (d) North (a) 5 kms (b) 3 kms
14. A man starts from his office and goes 5 km East. (c) 2 kms (d) 1 km
Then, he turns to the left and again walks for 3 21. A Driver left his village and drove North for 20
km. Again he turns left and walks 5 km. At what kms. after which he stopped for breakfast. The he
distance is he from the starting point? turned left and drove anothers 30 kms, when he
(a) 3 km (b) 4 km stopped for lutrich. After some rest, he again turned
(c) 6 km (d) 7 km left and drove 20 kms before stopping for evening
15. A man starts from a point, walks 4 miles towards tea. Once more the turned left and drove 30 kms
north and turns left and walks 6 miles, turns right to reach the town where he had supper. After
and walks for 3 miles and again turns right and eventing tea in which direction did he drive.
walks 4 miles and takes rest for 30 minutes. He (a) West (b) East
gets up and walks straight 2 miles in the same di- (c) North (d) South
rection and turns right and walks one mile. What 22. Rahim travelled straight from point E to F at a
is the direction he is facing? distance of 5 km. From F he turned left and trav-
(a) North (b) South elled 6 km and reached point G, there he took at
(c) South-east (d) West left turn and travelled 5 km to reach point H. He
16. Siddarth and Murali go for jogging from the same took another left turn and travelled 2 km and
point. Siddarth goes towards the east covering 4 reached point I. How far is he from the starting
kms. Murali proceeds towards the West for 3 kms. point?
Siddarth turns left and covers 4 kms and Murali (a) 3 km (b) 4 km
turns to the right to cover 4 kms. Now what will (c) 5 km (d) 7 km
be the distance between Siddarth and Murali? 23. A man is facing East, then he turns left and goes
(a) 14 kms. (b) 6 kms. 10 m, then turns right and goes 5 m, then goes 5 m
(c) 8 kms. (d) 7 kms. to the South and from there 5 m to West. In which
17. A starts from a point and walks 5 kms north, then direction in he from his original place?
turns left and walks 3 kms. Then again turns left (a) East (b) West
and walks 5 kms. Point out the direction in which (c) North (d) South
he is going now. 24. From her home Prerna wished to go to school.
(a) North (b) South From home she goes toward North and then turns
(c) East (d) West left and then turns right, and finally she turns left
18. Amit travelled 15 kms Eastward. the turned left and reaches school. In which direction her school
and travelled 5 kms, then turned left and travelled is situdated with respect to her home?
15 kms. How far was Amit from the starting point? (a) North - East (b) North - West
(a) 30 kms (b) 35 kms (c) South - East (d) South - West
(c) 15 kms (d) 5 kms Directions  (25-26):  In the following questions
19. A rat run 20' towards East and turns 10', and turns there are 6 checkposts A, B, C, D E and F. Check-
to right runs 9', and again turns to left runs 5' and post F F. Check-post F is 15 km to the North of D
then turns to left runs 12' and finally turns to left which is 25 km to the North-East of B. Checkpost
and runs 6'. Now what direction is the rat facing? A is 5 km West of E and 15 km to the South-West
SSC
of C. Check-posts B, A and E are in a straight line. elled 2 km then he would be in which direction
The checkposts B, and E are 30 km apart from from the original position?
each other. (a) South East Region (b)
25. If a jeep moves from E to F via A, B, and D, how North East Region
much distance will it have to cover? (c) South West Region (d)
(a) 70 km (b) 120 km Western Region
(c) 100 km (d) 90 km 33. Sonu travelled from a point A stragight to B, a
26. Which check-post is the farthest to the South-West distance of 12 km. He turned right and travelled 8
of D? km and reached point C. From that point took right
(a) A (b) B turn and travelled 6 km, and reached point D. How
far is he away from the starting point?
(c) C (d) D
(a) 10 km (b) 12 km
Directions (27 - 28): Kiran walks 20 m north, he
turns right and walks 30 m, then he turns right (c) 13 km (d) 14 km
and walks 35 m, then he turns left and walks 15 34. A and B start from a point simultaneously. A moves
m, then he turns left and walks 15 m. he again to his East and travels 2 km, and B moves to his
turns left and walks 15 m. south and travels 2 km. A takes turn 90° clock-
27. How far Kiran is from his starting point? wise and travels 2 km. B takes left turn 90°
anticlockwise and travels 2 km. Where would they
(a) 25 m (b) 15 m
be found from the starting point?
(c) 45 m (d) 30 m
(a) Both in South - East region
28. Which directions is Kiran facing now?
(b) Both in East region
(a) East (b) West
(c) A in East and B in North region
(c) North (d) South
(d) A in south and B in North region
Directions  (29-30):  Study the following
35. A man starts walking from a point and walks 12
infromation to answer the questions given below:
kms towards north. He turns 90° left and walks a
Ramu walks 2 km towards North and turn to his right distance and stopped. If the distance between ini-
and walks 4 km more. He then turns to his right and tial point and final positions is 13 kms. how much
walks 4 km and turns again to his right and walks an- distance he travelled after turning from the North?
other 4 km.
(a) 1 km (b) 5 km
29. Which direction is Ramu facing now?
(c) 7 kms (d) 2 kms
(a) East (b) North
36. A child walks 25 feet towards North, turns right
(c) South (d) West and walks 40 feet, turns right again and walks 45
30. How far away is Ramu from the starting point? feet. He then turns left and walks 20 fee. He turns
left again walks 20 feet. Finally, he turns to his
(a) 2 km (b) 4 km left to walk another 20 feet. In which direction
(c) 6 km (d) 8 km is the child from his starting point?
31. Ravi started walking from his houses east direc- (a) North (b) South
tion to stop which is 3 km away. Then he set off in (c) West (d) East
the bus straight towards his right to the school 4 37. A man drives his car 50 km to wards eastward
km away. What is the crow flight distance from direction. He turned right went for 30 km, then he
his house to the school? turned west and drive for 10km. How far is he
(a) 1 km (b) 4 km from the starting point?
(c) 6 km (d) 12 km (a) 50 km (b) 60 km
32. Raju moved to his North - West side for 2 km. (c) 100 km (d) 20 km
From there he turned 90° clockwise and moved 2
km. From there he turned 90° clockwise and trav-
SSC
38. Raju facing North and moves 20 km, then he 43. A tourist drives 10 km. towards west and turns to
turned to his right and move 20 km and then the left and takes a drive of another 4 km. He then
moves 10 km in North-East, then he turned to his drives towards east another 4 km and then turns
right and moves 20 km and then he turned to his to his right and drives 5 km. Afterwards he turns
right and moves 20 km, and again he turned to his to his left and travels 6 km. In which direction is
left and moves 20 km. Now in which direction he from the starting point?
Raju is facing? (a) North (b) East
(a) South-East (b) North-East (c) West (d) South
(c) South-West (d) North-West 44. Mohan travelled westward 5 kmm., turned left and
39. A child is looking for his father. He went 90 metres travelled 3 kms. turned right and travelled 9 kms.
in the east before turning to his right. He went 20 He then travelled north 3 kms. How far he is from
metres before turning to his right again to look for the starting point?
his father at his uncle's place 30 metres from this (a) 5 kms. (b) 3 kms.
point. His father was not there. From here he went (c) 6 kms. (d) 14 kms.
100 metres to his north before meeting his father
45. Leela starts from a point and walks 1 km east and
in a street. How far did the son meet his father
then turns left and walks 2 km and turns right and
from the starting point?
walks 2 km again. She starts towards a point 2 km
(a) 80 m (b) 100 m to-wards her right, from left. How far is she from
(c) 260 m (d) 140 m the starting point?
40. K is a place which is located 2 km away in the (a) 3 km (b) 4 km
north-west direction from the capital P. R is an- (c) 5 km (d) 2 km
other place that is located 2 km away in the south-
46. Karan facing towards south moved straight 2 km
west direction from K. M is another place and that
and from there turned to his right 90° and trav-
is located 2 km away in the north-west direction
elled 2 km. Then he took a 45° turn to his left and
from R.T is yet another place that is located 2 km
dravelled 1 km. Where would he be now with re-
away in the south-west direction from M. In which
spect to the starting point?
direction is T located in relation to P?
(a) South region (b) South-east region
(a) South-west (b) North-west
(c) North-west region (d)
(c) West (d) North
South-west region
41. Babu is Rahim's neighbour and his house is 200
47. In a square-shaped field A, B, C, D persons are
metres away in the north west direction. Joseph is
standing at the middle of each side. You have to
Rahim's neighbour and his house is located 200
bear in mind the directions to be followed as shown
metres away in the south west direction. Gopal is
in the figure.
Joseph's neighbour and he stays 200 metres away
in the south east direction. Roy is Gopal's A
neighbour and his house is located 200 metres E N

away in the north east direction. Then where is D B


the position of Roy's house in relation to Babu's?
S W
(a) South east (b) South west C
(c) North (d) North east
42. Peter walked 8 kms. west and turned right and 1
If A moves clockwise 2 sides and D moves anti-
walked 3 kms. The again he turned right and 2
walked 12 kms. How far is he from the starting 1
point? clockwise 2 sides they will be in
2
(a) 7 (b) 8 (a) South and West region
(c) 4 (d) 5 (b) North and East region
SSC
(c) South and North region (a) North (b) West
(d) North and West region (c) East (d) South
48. My friend and I started simulta-neously towards 55. Ganesh cycles towards South-West a distance of
each other from two places 100 m apart. After 8 m, then he moves toward East a distance of 20
walking 30 m. my friend turns left and goes 10m, m. From there he moves towards North-East a dis-
then he turns right and goes 20 m and then turns tance of 8 m, then he moves towards West a dis-
right again and comes backs to the road on which tance of 6 m. From there he moves towards North-
he had started walking. If we walk with the same East a distance of 2 m. Then he moves towards
speed, what is the distance between us at this point West a distance of 4 m and then towards South-
of time? West 2 m and stops at that point. How far is the
(a) 50 m (b) 20 m from the starting point?
(c) 30 m (d) 40 m (a) 12 m (b) 10 m
49. A man started walking West. He turned right, then (c) 8 m (d) 6 m
right again and finally turned left. Towards which 56. From my house I walked 5 km towards North. I
direction was he walking now? turned right and walked 3 km. Againt I went one
(a) North (b) South km to the south. How far am I from my house?
(c) West (d) East (a) 7 km (b) 6 km
50. Sudha travels 8 km to the South. Then she turns (c) 4 km (d) 5 km
to the right and walks 4 km. Then again she turns 57. A and B are standing at distance of 20 km from
to her right and moves 8 km forward. How many each other on a straight East-West road. A and B
km away is she from the starting point? start walking simultaneously eastwards and west-
(a) 7 (b) 6 wards respectively and both cover a distance of 5
(c) 4 (d) 8 km. Then A turns to his left and walks 10 km. B
turns to his right and walks 10 km at the same
51. One evening, Raja started to walk toward the Sun.
speed. Then both turn to their left and cover a dis-
After walking while, he turned to his right and
tance of 5 km at the same speed. What will be the
again to his right. After walking a while, he again
sistance between them?
turned right. In which direction is he facing?
(a) 10 km (b) 30 km
(a) South (b) East
(c) 20 km (d) 25 km
(c) West (d) North
58. A man starts from a point, walks 8 km towards
52. Vivek and Ashok start from a flxed point. Vivek
North, turns right and walks 12 km, turns left and
moves 3 km north and turns right and then covers
walks 7 km, turns and walks 24 km towards South,
4 km. Ashok moves 5 km west and turns right and
turns right and walks 12 km. In which direction is
walks 3 km. Now how far are they apart?
he from the starting point?
(a) 10 km (b) 9 km
(a) North (b) South
(c) 8 km (d) 6 km
(c) West (d) East
53. Five boys A, B, C, D, E are sitting in a park in a
59. Rama left home and walked 5 km southwards,
circle. A is facing South-West, D is facing South-
turned right and walked 2 km and turned right and
East, B and E are right opposite A and D respec-
walked 5 km and turnel left and walked 5 km. How
tively and C is equidistant between D and B.
many kilometres will she have to walk to rach her
Which direction is C facing?
home straight?
(a) West (b) South
(a) 5 (b) 7
(c) North (d) East
(c) 17 (d) 15
54. If a man on a moped starts from a point and rides
60. Daily in the morning the shadow of Gol Gumbaz
4 km South, then turns left and rides 2 km to turn
falls on Bara Kaman and in the evening the shadow
again to the right to ride 4 km more, towards which
of Bara Kaman falls on Gol Gumbaz exactly. So
direction is he walking now?
SSC
in which direction is Gol Gumbaz to Bara Kaman? 67. Gopal started walking 2 km straight from the
(a) Eastern side (b) Western side school. Then he turned right and walked 1 km.
(c) Northern side (d) Southern side Again he turned right and walked 1 km to reach
his house. If his house is south-east from his
61. A man starts from his house and walked straight
school, then in which direction did Gopal start
for 10 metres towards North and turned left and
walking from the school?
walked 25 metres. He then turned right and walked
5 metres and again turned right and walked 25 (a) East (b) West
metres. (c) South (d) North
Which direction is he facing now? 68. If the clock read 6.20 and if the minute hand points
(a) North (b) East North-East, in which direction will the hour band
point?
(c) South (d) West
(a) West (b) South-East
62. A and B start walking from the same point. A goes
north and covers 3 km, then turns right and covers (c) East (d) North-West
4 kms. B goes west and covers 5 kms, then turns 69. I go 5 km. cast then turn right and go 8 km. Then
right and covers 3 kms. How far apart are they turn left and go 5 km. and then I turn left and I go
form earch other? 8 km. At what distance I am from the starting
(a) 10 km (b) 9 km point?
(c) 8 km (d) 5 km (a) 13 km. (b) 10 km.
63. Radhika went 50 metres south from house then (c) 7 km. (d) 6 km.
turned left and went 20 metres then turning, to 70. If X stands on his head with his face towards south,
north she went 30 metres. to which direction will his left hand point?
In which direction is her home from this point? (a) East (b) West
(a) North (b) South-West (c) North (d) South
(c) East (d) Sorth-West 71. I drove East for 5 miles, then drove North 3 miles,
64. 'X' is in the East of 'Y' which is in the North of 'Z'. then turned to my left and drove for 2 miles and
If 'P' is the East of 'Z' then in which direction of again turned to my left. Which direction am I go-
'X', is 'P' ing now?
(a) South-West (b) South-East (a) South (b) North
(c) South (d) West (c) West (d) North-west
65. A walks towards north 4 kms. and turns right and 72. Facing the east Gopi walks straight 4 kms, turns
walks 5 kms. Then he turns towards south and left and walks 3 kms and again turns left and walks
walks 2 kms. Again, he takes a turn towards west, 4 kms. How far is he now from the starting point?
walks 3kms and stops a while. Then, he further (a) 2 km. (b) 3 km.
walks 2 kms to A. What is the distance of A from (c) 10 km. (d) 11 km.
his starting point? 73. If A stands on his head with his face towards north,
(a) 16 kms. (b) 12 kms. in which direction will his left hand point?
(c) 2 kms. (d) 4 kms. (a) North-East (b) North
66. Sujit travelled 15 km to the west, then turned right (c) East (d) North-West
and travelled 8 km. He turned left and travelled 9 74. A car travelling from south covers a distance of 8
km, then turned back and travelled 13km. Then kms, then turns right and runs another 9 kms and
he turned right and travelled 8 km. How far is he again turns to the right and was stopped. Which
from the starting point? direction does it face now?
(a) 17 km (b) 9 km (a) South (b) North
(c) 11 km (d) 7 km (c) West (d) East
SSC
75. A walks 3 kms northward and then he turns left (c) 20 km (d) 30 km
and goes 2 km. He again turns left and goes 3 km. 83. Eight People A, B, C, D, E, F, G and H are
He turns right and walks straight. In which direc-
tion is he walking now? N
A
(a) East (b) West H B
(c) North (d) South WG CE
76. 'A' walks southwards, then turns right, then left placed in F D
and then right. In which direction is he from the E
starting point? S
(a) South (b) East
(c) West (d) North this manner. All are facing outwards. If all of them
move two positions in clockwise direction, then
77. Ranga starts from his house towards south and
positions in clockwise direction, then the direc-
walks 60 m then turns towards east and walks 40
tions, H will be facing, is
m and turns towards north and walks 30 m and
stops. How far is he ways from his house? (a) North - East (b) North
(a) 30 m (b) 40 m (c) North - West (d) East
(c) 50 m (d) 35 m 84. Lakshmi walked 2 furlongs north from her house
and took a turn to left and continued to walk an-
78. A boy starts walking towards West, he turns right
other one kilometer and finally she turned left and
and again he turns right and then turns left at last.
reached the school. Which direction is she facing
Towards which direction is he walking now?
now?
(a) West (b) North
(a) West (b) North
(c) South (d) East
(c) East (d) South
79. I stand with my right hand extended side-ways to-
85. You are going straight, first eastwards, then turn
wards South. Towards which direction will my
to the right, then right again, then left. In which
back be?
direction would you be going now?
(a) North (b) West
(a) East (b) West
(c) East (d) South
(c) South (d) North
80. I started walking down a road in the morning fac-
86. Eight people A, B, C, D, E, F, G and H are placed
ing the Sun. After walking for sometime I turned
as shown in the diagram. All are facing in the out-
to my left. Then I turned to my right. In which
ward direction. If all of them move anticlockwise
direction was I poing then?
to three places then.
(a) East (b) West
(c) North (d) South N
A
81. I am facing West. I turn 90° in clockwise direc- H B
tion, then 135° in anticlockwise direction. What G C
direction am I facing? F D
(a) South (b) North E

(c) North-West (d) South-West (a) B is facing West


82. Rohit drives his car from Bangalore towards (b) E is facing East
Myscore. At a distance of 80 km. he turns right (c) H is facing North-West
and proceeds for 50 km. Again he turns right and
(d) A is facing South
travels for 70 km. Finally he turns again towards
87. If Ahmed travels towards North from his house,
right and travels for 50 km and stops there. How
then to left, then to South covering equal distances
far is Rohit from Bangalore?
in each direction to reach Sohan's house, in whic
(a) 10 km (b) 40 km
direction is Ahmed's house now?
SSC
(a) East (b) South indicates his path of journey correctly?
(c) North (d) West (a) P (b) P
88. You go North, turn right, then right again and then (c) C (d) C
go to the left. In which direction are you now? 95. Roy walks 2 km to East, then turns North-West
(a) South (b) East and walks 3 km. Then he turns South and walks 5
(c) West (d) North km. Then again he turns West and malks 2 km.
89. Ram and Shyam started walking in opposite di- Finally he turns North and walks 6 km. In which
rections from a point. Ram covered 7 km and direction, is he from the starting point?
Shyam covered 5km. Ram turned right and walked (a) South–West (b) South – East
3km. How far are they from each other? (c) North – West (d) North – East
(a) 8 km (b) 10 km 96. Rana travels 10km North, turns left and travels 4
(c) 12 km (d) 14 km km and then again turns right and covers another
90. Roopa starts from a point and walks 15 meter to- 5 km. He then turns to right–hand side and travels
wards west, turns left and walks 12 metre, turns another 4 km. How far is he from the point of start-
right again and walks. What is the direction she is ing of his journey?
now facing? (a) 5 km (b) 4 km
(a) South (b) West (c) 15 km (d) 10 km
(c) East (d) North 97. Mrs. Kiran starts from Gandhi Square, Kolkata,
91. X starts from a point P towards west and reaches drives her car towards west 6 kms and turning to
next point Q. From there he turns towards North her right she drives 4 kms, then again turns the
and reaches place R. From there he takes turn to- car to her right and moves 6 kms. How far is she
wards South-West and reaches place T. The path from the starting point?
of his journey is correctly shown by which of the (a) 6 kms (b) 16 kms
given responses. (c) 4 kms (d) 8 km
figure 98. A man walks 7 km towards north before taking
(a) P (b) P left turn and walks further 5 km. Then the takes
(c) C (d) C left turn and walks 15 km. Finally he takes left
turn again and walks 5 kim. How much distance
92. A starts walking from a point P. A goes westward
is he away from the starting point?
and covers a distance of 4 km and then turns to
his right and walks 3 km. How far A is from the (a) 8 km (b) 12 km
starting point? (c) 15 km (d) 16 km
(a) 7 km (b) 9 km 99. Seeta starts from a point, walks 2 km towards
(c) 2 km (d) 5 km north, turns towards her right and walks 2 km, turns
right again and walks, What is the direction she is
93. A man starts his journey facing the sun early morn-
facing now?
ing. He then turns right and walks 2 km. He then
walks 3 km after turning right again. Which is the (a) East (b) West
direction he is facing now? (c) South (d) North
(a) North-East (b) North 100. Ramesh erive his car 4 km to east, then takes left
(c) West (d) South turn and drives 6 km. He again turns to his left
hand side and travels 4 km. How many km is
94. Raju starts from a place A towards South and
Ramesh away from his starting place?
reaches second place D. From there he takes turn
towards N-W and reaches point E. From there he (a) 6 km (b) 10 km
takes turn towards N-E and reaches place F. From (c) 4 km (d) 14 km
there he takes turn towards S-E and reaches place
G. From there he takes turn towards South and
reaches place H. Which of the following figures
SSC
101. Shyam was facing East. He walked 5 km forward walks 3 km and stops for a while. Then he further
and then after turning to his right walked 3 km. walks 2 km. What is the distance of Sanmitra from
Again he turned to his right and walked 4 kms. starting point?
After this he turned back. Which direction was he (a) 16 k.m. (b) 2 k.m.
facing at that time? (c) 4 k.m. (d) 3 k.m.
(a) East (b) West 108. Suhas travelled km. towards East, then turned to-
(c) North (d) South wards North and travelled 15 km and turned to-
102. Raju is standing facing north. He goes 30 metres wards West and travelled 15 km. How far is he
ahead and turns left and goes for 15 metres. Now from the starting point?
he turns right and goes for 50 metres and finally (a) 15 km (b) 30 km
turns to his right and walks. In which direction is (c) 45 km (d) 0 km
he heading?
109. Ramesh walked 3 km. towards West and turned to
(a) North (b) East his left and walked 2 km. He, then turned to his
(c) South (d) West right and walked 3 km. Finally, he turned to his
103. Vijay starts from his office and walks 4 km to- right again and walked another 2 km. In which
wards north. Then he turns right and walks 2km, direction is Ramesh from his starting point now?
then turns right and walks 6 km, then again turns (a) East (b) West
right and walks 2 km and then turns right and walks (c) North (d) South
2 km. How for is he now from the starting point?
110. A travelled westward 8 km, turned left and trav-
(a) 0 km (b) 6 km elled 3 km, turned right and travelled 9 km. He
(c) 12 km (d) 16 km then travelled north 3 km. How far is he from the
104. Sanmitra starts from his house and walks 3 km starting point?
towards north. Then he turns right and walks 2 (a) 15 km (b) 17 km
km and then turns right and walks 5 km, then turns (c) 19 km (d) 11 km
right and walks 2 km. and then again turns right
111. Deepa starts walking northwards and after a while,
and walks 2 km. Which direction is he facing now?
she turns to her right. After walking some distance,
(a) North (b) South she turns to her left and walks a distance of one
(c) West (d) East kilometer. She then turns to her left again. She
105. Rama is standing at a point facing north. He walks then turns to her left again. In which direction is
10 km straight, turns left and walks another 15 she moving now?
km straight and finally turns left and walks 10 km. (a) North (b) West
How far is he from the starting points now? (c) East (d) South
(a) 10 km (b) 5 km 112. Ram walked 4 km West of his house and then
(c) 12 km (d) 15 km turned to South covering 8 km. Finally he moves
106. Raju is Ramu's neighbour and he stays 100 metres 6 km towards East and then again 2 km West. How
away towards southeast. Venu is Raju's neighbour far is he from his initial position?
and he stay 100 metres away towards southwest. (a) 4 km (b) 8 km
Khader is Venu's neighbour and he stays 100 (c) 10 km (d) 12 km
metres away to wards northwest. Then where is
113. A, B, C, D and E are standing in a line facing
the position of Khader's home in relation to
North. E is standing 40 metres left to B. A is stand-
Ramu's?
ing 40 metres left to B. A is standing 20 metres
(a) South-East (b) South-West left to C. D is standing 20 metres right to E and 50
(c) North-West (d) North metres right to C. What is the distance between A
107. Sanmitra walks 4 kms. towards north, turns right and D?
and walks 5 k.m. Then he turns towards south and (a) 50 metres (b) 60 metres
walks 2 k.m. Again he takes a turn towards west (c) 70 metres (d) 80 metres
SSC
114. X walks southwards and then turns right, then left (c) 15 km (d) 13 km
and then right. In which direction is he moving 121. Mitan travelled 12 kms southward, then turned left
now? and travelled 10 kms, then turned left and trav-
(a) South (b) North elled 12 km. How far was Mitan from the starting
(c) West (d) South West point?
115. Kalpana travelled from point B to straight to C a (a) 8 km (b) 10 km
distance of 8 feet. She turned left and walked 5 (c) 12 km (d) 14 km
feet away. Again she turned left and walked 7 feet 122. Mohan starts from point A and walks 1 km to-
and finally turned left and walked 5 feet. How far wards south, turns left and walks 1 km. Then turns
is she from the starting point? left again and walks 1 km. Now which direction
(a) 3 feet (b) 4 feet he is facing?
(c) 1 feet (d) 5 feet (a) East (b) West
116. A, B, C, D and E are standing in a line facing (c) North (d) South-West
North. E is standing 40 metres left to B, A is stand- 123. I went 15 m to the north, then I turned west and
ing 20 metres left to C. D is standing 20 metres covered 10 m, then I turned south and covered 5
right to E and 50 metres right to C. What is the m and then turned east and covered 10 m. In which
distance between A and B? direction am I from the starting point?
(a) 70 metres (b) 80 metres (a) East (b) West
(c) 90 metres (d) 100 metres (c) North (d) South
117. A man started to walk East. After moving a dis- 124. A person moves 400 metres to East. He turns to
tance he turned to his right. After moving a dis- his left and moves 400 metres. Then again he turns
tance he turned to his right again. After moving a to his loeft and walks 400 metres. Finally he turns
little he turned in the end to his left. In which di- to his right and moves 100 metres. Now how far
rection was he going now? awys in metres. he is from the starting point?
(a) North (b) South (a) 1300 (b) 900
(c) East (d) West (c) 800 (d) 500
118. Ramesh went 20 m to the east, then he turned left 125. Anita drives from point A towards north and trav-
and after 15 m turned right and went 25 m and els 30 kms. She then turns to her right and travels
then turned right and went 15 m. How far was 4 kms, and then again turns to the right and drives
Ramesh from the starting point? straight for 30 kms. How much distance she to
(a) 60 m (b) 35 m cover to go straight to the starting point?
(c) 40 m (d) 45 m (a) 26 kms (b) 8 kms
119. A taxt driver commenced his journey from a point (c) 22 kms (d) 4 kms
and drove 10km towards North and turned to his 126. Ravi in his evening walk travelled towards the sun
left and drove another 5 km. After waiting to meet 3 km, then he turned to his left walked 2 km. He
one of his friends, he turned to his right and again turned to his right and walked 3 km. Finally,
continuted to drive another 10 km. He has cov- he turned to his right walked another 2 km. In
ered a distance of 25 km so far but in which direc- which direction is Ravi walking now?
tion he now may be? (a) East (b) West
(a) North (b) East (c) North (d) South
(c) West (d) South 127. A man starts for his office in the North direction.
120. Sita cycled 8 km southward from her home turned He turns to his left, and then to his right, and again
right and cycled 5 km, turned right and cycled 8 to his right. In which direction will he be facing?
km, turned left and cycled 10 km. How many kms (a) South (b) West
will she have cycle to reach straight home?
(c) East (d) North
(a) 8 km (b) 10 km
SSC
128. A man goes 5 km East, then he turns right and 135. Ram starts from his house and walks 4 kms North,
goes 4 km, then he turns left and goes 5 km. Which then 3 kms West, then 8 kms South. How many
direction is he facing now? kilometres away from his home was he?
(a) North (b) South (a) 6 kms (b) 7 kms
(c) East (d) West (c) 5 kms (d) 8 kms
129. Two cars leave the same place at the same time. 136. John and Khan start together from a particular
One runs at 20 km/hr towards north and the other point in the opposite direction of motorcycle. The
at 15 km/hr towards east. What will be the dis- speed of John is 25 km/hr and Khan is 35 km/hr.
tance between them (in km) after 2 hours of trav- What will be the distance between them after 15
elling? minutes?
(a) 25 (b) 50 (a) 15 km (b) 20 km
(c) 70 (d) 10 (c) 5 km (d) 10 km
130. Prakash travelled 6 km north ward, then turned 137. A cyclist goes 30 km to North and then turning to
left and travelled 4 km, then turned left and trav- East he goes 40 km. Again he turns to his right
elled 6 km. How far was Prakash from the start- and goes 20 km. After this he turns to his right
ing point? and goes 40 km. How far is he from his starting
(a) 6 km (b) 4 km point?
(c) 10 km (d) 8 km (a) 20 km (b) 10 km
131. Arun travels 8 km towards the North, turns left (c) 25 km (d) 40 km
and travels 3 km and then again truns right and 138. A man travels 3 km in the East and turns to South
covers another 4 km and then turns right and trav- and moves 4 km. How far is he from the starting
els another 3 km. How far is he from the starting point?
point? (a) 5 km (b) 6 km
(a) 18 km (b) 11 km (c) 2 km (d) 10 km
(c) 12 km (d) 15 km 139. Ganesh is standing at a point. He walks 20 m to-
132. In a double track rall route, two trains start simul- wards the East and further 10 m towards the South;
taneously from the opposite stations, 200km apart. then he walks 35 m towards the East. What is the
The train A runs westwards at a speed of 50 kmph straight distance inmetres between his starting
and the opposite train B runs eastwards at 40 kmph. point and the point where he reached last?
After 3 hours, what will be the distance between (a) 20 m (b) 5 m
these two trains? (c) 10 m (d) 36 m
(a) 30 kms (b) 50 kms 140. A boy walks northwards. After a while he turns
(c) 80 kms (d) 70 kms towards his right and a little further to his left.
133. A man walks 4km to the east turns left walks 4km Finally after walking a distance of one kilometre,
and turns left and walks 4km. Which direction is he turns to his left again. In which direction he is
he facing? moving now?
(a) South (b) East (a) North (b) South
(c) West (d) North (c) East (d) West
134. Khan travels 8 km from East to West and turns
right to travel 2km. Now he turns right and travels
5 km. In which direction is he now positioned with
reference to the starting point?
(a) South-East (b) North-West
(c) West (d) North
SSC
Answers
1. (a) 2. (c) 3. (a) 4. (c) 5. (c) 6. (a) 7. (b) 8. (c) 9. (c) 10. (d)
11. (d) 12. (b) 13. (c) 14. (a) 15. (b) 16. (d) 17. (b) 18. (d) 19. (b) 20. (d)
21. (b) 22. (b) 23. (c) 24. (b) 25. (a) 26. (b) 27. (d) 28. (b) 29. (d) 30. (a)
31. (b) 32. (b) 33. (a) 34. (a) 35. (b) 36. (d) 37. (a) 38. (d) 39. (b) 40. (c)
41. (a) 42. (d) 43. (d) 44. (d) 45. (b) 46. (d) 47. (c) 48. (b) 49. (a) 50. (c)
51. (a) 52. (b) 53. (d) 54. (d) 55. (b) 56. (d) 57. (a) 58. (b) 59. (b) 60. (b)
61. (b) 62. (b) 63. (d) 64. (c) 65. (c) 66. (c) 67. (a) 68. (b) 69. (b) 70. (a)
71. (a) 72. (b) 73. (c) 74. (b) 75. (b) 76. (c) 77. (c) 78. (b) 79. (b) 80. (a)
81. (d) 82. (a) 83. (a) 84. (d) 85. (c) 86. (a) 87. (a) 88. (b) 89. (c) 90. (b)
91. (c) 92. (d) 93. (c) 94. (c) 95. (c) 96. (c) 97. (c) 98. (a) 99. (c) 100. (a)
101. (b) 102. (b) 103. (a) 104. (a) 105. (d) 106. (c) 107. (b) 108. (a) 109. (b) 110. (b)
111. (b) 112. (b) 113. (c) 114. (c) 115. (c) 116. (c) 117. (b) 118. (d) 119. (a) 120. (c)
121. (b) 122. (c) 123. (c) 124. (d) 125. (d) 126. (c) 127. (c) 128. (c) 129. (b) 130. (b)
131. (c) 132. (d) 133. (b) 134. (c) 135. (c) 136. (a) 137. (b) 138. (a) 139. (a) 140. (d)
SSC
9. CLOCK & CLENDAR
Clock O’clock will the hand of a clock  be in the
A clock has 2 hands, the smaller one is called the same straight line but not together?
hour hand or short hand while the larger one is
called the minute or long hand. (a) min past 9
Important points
1. In every hour
(b) min past 8
(a) Both the hands coincide once. At this
point the angle between them is 0°.
(b) The hands are straight (point in opposite (c) min past 8
directions) once. At this point the angle
between them is 180°. (d) min past 7
(c) The hands are twice perpendicular to each
other. At this point the angle between them Explanation: Option (c) is correct because
is 90°. at 8 O’clock the hour hand is at 8 and the
2. ( a ) In 1 mi n t h e mi n u t e h a n d c o v e r s minute hand is at 12. Thus, the two hands
are 20 min spaces apart.
To be in the same straight line but not to-
gether, they will be 30 min spaces apart.
(b) In 12 h the hour hand covers 360° The minute hand will gain (30-20) min =
T h u s , i n 1 mi n t h e h o u r h a n d c o v e r s 10 min spaces over the hour hand.
55 min spaces are gained by hour hand in 60
min.
10 min spaces will be gained by hour hand
Thus, in one minute, the minute hand gains
in min = min
, than the hour hand. The hands will be in the same straight line
3. (a) When the two hands are at right angles, but not together at min past 8.
they are 15 min spaces apart.
(b) When the two hands are in opposite Example 2: Find the angle between the hour
directions, they are 30 min spaces apart. hand and the minute hand of a clock when
(c) In 60 min the minute hand gains 55 min the time is 5 : 35.
on the hour hand.
(d) The minute hand moves 12 times as fast (a) (b)
as the hour hand.
4. (a) Too fast if a clock indicates 8 : 15, when (c) (d)
the correct time is 8, it is said to be 15 min
too fast. Explanation: Option (d) is correct because
(b) Too slow if a clock indicates 8 : 30, when Angle traced by hour hand in 12 h = 360°.
the correct time is 8 : 45, it is said to be 15 A n gl e t r a c e d b y t h e h o u r h a n d i n
min too slow.
Formats of the questions
Example 1: At what time between 8 and 9
SSC
Angle traced by minute hand in 60 min = Number of odd days Day of the week
360° 0 Sunday
Angle traced by minute hand in 35 min = 1 Monday
2 Tuesday
3 Wednesday
4 Thursday
Required angle = 5 Friday
6 Saturday
Calendar (Important points) Formats of the questions
1. Odd days The number of days more than the Example 1: What was the day of the week on
complete weeks for a given period called odd 26th June, 1816?
days. (a) Saturday (b) Tuesday
2. Ordinary Year An ordinary year has 365 days (c) Wednesday (d) Monday
and is not a leap year.
Explanation: Option (c) is correct because 26th june
3. Leap Year 1816 = (1815yr + Period from 1.1.1816 to
(a) Any year (except a century) which is divis- 26.6.1816) counting of odd days. Number of odd
ible by 4 is a leap year days in 1600 yr = 0
(b) However, every 4th century is a leap year, ie, Number of odd days in 200 yr = 3
a century is a leap year when it is divisible by 15yr = 3leap years + 12 ordinary years =
400.
(3 2 + 12  1) odd days = 18 odd day
(c) A leap year has 366 days.
= (2 weeks + 4 days) = 4 odd days
Example of Leap year: 1924, 1908, 1944, 2008,
1815 yr have = (0 + 3 + 4) = 7 odd days = 0 odd
2012 etc. are all leap years.
day
4. Counting of odd days
Jan Feb March April May June
(a) 1 ordinary year = 365 days = (52 weeks +
(31 + 29 + 31 + 30 + 31 + 26) = 178 days.
1day) 1 ordinary year = 1 odd day..
178 days = (25 weeks + 3 days) = 3 odd days.
(b) 1 leap year = 366 days = (52 weeks + 2 days)
Total number of odd days = (0 + 3) = 3 odd
1 leap year = 2 odd day
days
(c) 100 yr = 76 ordinary years + 24 leap years =
Hence, the required day is wednesday.
(76 1 + 24 2) odd days = 124 odd days =
(17 weeks + 5 days) = 5 odd days Example 2: If fourth day after January 6 is a
Saturday,  which  day  of  the  week  was  on
(d) 200 yr = (5 2) = 10 odd days = (1 week + 3
December 1 of the previous year?
days) = 3 odd day
(a) Saturday (b) Friday
(e) 300 yr = (5 3) = 15 odd days = (2 weeks + 1
day) = 1 odd days (c) Sunday (d) Thursday
(f) 400 yr = (5 4 + 1) = 21 odd days = (3 weeks Explanation: Option (c) is correct because January
+ 0 day) = 0 odd day 10 is on Saturday.
Similarly, 800 yr , 1200 yr, 1600 yr, 2000yr, have Days between December 1 to January 10 = (30
0 odd days. + 10) = 40
5. Day of the week with respect to the number of December 1 was Sunday..
odd days.
SSC
Exercise
1. A clock strikes 12 taking 48 s. In order to (c) Both (a) and (b)(d) Data inadequate
strike 3 at the same rate, the time taken is 8. A  watch  which  gains  uniformly  is  2  min
(a) 24 s (b) 12 s
slow at 10 am on Wednesday and is  min
(c) s (d) s
fast at 1 pm the following day. When was it
2. At what time between 9 and 10 O’clock will correct?
the hands of a clock be in the same straight (a) 10 : 30 pm Wednesday
line but not together? (b) 9 : 15 pm Wednesday
(c) 10 : 45 pm Wednesday
(a) min past 9
(d) 10: 00 pm Wednesday
9. On what dates of March, 2003 did Tuesday
(b) min past 9 fall?
(a) 5th, 12th, 19th, 26th
(c) min past 9 (b) 4th, 11th, 18th, 25th
(c) 3rd, 10th, 17th, 24th
(d) Data indaequate
(d) min past 9
10. What  will  be  the  day  on  8th  September,
3. A  cl ock  is  started  at   6  O’ cl ock  in  t he 1998?
morning. Through  how many  degrees will (a) Monday (b) Tuesday
the hour hand rotate when the clock shows (c) Wednesday (d) Friday
11 O’clock in the morning? 11. What  will  be  the  day  of  the  week  on  15th
(a) 140° (b) 180° July, 1996?
(c) 150° (d) 160° (a) Thursday (b) Monday
4. At 6:15, the hour hand and the minute hand (c) Wednesday (d) Sunday
of a clock form an anlge of 12. The calendar for the year 2002 is the same
(a) 90° (b) 91.5° as for the year.
(c) 97.5° (d) 94.5° (a) 2006 (b) 2008
5. A  clock  is  started  at  5  pm.  Through  how (c) 2009 (d) 2010
many  degrees  will  the  hour  hand  rotate 13. January 1, 2001 was a Monday. What day
when the clock shows quarter past 9? of the week lies on January 1, 2002?
(a) 128.5° (b) 142.5° (a) Saturday (b) Tuesday
(c) 127.5° (d) 141.5° (c) Monday (d) Sunday
6. At what time between 1 and 2 O’clock are 14. If 9th March of 1995 is a Saturday, then the
the hands of a clock together? 9th March of 1996 is a
(a) Wednesday (b) Tuesday
(a) min past 1 (b) min past 1
(c) Sunday (d) Monday
15. The last day of February, 2006 was a
(c) min past 1 (d) min past 1 (a) Monday (b) Tuesday
(c) Wednesday (d) Thurday
7. At what time between 5 and 6 O’clock will
the hands of a clock be at right angle? 16. Which will be the first leap year after 2096?
(a) 2100 (b) 2104
(a) min past 5(b) min past 5 (c) 2102 (d) 2108
SSC
17. If the seventh day of a month is three days min  before his steno who  in  turn was  late
earlier than Friday, what day will it be on by 30 min to the meeting. At what time was
the nineteenth day of the month? the meeting supposed to start?
(a) Sunday (b) Monday (a) 12 : 00 (b) 12 : 15
(c) Wednesday (d) Friday (c) 12 : 10 (d) 12 : 20
18. If a month in an year starts with Monday, 27. The  priest  told  the  devotee,  “The  temple
then  the  date  of  the  fourth  day  after  the bell is rung at regular intervals of 45 min.
second Saturday in that month, will be The last bell was rung 5 min ago. The next
(a) 16 (b) 17 bell is due to be rung at 7 : 45 am”. At what
(c) 18 (d) 19 time did the priest give the information to
the devotee?
19. If 9th May 2008 was Friday, then what day
was February 14th of the same year? (a) 7 : 00 am (b) 7 : 05 am
(a) Thursday (b) Monday (c) 7 : 40 am (d) 6 : 55 am
(c) Friday (d) Wednesday 28. A clock is set right at 10 am. The clock gains
10 min in a day. What will be the true time
20. The time on the watch is 9 : 15 and the hour
when  the  watch  indicates  3  pm  the  next
hand points towards West. The direction of
day?
the minutes hand is
(a) 12 min past 2 pm
(a) North (b) South
(b) 45 min past 2 pm
(c) East (d) West
(c) 48 min past 2 pm
21. If the first day of June is a Saturday, then
the date on which the last Saturday of that (d) 30 min past 2 pm
month falls is 29. A  clock  strikes  5  taking  8  s.  In  order  to
(a) 22 (b) 28 strike 9 at the same rate, the time taken is
(c) 29 (d) 30 (a) 16 s (b) 12 s
22. If a wall clock shows 9 h 30 min. What time (c) 14 s (d) 18 s
will the clock show in the mirror? 30. A clock is set to show the correct time at 7
(a) 8 h 45 min (b) 8 h 50 min am Monday. The clock loses 15 min in 24 h.
What will be the true time when the clock
(c) 9 h 50 min (d) 9 h 30 min
indicates 6 am. on the following Friday?
23. How much does a watch loes per day, if its
(a) 6 : 15 am Friday
hands coincide every 64 min?
(b) 6 : 30 am Friday
(a) 96 min (b) 90 min
(c) 7 : 00 am Friday
(c) min (d) min (d) 7 : 15 am Friday
31. Anil reached a place on Friday. He came to know
24. In  a  clock,  what  is  the  angle  between  the that he was three days earlier than the scheduled
two hands at 5 h 10 min? day. If he had reached there on the following Sun-
(a) 60° (b) 95° day, how many days late/early he would have
(c) 90° (d) 100° been?
25. The  angle  between  the  hands  of  a  clock (a) One days earlier (b) One day late
w hen  the  time  shown  is  12  min  past  5 (c) Two days late (d) Two days earlier
O’clock is 32. If day before yesterday was Friday, what will be
(a) 78° (b) 80° the third day after the day - after - tomorrow?
(c) 84° (d) 86° (a) Thursday (b) Friday
26. The director came to the office to attend a (c) Saturday (d) Sunday
meeting at 15 min past 12 and he came 25 33. If the day before yesterday was Thursday, when
SSC
will Sunday be? (a) Tuesday (b) Wednesday
(a) Tomoorow (b) Day after tomorrow (c) Monday (d) Sunday
(c) Today (d) Two days after today 41. At what time are the hands of clocks together be-
34. If the day before yesterday was Sunday, what day tween 6 and 7?
will it be three days after the day after tomorrow? 8
(a) 32 minutes past 6
(a) Sunday (b) Monday 11
(c) Wednesday (d) Saturday 8
(b) 34 minutes past 6
35. If the day after tomorrow is Sunday, what day was 11
tomorrow's day before yesterday?
8
(a) Friday (b) Thursday (c) 30 minutes past 6
11
(c) Monday (d) Tuesday
5
36. Mrs. Susheela celebrated her wedding anniversary (d) 32 minutes past 6
7
on Tuesday, 30th Septermber 1997. When will she
celebrate her next wedding anniversary on the 42. In the year 1996, the Republic day was celebrated
same day? on Friday. On which day was the Independence
day celebrated in the year 2000?
(a) 30 September 2003
(a) Tuesday (b) Monday
(b) 30 September 2004
(c) Friday (d) Saturday
(c) 30 September 2002
43. In Ravi's clook shop, two clocks were brough for
(d) 30 October 2002
repairs. One clock has the cuckoon coming out
37. A clock gains five minutes every hour. What will every sixteen minutes, while the other one has the
be the angle traversed by the second hand in one cuckoon coming out every eighteen minutes. Both
minute? cuckoos come out at 12.00 noonl When will they
(a) 360° (b) 360.5° both come out to gether again?
(c) 390° (d) 380° (a) 2 : 06 PM (b) 2 : 08 PM
38. If John celebrated his victory day on Tuesday, 5th (c) 2 : 24 PM (d) 2 : 32 PM
January 1965, when will he celebrate his next vic- 44. Hari remembers that his father's birthday is be-
tory day on the same day? tween 13th and 16th June, whereas his sister re-
(a) 5th January 1970 members that their father's birthday is between
(b) 5th January 1971 14th and 18th of June. One which day is their
(c) 5th January 1973 father's birthday, which both agree?
(d) 5th January 1974 (a) 14th June (b) 15th June
39. After 9'0 clock at what time between 9 p.m. and (c) 16th June (d) 17th June
10 p.m. will the hour and minute hands of a clock 45. 5th of a month falls two days after Monday. What
point in opposite direction? day of the month will precede 19th of it?
(a) 15 minutes past 9 (a) Wednesday (b) Thursday
(b) 16 minutes past 9 (c) Tuesday (d) Monday
4 46. Kamala would like to complete all her home work
(c) 16 11 minutes past 9 before 10..00 p.m. in order to watch an important
TV programme. She has 40 minutes assignment
1
(d) 17 11 minutes past 9 in each of her five prepared subjects. What is the
latest time at which she can start and still com-
40. Suresh was born on 4th October 1999. Shashikanth plete her home-work in the time for the
was bron 6 days before Suresh. The Independence programme?
Day of that year fell on Sunday. Which day was (a) 6 : 40 p.m. (b) 6 : 30 p.m.
Shashikanth born?
(c) 7 : 10 p.m. (d) 7 : 20 p.m.
SSC
47. Reaching the place of meeting on Tuesday 15 min- (c) Sunday (d) Wednesday
utes before 08 : 30 hours I found myself half an 56. If Friday falls on 15th of September 2000, what
hour earlier than the man who was 40 minutes late. will be the day of 15th of September 2001?
What was the scheduled time of the meeting? (a) Friday (b) Saturday
(a) 08 : 00 hours (b) 08 : 05 hours (c) Thursday (d) Sunday
(c) 08 : 15 hours (d) 08 : 45 hours 57. If three days before yesterday was Wednesday,
48. The Independence day in 1988 was celebrated on what will be two days after tomorrow?
a Wednesday. on what day was it celebrated in the (a) Wednesday (b) Monday
year 1989?
(c) Friday (d) Tuesday
(a) Monday (b) Tuesday
58. Find out the day from the problem given. If the
(c) Friday (d) Thursday 1st of Novermber falls on Monday, what day will
49. If the 23rd of a month is a Sunday, what day it the 25th of Novermber be?
would have been two weeks and four more days (a) Tuesday (b) Thursday
earlier?
(c) Wednesday (d) Friday
(a) Monday (b) Tuesday
59. If Friday is the first day of a non-leap year, what
(c) Wednesday (d) Thursday day would the last day of the year be?
50. If the day after tomorrow is Tuesday what day will (a) Friday (b) Sunday
two days after the day after tomorrow be?
(c) Monday (d) Tuesday
(a) Monday (b) Wednesday
60. If day after tomorrow is Saturday what day was
(c) Saturday (d) Thursday three days before yesterday?
51. Tell the number of days from 26 January 2006 to (a) Thursday (b) Monday
23 September 2006 (including both dates).
(c) Saturday (d) Sunday
(a) 214 (b) 241
61. Reaching the place of meeting 20 minutes before
(c) 249 (d) 251 8 : 50 hrs Satish found himself thirty minutes ear-
52. If two days before yesterday was Firday, what day lier than the man who came 40 minutes late. What
will be day after tomorrow? was the schedule time of the meeting?
(a) Monday (b) Sunday (a) 08 : 20 (b) 08 : 10
(c) Saturday (d) Wednesday (c) 08 : 05 (d) 08 : 00
53. Today is Friday. On the last monday the date was 62. Saturday comes after 3 days on 4th of a month.
29th December, 1975. The date today is Find out the day on 27th of that month.
(a) 25 December, 1975 (a) Monday (b) Thursday
(b) 3rd January, 1976 (c) Friday (d) Saturday
(c) 2nd January, 1976 63. If the 2nd of a month falls on Sunday, what day
(d) 2nd January, 1975 will the 31st of that month be?
54. A clock goes fast by one minute during the first (a) Tuesday (b) Saturday
hour, by two minutes at the end of the second hour, (c) Friday (d) Monday
by 4 minutes at the end of 3rd hour, by eight min- 64. Day after tomorrow is Kiran's birthday. On the
utes by the end of 4th hour, and so on. At the end same day next week falls 'Shivratri'. Today is
of which hour, will it be fast by just over sixty Monday. What will be the day after 'Shivratri"?
minutes?
(a) Wednesday (b) Thursday
(a) Fifth (b) Sixth
(c) Friday (d) Saturday
(c) Seventh (d) Eighth
65. If the day before yesterday was Thursday, when
55. If day before yesterday was Thursday, what day will Sunday be?
will be four days after tomorrow?
(a) Day after tomorrow
(a) Monday (b) Thursday
SSC
(b) Today 68. If the day before yesterday was Thursday, when
(c) Tomorrow will Sunday be?
(d) Two days after today (a) Today
66. If the day after tomorrow is Friday, what day (b) Two days after today
willthird day after the tomorrow be? (c) Tomorrow
(a) Saturday (b) Monday (d) Day after Tomorrow
(c) Sunday (d) Friday 69. If day before yestereday was Wednesday, when
67. Mohini went to movies nine days ago. She goes will Sunday be?
to the movies only on Thursday. What day of the (a) 3 days after today(b) Tomorrow
week is today? (c) Today (d) Dady after tomorrow
(a) Saturday (b) Thursday 70. If the day before yesterday was Friday, what day
(c) Sunday (d) Tuesday will two days after the day after tomorrow be?
(a) Saturday (b) Thursday
(c) Firday (d) Sunday

Answers
1. c 2. a 3. c 4. c 5. c 6. a
7. c 8. d 9. b 10. c 11. b 12. b
13. b 14. d 15. b 16. b 17. a 18. b
19. a 20. c 21. c 22. c 23. c 24. c
25. c 26. c 27. d 28. c 29. a 30. a
31. a 32. b 33. a 34. a 35. b 36. a
37. b 38. b 39. c 40. b 41. a 42. a
43. c 44. b 45. a 46. a 47. b 48. d
49. b 50. d 51. b 52. b 53. c 54. b
55. a 56. b 57. d 58. b 59. d 60. a
61. b 62. b 63. a 64. a 65. b 66. d
67. a 68. d 69. b 70. c
SSC
10. NUMBER MATRIX
What is number matrix?
Ex. 3 Replace ?
This is arrangement of digits/numbers in side or out-
side of geometrical figure according to the certain pat-
tern. In this arrangement one or more number is miss-
ing . The candidate is required to find out the pattern
and then trace the missing number. The formats of the
questions will give better idea about number matrix.
Formats of the questions
Ex. 1 Find the missing number? (a) 25 (b) 125
(c) 156 (d) 625
Explanation:   Option (d) is correct.
Clearly, (2 + 3) 2 = 25, (15 + 6) 2 = 441, (10 + 7) 2 =
289
So missing number = (12 + 13) 2 = 625
(a) 42 (b) 46 Ex. 4 What is the missing number?
(c) 48 (d) 50
Explanation:   Option (b) is correct because we have:
(8 6) - (2 6) = 36, (7 8) - (2 5) = 46
Missing number = (7 10) - (4 6) = 46.
Ex. 2 Replace the questionmark.

(a) 327 (b) 386


(c) 438 (d) 469
Explanation:   Option (d) is correct.
(a) 7 (b) 109 we have: 4 2 - 1 = 7, 7 2 + 1 = 15, 15 2-
(c) 49 (d) 129 1 = 29, 29 2 + 1 = 59, 59 2 - 1 = 117, 117
Explanation:   Option (b) is correct because we have: 2 + 1 = 235.
(16 - 6) 2 + (5 - 2) 2 = 10 2 + 3 2 = 109; (22 - 15) 2 + So missing number = 235 2 - 1 = 469
(21 - 19) 2 = 7 2 + 2 2 = 53;
So missing number = (16 - 6) 2 + (51 - 48) 2 = 10 2
+ 3 2 = 109;

Exercise
Directions  (Q.  no.  1   -  4 0 ):  Find  the  missing
number. 2.

1.

(a) 2 (b) 3
(c) 6 (d) 12
(a) 135 (b) 221
(c) 236 (d) 170
SSC
(a) 1 (b) 26
3.
(c) 39 (d) 45

10.

(a) 80.2 (b) 75


(c) 90 (d) 74.25
4.

(a) 1 (b) 731


(c) 1625 (d) 2031

(a) 4 (b) 2 11.


(c) 1 (d) 5

5.

(a) 21 (b) 25
(a) 4 (b) 305 (c) 35 (d) 45
(c) 216 (d) 729 12.

6.

(a) 12 (b) 21
(c) 24 (d) 35
(a) 0 (b) 8
(c) 125 (d) 216 13.
7.

(a) 5 (b) 6
(a) 13 (b) 14 (c) 8 (d) 9
(c) 20 (d) 21
14.
8.

(a) 5 (b) 12 (a) 25 (b) 37


(c) 13 (d) 26 (c) 41 (d) 47
9. 15.
SSC
(a) 2 (b) 6
23.
(c) 8 (d) 64
16.

(a) 27 (b) 35
(c) 54 (d) 64 (a) 4 (b) 8
17. (c) 12 (d) 16

24.

(a) 26 (b) 32
(c) 36 (d) 117
18.

(a) 16 (b) 21
(c) 61 (d) 81
19. (a) 3 (b) 4
(c) 8 (d) 12
25.

(a) 12 (b) 14
(c) 15 (d) 18
20.
(a) 1441 (b) 3529
(c) 8281 (d) 9361
26.

(a) 35 (b) 37
(c) 45 (d) 46
21.

(a) 10 (b) 11
(c) 12 (d) 13
(a) 232 (b) 268 27.
(c) 298 (d) 350
22.

(a) 35 (b) 48
(c) 72 (d) 120 (a) 9 (b) 18
(c) 24 (d) 27
SSC
28. 35.

(a) 6 (b) 8
(c) 9 (d) 12
(a) 10 (b) 11 36.
(c) 12 (d) 13
29.

(a) 678 (b) 769


(c) 824 (d) 937
(a) 49 (b) 76
37.
(c) 89 (d) 94
30.

(a) 2 (b) 3
(c) 4 (d) 5
(a) 6 (b) 9
(c) 12 (d) 18
31. 38.

(a) 20 (b) 25 (a) 127 (b) 142


(c) 50 (d) 75 (c) 158 (d) 198
32. 39.

(a) 125 (b) 215


(c) 251 (d) 512
33.

(a) 535 (b) 577


(c) 755 (d) 775
40.
(a) 260 (b) 269
(c) 324 (d) 429
34.
(a) 1 (b) 2
(c) 3 (d) 4

(a) 35 (b) 37
(c) 45 (d) 47
SSC
Answers

1. d 2. d 3. d 4. c 5. c 6. d
7. b 8. a 9. c 10. d 11. b 12. b
13. d 14. c 15. c 16. b 17. c 18. a
19. a 20. c 21. a 22. b 23. b 24. b
25. c 26. c 27. a 28. b 29. a 30. b
31. c 32. a 33. c 34. b 35. a 36. a
37. c 38. b 39. b 40. b
SSC
11. RANKING
What is ranking test? (i) 6 students who have ranks higher than
Under this segment, generally the rank of a person both Aakash;
from the top/left and from the bottom/right are men- (ii) Aakash; and
tioned and the total number of person is asked. How- (iii) 25 students who have ranks lower than
ever, sometimes this question is put in the form of a Aakash, i.e., (6 + 1 + 25) = 32 students.
puzzle of interchanging seats by two persons. Example 2: Manish is fifteenth from the right end
Formats of the questions in a row of 40 boys. What is his position from the
Example 1: Aakash ranks seventh from the top and left end?
twenty-sixth from the bottom in a class. How many (a) 24th (b) 25th
students are there in the class? (c) 26th (d) 27th
(a) 31 (b) 32 Explanation:  Option (c) is correct because
(c) 33 (d) 34 number of boys towards the left of Manish =
Explanation: Option (b) is correct because the (40 - 15) = 25
whole class consists of: So, Manish is 26th from the left end.

Exercise

1. If you are eleventh in a queue starting either end, has two persons taller and two persons shorter than
how many are there in the queue? him?
(a) Eleven (b) Twenty (a) R (b) S
(c) Twenty one (d) Twenty two (c) K (d) A
2. In a row of boys, if A who is 10th from the left 6. Of the five members of a panel sitting in a row, A
and B who is 9th from the right interchange their is to the left of B, but on the right of C, D is on the
positions, A becomes 15th from left. How many right of B but is on the left of E. Find the member
boys are there in the row? who is sitting in the middle.
(a) 23 (b) 27 (a) B (b) D
(c) 28 (d) 31 (c) A (d) C
3. A is older than B but younger than C, D is younger 7. A, B, C, D and E are sitting on a bench. A is sit-
than E but older than A. If C is younger than D, ting next to B, C is sitting next to D, D is not sit-
who is the oldest of all? ting with E who is on the left end of the bench. C
(a) A (b) C is on the second position from the right. A is on
(c) D (d) E the right of B and E. A and C are sitting together.
In which position is A sitting?
4. Heavier coins are costlier. Ram's coin is heavier
than Mohan's and costlier than Ramesh's. Naresh's (a) Between B and D(b) Between B and C
coin is costiler than Ram's but lighter than (c) Between E and D (d) Between C and E
Yogesh's. Ramesh's coin the costlier than Mohan's. 8. A, P, R, X, S and Z are sitting in a row. S and Z are
So who is the owner of the costliest coin? in the centre, and A and P are at the ends. R is
(a) Ram (b) Ramesh sitting on the left of A. Then who is on the right of
(c) Yogesh (d) Naresh P?
5. There are five friends–S, K, M, A, R, S is shorter (a) A (b) X
than K, but taller than R, M is the tallest. A is a (c) S (d) Z
little shorter than K and little taller than S. Who 9. A is richer than B
SSC
C is richer than A (c) Kenny (d) Bobby
D is richer than C 16. Out the six members panel sitting in a row 'U' is to
E is the richest of all the left of 'V' but on the right of 'W' who is to the
If they are made to sit in the above degree of rich- right of 'X' and 'Y' is at the right oa 'Z' who is to
ness who will be in the middle position (central the left to 'X'. Find the members sitting right in
position) ? the middle.
(a) P (b) Q (a) ZY (b) XW
(c) R (d) T (c) UV (d) WV
10. P, Q, R and T answered an Examination. In the 17. Of the six members of a panel sitting in a row X is
results Q was immediately followed by 'P' but no to the left of Q but on the right of P, Y is on the
one was there after 'P', 'R' was ahead of 'Q' but right of Q but is on the left of Z, Z is to the left of
could not score as much as 'T'. Who scored the R. Find the members who are at the extreme?
second hightest? (a) QZ (b) XZ
(a) A (b) B (c) PR (d) QY
(c) C (d) D 18. Six girl are standing in a circle facing to the cen-
11. A, B, C, D, E, F & G are sitting in line facing the tre. Bindu is to the left of Viji. Rekha is in be-
East. C is immediate right of D. B is at an extreme tween Bindu and Mumtaz. Jessa is in between Viji
end and has E as his neigbour. G is between E and and Nirmala. Who is to the left of Mumtaz?
F. D is sitting third from the South end. Who are (a) Rekha (b) Nirmala
the persons sitting at the extreme ends? (c) Viji (d) Bindu
(a) A & E (b) A & B 19. 'Suma' is shorter than 'Uma', 'Neha' is taller then
(c) F & B (d) C & D 'Suma', 'Sudha' is taller than 'Uma' but shorter than
12. Five boys are sitting in a row, a is on the right of 'Hema', 'Uma' is taller than 'Neha'. Who is the tall-
B, E is on the left of B, but to the right of C. If A is est among them?
on the left of D, who is sitting in the middle? (a) Hema (b) Uma
(a) E (b) B (c) Sudha (d) Neha
(c) A (d) C 20. In a row of 16 girls, when Hema was shifted by
13. While the group photo of a family was taken, the two places towards the left she become 7th from
father was found to be sitting to the left of the son the left end. What was her earlier position from
and right to the grandfather, Mother was sitting to the right end?
the right of her daughter but left to grandfather. (a) 7th (b) 8th
Who is occupying the central place? (c) 9th (d) 10th
(a) Son (b) Grandfather 21. If
(c) Father (d) Mother A. Sunitha is taller than Anitha.
14. A family went out for a walk. Daughter walked B. Reena is taller than Chitra but shorter than
before the father. Son was was walking behind Banu.
the mother and ahead of father. Who walked last? C. Anitha is shorter than Chitra.
(a) Son (b) Father D. Chitra is taller than Sunitha.
(c) Mother (d) Daugther then who is the shortest?
15. Six friends are sitting in a cricle and playing cards. (a) Sunitha (b) Anitha
Kenny is to the left of Danny. Michael is in be- (c) Reena (d) Banu
tween Bobby and Johnny. Roger is in between
22. In a row of children. Ravi is fourth from right and
Kenny and Bobby. Who is sitting to the right of
Sham is second from left. When they interchange
Michael?
positions Ravi is ninth from right. What will be
(a) Danny (b) Johnny Sham's position from left?
SSC
(a) Fifth (b) Sixith (a) E (b) A
(c) Seventh (d) Eighth (c) D (d) B
23. In a row of boys, Srinath is 7th from the left and 30. Six persons A, B, C, D, E, F sit in 2 rows, 3 in
Venkat is 12th from the right. If they interchange each. If E is not at any end, D is second to left of
their positions, Srinath becomes 22nd from the left. F, C is neighbour of E and is sitting diagonally
How many boys are there is the row? opposite to D and B is neighbour of F, who will
(a) 19 (b) 31 be opposite to B?
(c) 33 (d) 34 (a) A (b) E
24. In a classroom, there are 5 rows, and 5 children A, (c) C (d) D
B, C, D and E are seated one behind the other in 5 31. B is twice as old as A but twice younger than F. C
separate rows as follows: is half the age of A but is twice older than D. Who
A is sitting behined C, but in front of B. is the second oldest?
C is sitting behind E. D is sitting in front of E. (a) B (b) F
The order in which they are sitting from the first (c) D (d) C
row to the last is 32. Ramesh ranks 13th in a class of 33 students. There
(a) DECAB (b) BACED are 5 students. There are 5 students below Suresh
(c) ACBO (d) ABEDC rankwise. How many students are there between
Ramesh and Suresh?
25. A group of friends are sitting in an arrangement
one each at the corner of an octagon. All are fac- (a) 12 (b) 14
ing the centre. Mahima is sitting diagonally oppo- (c) 15 (d) 16
site Rama, who is on Sushma and opposite Girdhar 33. In a row of trees, a tree is 7th from left end and
who is on Chandra's left. Saviri is not on Mahima's 14th from the right end. How many trees are there
right bu opposite Shalini. Who is on Shalini's right? in the row?
(a) Ravi (b) Mahima (a) 18 (b) 19
(c) Girdhar (d) Rama (c) 20 (d) 21
26. Suresh is 7 ranks ahead of Ashok in the class of 34. Five girls M, N, O, P and Q are standing in a row.
39 students. If Ashok's ranks is 17th from the last, P is on the right of Q. N is on the left of Q. but is
what is Suresh's rank from the start? on the right of M. P is on the left of O. Who is
(a) 16th (b) 23th standing on the extreme right ?
(c) 24th (d) 15th (a) Q (b) N
27. Sudheesh ranks seventh from the top and 28th from (c) O (d) P
the bottom. How many students are there in the 35. Sita is elder than Swapna. Layanya is elder than
class? Swapna but younger than Sita. Suvarna is younger
(a) 34 (b) 35 than both Hair and Swapna, Swapna is elder than
(c) 28 (d) 21 Hari. Who is the youngest?
28. Five people are sitting in a row facing you. Y is at (a) Sita (b) Lavanya
the left of X, W is sitting at the right of Z. V is (c) Suvarna (d) Hari
sitting at the right of X and W is sitting at the left 36. In a row of girls, Kamal is 9th from the left and
of Y. If Z is sitting at one end who is in the middle? Veena is 16th from the right. If they interchage
(a) V (b) X their positions, Kamla becomes 25th from the left.
(c) Y (d) Z How many girls are there in the row?
29. A, B, C, D and E are 5 schools facing towards (a) 34 (b) 36
north. A is in the middle of E and B. E is to the (c) 40 (d) 41
right of D. If C and D are at two ends, which school 37. In a class Rajan got the 11th rank and he was 31st
is on the left side of C? from the bottom of the list of boys passed. Three
SSC
boys did not take the examination and one falled. ing must meet the following conditions: N is
What is the total strength of the class? shown before L, J is shown third, Q is shown fifth.
(a) 32 (b) 42 If N is shown immediately after P, then P could be
(c) 45 (d) 46 shown
38. Four children, Akram, Bopsi, Priya and Tulsi are (a) Third (b) Fourth
on a ladder. Akram is further up the ladder than (c) Fifth (d) Sixth
Bopsi, Bopsi is in between Akram and Priya. If 45. Five students A, B, C, D, E are sitting in a circle
Tulsi is still further than Akram, who is the sec- facing each oether. If E is between A and D and A
ond persons from the bottom? is to the right of B, then who is to the left of B ?
(a) Tulsi (b) Akram (a) A (b) B
(c) Priya (d) Bopsi (c) C (d) E
39. Five persons A, B, C, D and E are sitting in a row 46. A runs faster than B but not as fast as C who is
facing you such that D is on the left of C and B is slower than D. Who is the fastest runner?
on the right of E. A is on the right of C and B is on (a) D (b) C
the left of D. If E occupies a corner postion. then (c) A (d) B
who is sitting in the centre?
47. There are three girls G1, G2, B3 and three boys B1,
(a) A (b) B B2, B3 in a school talks. The restriction is no two
(c) C (d) D girls should speak together. If in an arrangement
40. Shailendra is shorter than Keshav but taller than B1 speaks first what is the position of B2 and G2?
Rakesh. Madhav is the tallest. Ashish is a little (a) 2nd and 3rd (b) 3rd and 4th
shorter than Keshav and little taller than (c) 4th and 5th (d) 5th and 6th
Shailendra. If they stand in the order of increas-
48. Five students are sitting in a row, S is on the right
ing heights, who will be the second?
of L, P is on the left of L but is on the right of K, S
(a) Ashish (b) Shailendra is on the left of Q. The first student in the row
(c) Rakesh (d) Madhav from the left is
41. In a row A is at the 11th position from the left and (a) K (b) L
B is 10th from the right. If they interchanged po- (c) P (d) S
sitions A becomes 18th from the left. How many
49. Four friends were playing a game of cards sitting
persons are there in that row?
in a circle. Shankar was right to Ram and Gopal
(a) 28 (b) 29 was left to Arvind. Which one of the following
(c) 27 (d) 31 pairs were the partners?
42. Six persons M, N, O, P, Q and R are sitting in two (a) Ram and Arvind (b) Gopal and Shankar
rows, three in each. Q is not at the end of any row, (c) Ram and Shankar(d) Gopal and Ram
P is second to the left of R. O is the neighbour of
50. 4 girls and 3 boys (G1, G2, G3, G4) and (B1, B2,
Q and is sitting diagonally opposite to P. N is the
B3). are to sit for a dinner such that no two boys
neigonally opposite to P. N is the neighbour of R.
should sit together nor two girls. what is the posi-
On the basis of this information who is facing N?
tion of B2 and G3?
(a) R (b) Q
(a) 3rd and 4th (b) 4th and 5th
(c) P (d) M
(c) 5th and 6th (d) 2nd and 3rd
43. Sita is older than Renu. Geeta is younger than
51. Four girls and four boys are sitting in a square
Renu. Priya is older than Sita. Who is the eldest
facing the centre. They are sitting one each at the
of them?
corners and one each at the mid-points of the sides
(a) Priya (b) Sita of the square. Madhu is sitting diagonally oppo-
(c) Renu (d) Geeta site to Usha who is to Geetha's right. Roy is next
44. At a certain flim festival, eight films will be shown Geetha and opposite to Gopi who is on Bose's left.
J, K, L, M, N, P, Q and R. The order of the show- Suma is not on Madhu's right but opposite to
SSC
Prema. Who is opposite to Bose? 59. In a sports competition, the position of one of the
(a) Geetha (b) Prema players is 8th from the top and 84th from the bot-
(c) Suma (d) Madhu tom. What is the total number of the competitiors?
52. Ub a circular arrangement of 3 boys (B1, B2, and (a) 93 (b) 91
B3) and 3 girls (G1, G2 and G3) sitting for a dinner (c) 89 (d) 88
successively, what will be the position of G3 and 60. Mehrunnisa is eleventh from either end of a row
B3, if no two girls sit together? comprising of girls. How many girls are there in
(a) 4th and 5th (b) 5th and 6th the row?
(c) 3rd and 4th (d) 3rd and 6th (a) Nineteen (b) Twenty
53. In a group, P is smarter than R, Q is duller than T. (c) Twenty-one (d) Twenty-two
P is smarter than T. Who is the smartest? 61. Reema's height is 5'2". Anita is taller than Reema
(a) P (b) R but she is not taller than Pinky. is shorter than her
(c) Q (d) T cousin Rani but she is not shorter than Reema.
Who is the tallest in the group?
54. On the seven members panel sitting in a row X is
to the left of Y, but on the right of O. P is on the (a) Anita (b) Rani
right of Y but is on the left of N and M is on the (c) Pinky (d) Reema
left of Z, who is to the left of O. Find the member 62. Age of Amit is equal to that of Summit as they are
sitting right in the middle. twins. Richa is younger than Summit, Richa is
(a) Z (b) P younger than Jyotsna but elder than Saurabh. Sum-
(c) X (d) O mit is younger than Jyotsna. Who is the eldest of
all?
55. Six boys are sitting in a row. Jose and Manu are
sitting adjacent to Raju. Uday has Gopi and Ram (a) Amit (b) Jyotsna
as his adjacents. Gopi is not next to either Jose or (c) Richa (d) Saurabh
Manu. Ram is not sitting next to Manu. Who are/ 63. There are 17 girls in a row. The position of the
is sitting adjacent to Jose? girls at the middle is 9th from the beginning. What
(a) Raju and Uday (b) Raju and Manu will be the position of the girl at the middle from
(c) Raju and Ram (d) Only Raju the end?
56. Raman is a student of 10th class. In his class, his (a) 7th (b) 9th
position is 16th from the top and 49th from the (c) 8th (d) 10th
bottom. What is the total number of the students 64. In a panel of five members sitting in a circle fac-
in the class? ing inward, A in in the middle of B and E, D is to
(a) 64 (b) 65 the right of E and left of C. Find the position of B
(c) 66 (d) 63 in the panel.
57. Five boys A, B, C, D, E are in a row. A is on the (a) Immediate left of A and E
right of B, E is on the left of B but on the right of (b) Immediate left of D
C and A is on the left of D. Who is second from (c) Immediate right of A
the left? (d) Immediate right of C
(a) D (b) A 65. Rama ranks sixteenth from the top and fifteenth
(c) E (d) B from the bottom in a certain examination. How
58. In a row of 15 children, when Raju was shifted many students are there in the class?
three places towards right, he becomes 8th from (a) 30 (b) 31
the right end. What was his earlier position from (c) 32 (d) 33
the left end of the row? 66. If Seshan is taller than Ammu but shorter than Raju
(a) 14 (b) 5 and Ammu is just as tall as Nitin but taller thank
(c) 6 (d) 12 Kishore, then Nitin is
SSC
(a) just as tall as Seshan (c) 26th (d) 28th
(b) shorter than Raju 74. Five different coloured buses are standing in a row
(c) taller than Raju facing South. Black coloured bus is standing in
(d) shorter than Seshan the immediate right of Read. Green colour is be-
tween Blue and Yellow. Yellow colour is between
67. Gopal is elder to Mohan, but younger to Ram.
Black and Green. Which coloured bus is standing
Mohan is elder to Sohan, but younger to Ram. Who
in the middle?
is the eldest?
(a) Yellow (b) Blue
(a) Gopal (b) Mohan
(c) Black (d) Green
(c) Ram (d) Sohan
75. In a row of children. Harish is eleventh from the
68. On the seven members sanel sitting in a row 'A' is
left and Mangesh is seventeenth from the right.
to the immediate left of 'B', but on the immediate
When they exchange their places, Harish will be
right of 'D',. 'Q' is on the right of 'A' but is on the
thirteenth from the left. Which of the following
left of 'S', and 'T' is on the left of 'V' who is to the
will be the new position of Mangesh from the
left of 'D'. Find the member sitting right in the
right?
middle.
(a) Eleventh (b) Twenty - first
(a) V (b) A
(c) Nineteenth (d) Twenty - ninth
(c) Q (d) D
Directions (76-77):  A, B, C, D and E are five boys
69. Jill has more money than Mani but less than Babu.
sitting in a circle C is sitting immediately to the
If the amounts held by Jill, Mani and Babu are x,
left of E. A is sitting between D and E.
y and z, respectively. which of the following is
true? 76. Who is sitting to the immediate left hand side of
C?
(a) z < x < y (b) x < z < y
(a) E (b) A
(c) y < x < z (d) x < y < z
(c) B (d) D
70. A scores more runs than B but less than C. D scores
more than B but but les than A. Who is the lowest 77. Who is sitting between B and A?
scorer? (a) C (b) E
(a) A (b) B (c) D (d) None
(c) C (d) D 78. Of the six members of a panel sitting in a row E is
71. Harish sits on the right of Satish, Satish sits be- the left of B, but on the right of A. F is on the right
tween Manish and Girsh. Who sits farthest to the of B but is on the left of G who is to the left of C.
right? Find the members sitting right in the middle.
(a) Satish (b) Girish (a) A E (b) B F
(c) Harish (d) Manish (c) G C (d) F G
72. Nisha is taller than Suja, Nina is taller than Nisha. 79. Raju ranks 10th from the top and Ravi ranks 21st
Nila is taller than Nina. Nisha is the tallest of all. from the bottom. If these are 3 students between
If they stand according to their height, who will them, how many students are there is the class?
be in the midle? (a) 34 (b) 33
(a) Nisha (b) Nina (c) 31 (d) 32
(c) Nisha (d) Nina 80. If you are 9th person in a queue starting from one
73. In a row of girls, Kamala is tenth from the left and end and 11th from another end, what is the num-
Vimala is twelfth from the right. When they ex- ber of persons in the queue?
change their places, Kamala is sixteenth from the (a) 20 (b) 19
left. What is the new position of Vimala from the (c) 21 (d) 18
right? 81. Five students are sitting in a row. 'T' is on the right
(a) 18th (b) 22nd of 'Z', 'M' is on the left of 'Z' but is on the right of
SSC
'L'. 'T' is on the left of 'Q'. Who is sitting first from 89. Five birds Crow, Pigeon, Little Pigeon, Big Crow
the left? and Eagle fly one after other from a tree brache.
(a) Z (b) Q Big Crow flew after Crow but is ahead of Eagle.
(c) T (d) L Pigeon is between Crow and Big Crow. Little Pi-
geon is before Crow. Which bird is the last?
82. Arun ranks 17th in a class of 31 students. What is
his ranks from the last? (a) Pigeon (b) Big Crow
(a) 14 (b) 15 (c) Ealgle (d) None of these
(c) 16 (d) 17 90. P, Q, R and S are four friends. P, is shorter than Q
but taller than R who is shorter than S. Who is the
83. At a college party five girls are sitting in a row. P
shortest among all?
is to the left of M and to the right of O. R is sitting
to the right of N, but to the left of O. Who is sit- (a) P (b) Q
ting in the middle? (c) R (d) S
(a) O (b) R 91. In a raw of trees one tree is the 7th from either end
(c) P (d) M of the row. How many tree are there in the row?
84. In a row of 16 boys, when Prakash was shifted by (a) 11 (b) 13
two places towards the left, he became 7th from (c) 15 (d) 14
the left end. What was his earlier position from 92. Akhilesh is taller than Sheebu. Aman is not as tall
the right end of the row? as Akhilesh but is taller than Tejined. Sheebu is
(a) 7th (b) 8th also not as tall as Aman but is taller than Tejinder.
(c) 9th (d) 10th Who is the tallest?
85. Five birds are sitting on a tree. The Pigeon is to (a) Akhilesh (b) Sheebu
the right of the Parrot. The Sparrow is above the (c) Aman (d) Tejinder
Parrot. The Crow is next to the Pigeon. The Crane 93. There are five friends - Satish, Kishore, Mohan,
is below the Crow. Which bird is at the centre? Anil and Rajesh, Mohan is the tallest. Satish is
(a) Crow (b) Pigeon shorter than Kishore but taller than Rajesh. Anil
(c) Parrot (d) Sparrow is little shorter than Kishore but little taller than
Satish. Who is taller than Rajesh but shorter than
86. In an examination, Rahul got the 11th rank and he
Anil.?
was 47th from the bottom among those who
passed, 3 students could not appear for the exam. (a) Anil (b) Kishore
1 student failed. What is the total number of stu- (c) Rajesh (d) Satish
dents? 94. There are five friends Suresh, Kaushal, Madhur,
(a) 60 (b) 62 Amit and Ramesh, Suresh is shorter than Kaushal
(c) 59 (d) 61 but taller than Ramesh. Madhur is the tallest. Amit
is a little shorter than than Kaushal but little taller
87. There are five houses A, B, C, D, O in a row. A is
than Suresh. If they stands in the order of their
right side of B and left side of C. O is in the right
heights who will be the shortest?
side of A. B is right of D. Which house in the
middle? (a) Amit (b) Madhur
(a) O (b) A (c) Ramesh (d) Kaushal
(c) B (d) C 95. If Ram runs less fast than Shyam and Shyam runs
as fast as Lal but less fast than Tom, who runs
88. Four persons M, N, O and P are playing cards. M
fastest?
is on the right of N and P is on the left of O. Then
which of the following are partners? (a) Lal (b) Shyam
(a) P and O (b) M and P (c) Tom (d) Tom and Lal
(c) M and N (d) N and P
SSC
96. Six friends A, B, C, D and E are sitting in a row 103. In a group of five districts Akbarpur is smaller
facing East. C is between A and E. B is just to the than Fatehpur, Dhanbad is bigger than Palamu and
right of E but left of D. F is not at the right end. Bara Banki is bigger than Fatehpur but not as big
Who is at the left end? as Palamu. Which district is the biggest?
(a) A (b) F (a) Akbarpur (b) Fatehpur
(c) C (d) B (c) Dhanbad (d) Palamu
97. F has less money than H but more then G. E has 104. Six friends A, B, C, D, E and F are sitting in a row
more than F but less than H. Who is the poorest? facing East. C is between 'A' nnd 'E', 'B' is just to
(a) F (b) E the right of 'E' but left of 'D'. 'F' is not at the right
(c) H (d) G end. How many persons are to the right of 'E'?
98. Six friends A, B, C, D, E and F are sitting in a row (a) 1 (b) 2
facing East. C is between A and E. B is just to the (c) 3 (d) 4
right of E but left of D. F is not at the right end. 105. Five friends are sitting on a bench. A is to the left
Who is at the right end? of B but on the right of C. D is to the right of B but
(a) D (b) B on the left of E. Who are at the extremes?
(c) E (d) C (a) AB (b) AD
99. If Anil runs less fast than Sunil and Sunil runs as (c) BD (d) DE
fast but not faster than Suraj, then Suraj runes: 106. Umesh is taller than Satish, Suresh is shorter than
(a) As fast as Anil (b) Faster than Sunil Neeraj but taller than Umesh. Who is the tallest
(c) Faster than Anil (d) Less fast than Anil among them?
100. Six friends A, B, C, D, E and F are sitting in a row (a) Umesh (b) Suresh
facing East. 'C' is between 'A' and 'E', 'B' is just to (c) Satish (d) Neeraj
the right of 'E' bul left of 'D', 'F' is not at the right 107. K is more beautiful than B. B is not as beautiful
end. Which pair is sitting by the side of, 'D' ? as Y. J is not as beautiful as B or Y. Whose beauty
(a) CE (b) FA is in the least degree?
(c) EB (d) FD (a) B (b) J
101. There are five Shailendra, Keshav, Madhav, Ashish (c) Y (d) K
and Rakesh, Shailendra is shorter than Keshav but 108. Age of Nareen is equal to Naveen as they are twins.
taller than Rakesh, Madhav is the tallest, Ashish Nakul is younger than Nareen, Priyanka is younger
is a little shorter than Keshav and little taller than than Balaji but elder than Naveen. Who is the
Shailendra. Who is the shortest? eledest of all?
(a) Rakesh (b) Shailendra (a) Nareen (b) Balaji
(c) Ashish (d) Keshav (c) Nakul (d) Naveen
102. Six friends A, B, C, D, E and F are sitting in a 109. X is poorer than W, but not as poor as B. C is not
row facing East. C is between A and E. B is just to as poor as X. Who is the poorest of all?
the right of E but left of D. F is not at the right (a) B (b) X
end. Who is to the left of A? (c) C (d) W
(a) E (b) C 110. Srini is taller than Anlu. Ragu is taller than
(c) D (d) F Chandru but shorter than Brinda. Srini is shorter
than Chandru. Who is the tallest?
(a) Srini (b) Ragu
(c) Chandru (d) Brinda
SSC
Answers

1. (c) 2. (a) 3. (d) 4. (c) 5. (d) 6. (a) 7. (b) 8. (b) 9. (c) 10. (c)
11. (b) 12. (b) 13. (b) 14. (b) 15. (d) 16. (b) 17. (c) 18. (b) 19. (a) 20. (b)
21. (b) 22. (c) 23. (c) 24. (a) 25. (a) 26. (a) 27. (a) 28. (c) 29. (d) 30. (b)
31. (a) 32. (b) 33. (c) 34. (c) 35. (c) 36. (c) 37. (c) 38. (d) 39. (d) 40. (b)
41. (c) 42. (b) 43. (a) 44. (d) 45. (c) 46. (a) 47. (b) 48. (a) 49. (d) 50. (b)
51. (a) 52. (b) 53. (a) 54. (c) 55. (d) 56. (d) 57. (c) 58. (b) 59. (b) 60. (c)
61. (b) 62. (b) 63. (b) 64. (d) 65. (a) 66. (d) 67. (c) 68. (b) 69. (c) 70. (b)
71. (c) 72. (b) 73. (a) 74. (a) 75. (c) 76. (c) 77. (d) 78. (b) 79. (a) 80. (b)
81. (d) 82. (b) 83. (a) 84. (b) 85. (b) 86. (b) 87. (b) 88. (d) 89. (c) 90. (c)
91. (b) 92. (a) 93. (d) 94. (c) 95. (c) 96. (b) 97. (d) 98. (a) 99. (c) 100. (c)
101. (a) 102. (d) 103. (c) 104. (b) 105. (d) 106. (d) 107. (b) 108. (b) 109. (a) 110. (d)
111. (a) 112. (b) 113. (c) 114. (c)
SSC
12. CUBES & DICE
What is cube? Have only one surface painted
It is a three dimensional body having (2) Middle Cube:
(i) 6 equal faces Have only two faces painted
(ii) 12 edges (3) Corner Cube:
(iii) 8 corners Have only three surface painted
(iv) Length = Breadth = Heigth (4) Inner central Cube:
Have no surface painted
Finding number of smaller cubes when a larger cube
is cut
Inner cube = (n - 2)3
Central cube = 6(n - 2)2
Middle cube = 12(n - 2)
Corner cube = Always is 8.
Total no. of cube = (n)3
Where n = Cube root of total smaller cubes
When a cube is cut into smaller cubes of equal di- If total smaller cubes are 64, then
mension
n= =4
Formats of the questions
Example:Directions (Q. 1 - 6): Read the following
information to answer the questions that follow.
A cube of side 4cm is painted black on all its surface
and the divided into various smaller cubes of side 1cm
each. The smaller cubes so obtained are separated.
1. What is the total number of cubes so obtained?
(a) 54 (b) 64
(c) 58 (d) 65
(e) None of these
Central Cube = C = 
Explanation: Option (b) is correct. Let us see:-
Corner Cube = Cr = 
Middle Cube = M =  Required no. of cubes =
(1) Central Cube (C): 2. Find the number of division on the surface of
Exists at the centre of the every surface. the bigger cube?
(2) Middle Cube (M): (a) 4 (b) 5
Exists at the middle of each edge. (c) 8 (d) 3
(3) Corner Cube (Cr): (e) None of these
Exists at the every corner. Explanation: Option (a) is correct. Let us see:-
(4) Inner central Cube: Required no. of division (n)
Hidden and exists at the centre of the larger cube
Position of a painted cube =
(1) Central Cube:
SSC
3. How many cubes have two surfaces painted?
(a) 35 (b) 30
(c) 28 (d) 24
(e) None of these
Explanation: Option (d) is correct. Let us see:- Sum of the digits or points of opposite surfaces
Middle cubes = (n - 2) 12 = (4 - 2) 12 = 24 1+6=7 4+3=7 2+5=7 5+
4. How many cubes have one surface painted? 2=7 3+4=7 6+1=7
(a) 24 (b) 37 General Dice:
(c) 18 (d) 20 In a general dice, the sum of digits or points of any two
(e) None of these adjacent surfaces is equal to 7.
Explanation: Option (a) is correct. Let us see:-
Central cubes = (n - 2)2 6 = (4 - 2)2 6 = 24
5. How many cubes have three surface painted?
(a) 8 (b) 4
(c) 15 (d) 2 Sum of the digits or points of adjacent faces:
(e) None of these 1+2=3 2+1=3 3+2=5
Explanation:  Option (a) is correct as no. of corner 4+1=5 5+1=6 6 + 1 = 7
cubes are 8. 1+4=5 2+3=5 3 + 4 = 7
6. Find  the  number  of  smaller  cubes  with  no 4+2=6 5 + 2 = 7 6+3=9
surface painted?
1+5=6 2+4=6 3+5=8
(a) 5 (b) 9
4 + 3 = 7 5+3=8 6 + 4 = 10
(c) 8 (d) 4
1 + 6 = 7 2 + 5 = 7 3+6=9
(e) None of these
4 + 6 = 10 5 + 6 = 11 6 + 5 = 11
Explanation: Option (c) is correct. Let us see:-
Unfolded Dice:
Inner cubes = (n - 2)3 = (4 - 2)3 = 23 = 8
When the surfaces of dice are unfolded and placed on
What is dice? a plane, then the figure of dice so obtained will look
It is a cube having digits or points at each of like one of the following:
its surface. Some times other things like letters, signs, I. Opposite surfaces
words etc. are also given on its surfaces. 1&5
2&4
3&6
       
II. Opposite surfaces
1&6
2&4
3&5

III. Opposite surfaces
Standard Dice: 1&3
In a standard dice, the sum of digits or points of oppo- 2&5
site surfaces is 7.
4&6
SSC
(a) 3/5 (b) 3/4/5
IV. Opposite surfaces (c) 3 (d) 4/5
1&6 (e) None of these
2&5 Explanation: Option (b) is correct it is not a standard
3&4 dice as 6 and 1 exist at adjacent surfaces. Hence 3/
4/5 will opposite 2.
Example 2: Which of the following is not a standar
V. Opposite surfaces dice?
1&6
2&5

3&

(a) (ii) (b) (i)


VI. Opposite surfaces
1&6 (c) (iii) (d) (i) & (iii)
2&5 (e) None of these
Explanation: Option (a) is correct as it has 6 and 1 at
& its adjacent sides and 6 + 1 = 7 which is possible
only in general dice.
Example 3: What come in place of (?) ? (all figures
are of the same cube)
VII. Opposite surfaces
1&6
2&5
3&
(a) 1 (b) 6
(c) 5 (d) 2
VIII. Opposite surfaces (e) None of these
1&6 Explanation:  Option (a) is correct as the unfolded
2&5 surfaces of the cube is as follows
3& Opposite surfaces 1&
5
Formats of the questions 2&4
3&6
Example 1: What comes opposite 2 in the given dice?
SSC
Exercise
1. Find the digit at the opposite surface of 3 in the given
figure.

¼a½ 1 ¼b½ 2
¼c½ 3 ¼d½ 6
¼a½ 1 ¼b½ 5
(e) None of these
¼c½ 6 ¼d½ 1/5/6 6. See  the  following  figure  and  find  the  digit
(e) None of these opposite to 1.
2. What  digit  will  be  opposite  to  4  in  the  given
dice?

¼a½ 2 ¼b½ 3
¼c½ 4 ¼d½ 5
¼a½ 1 ¼b½ 5
(e) None of these
¼c½ 6 ¼d½ 1/5/6 7. The  digit  opposite  to  4  in  the  following  figure
(e) None of these is.......
3. In the given dice, what digit is opposite to 4?

¼a½ 1 ¼b½ 2
¼a½ 1 ¼b½ 2 ¼c½ 5 ¼d½ 6
¼c½ 1/6 ¼d½ Data inadequate (e) None of these
8. Find the digit which is on the opposite surface
(e) None of these
of 6.
Directions (Q 4 - 12): More than one dice/cube given
in each question are different positions of same dice/
cube.
4. What comes opposite to 3 in the given figure?

¼a½ 1 ¼b½ 3
¼c½ 4 ¼d½ 5
(e) None of these
¼a½ 1 ¼b½ 2 9. 3 is opposite to ........... in the given figure.
¼c½ 5 ¼d½ 6
(e) None of these
5. Find the digit opposite to 4.
SSC
¼a½ 1 ¼b½ 2
¼c½ 4 ¼d½ 5
(e) None of these ¼a½ ¼b½
10. How  many  points  are  there  on  the  surface
opposite to the surface having 3 points.

¼c½ ¼d½

(e) None of these


¼a½ 1 ¼b½ 2 14.
¼c½ 5 ¼d½ 6
(e) None of these
11. What  is  the  number  of  points  at  the  surface
opposite to the surface having 3 points.

¼a½ ¼b½

¼a½ 2 ¼b½ 4
¼c½ 5 ¼d½ 6
(e) None of these
12. In  the  following  figure  2   points  surface  is ¼c½ ¼d½
opposite to ......... points surface.

(e) None of these


15.

¼a½ 1 ¼b½ 4
¼c½ 5 ¼d½ 6
(e) None of these
Directions (Q 13 - 15): Find which dice/cube can be
¼a½ ¼b½
made with the given unfolded figure of a dice/cube.
13.

¼c½ ¼d½

(e) None of these


Directions (Q 16 - 21): A cube is painted red on two
adjacent surfaces and black on the surfaces opposite
to  red  surfaces  and  green  on  the  remaining  faces.
SSC
Now the cube is cut into sixty four smaller cubes of (a) 20 (b) 8
equal sizes. Answer the following questions. (c) 24 (d) 32
16. How many smaller cubes have only one surface
(e) None of these
painted?
25. How  many  cubes  will  have  only  two  surfaces
(a) 8 (b) 16 painted with red and blue colours, respectively?
(c) 24 (d) 32 (a) 8 (b) 12
(e) None of these (c) 24 (d) 30
17. How  many  smaller  cubes  will  have  no  surface
(e) None of these
painted?
Directions (Q. No. 26 - 30): A cube is coloured red
(a) 0 (b) 4 and two opposite faces, blue on two adjacent faces
(c) 8 (d) 16 and  yellow  on  two  remaining  faces.  It  is  then  cut
(e) None of these into two halves along the plane parallel to the red
18. How  many  smaller  cubes  have  less  than  three faces.  One  piece  is  then  cut  into  four  equal  cubes
surfaces painted? and the other one into 32 equal cubes.
26. How  many  cubes  do  not  have  any  coloured
(a) 8 (b) 24
faces?
(c) 28 (d) 48
(a) 0 (b) 16
(e) None of these
(c) 4 (d) 8
19. How  many  smaller  cubes  have  three  surfaces
painted? (e) None of these
27. How many cubes do not have any red faces?
(a) 4 (b) 8
(a) 8 (b) 16
(c) 16 (d) 24
(c) 20 (d) 24
(e) None of these
20. How  many  smaller  cubes  with  two  surfaces (e) None of these
painted  have  one  face  green  and  one  of  the 28. How  many  cubes  have  at  least  two  coloured
adjacent faces black or red? faces?
(a) 8 (b) 16 (a) 20 (b) 24
(c) 24 (d) 28 (c) 28 (d) 32
(e) None of these (e) None of these
21. How  many  smaller  cubes  have  at  least  one 29. How many cubes have each a yellow face with
surface painted with green colour? other faces without colour?
(a) 8 (b) 24 (a) 4 (b) 14
(c) 32 (d) 56 (c) 17 (d) 20
(e) None of these (e) None of these
Directions (Q 22 - 25): A cube of side 4cm has been 30. How many cubes have at least one face painted
painted  on  its  surfaces  in  such  a  way  that  two blue?
opposite  surfaces have  been  painted  blue and  two (a) 4 (b) 14
adjacent  surfaces  have  been  painted  red.  Tw o (c) 17 (d) 20
remaining surfaces have been left unpainted. Now
(e) None of these
the cube is cut into smaller cubes of side 1cm each.
Directions (Q. No. 31 - 35): More than one dice/cube
Now answer the following questions.
given in each question are different positions of the
22. How many cubes will have no surface painted?
same dice/cube.
(a) 18 (b) 16 31. What comes opposite to 6 in the given dice?
(c) 22 (d) 8
(e) None of these
23. How  many  cubes  will  have  at  least  red  colour
on its surfaces?
(a) 20 (b) 22 (a) 1 (b) 3
(c) 28 (d) 32 (c) 5 (d) 1/3
(e) None of these (e) None of these
24. How many cubes will have at least blue colour
on its surfaces?
SSC
32. Which  digit  will  come  at  the  surface  opposite
to the surface having digit 3?
(a) (b)

(a) 4 (b) 5 (c) (d)


(c) 6 (d) 2
(e) None of these (e) None of these
33. Find the digit at the surface  which is opposite Directions  (Q.  No.  36  -  40):A  solid  cube  has  been
to the surface having digit 4. pa inted  y ellow,  blue  and  black  on  the  pair  of
opposite  surfaces.  The  cube  is  then  cut  into  36
smaller  cubes  such  that  32  cubes  are  of  the  same
size while four others are of bigger size. Also no face
of any of the bigger cubes is painted blue.
36. How many cubes have at least one face painted
black?
(a) 1 (b) 2
(a) 20 (b) 8
(c) 3 (d) 5
(c) 16 (d) 32
(e) None of these
34. In  the  following  figures,  how  many  points  are (e) None of these
there  at  the  surface  opposite  to  the  surface 37. How many cubes have only one face painted?
having 4 points? (a) 0 (b) 4
(c) 8 (d) 12
(e) None of these
38. How many cubes have only two faces painted?
(a) 24 (b) 20
(c) 16 (d) 12
(e) None of these
(a) 1 (b) 2 39. H o w   ma n y   c ub e s  h a v e   t w o   o r  mo re   f a ce s
(c) 3 (d) 4 painted?
(e) None of these (a) 36 (b) 20
35. Which  dice/cube  can  be  made  by  folding  the (c) 28 (d) 24
given unfolded figure? (e) None of these
40. How many cubes have only three faces painted?
(a) 8 (b) 4
(c) 2 (d) 0
(e) None of these

Answers
1. d 2. d 3. d 4. b 5. c 6. c
7. d 8. a 9. c 10. b 11. d 12. d
13. b 14. b 15. d 16. c 17. c 18. d
19. b 20. b 21. c 22. a 23. c 24. d
25. b 26. c 27. b 28. a 29. a 30. c
31. a 32. a 33. a 34. a 35. b 36. a
37. c 38. b 39. c 40. a
SSC
13. BLOOD RELATION
What is blood relation? and R is male, then we can easily say that R is the
Blood relation is biological relation. Remember a wife son of P & Q. Further, we can also say that P is the
and husband are not biologically related but they are father of R and Q is the mother of R.
biological parents of their own children. Similarly, (2) Gender can not be decided on the basis of name.
brother, sister, paternal grandfather, paternal For example, in sikh community the names like
grandmother, maternal grandfather, maternal Manjit, Sukhvinder etc. are the names of both male
garandmother, grandson, grandmother, niece, cousin and female. Similarly, in the Hindu community
etc. are our blood relatives. 'Suman' is the name of both male and female.
Types of Blood Realtions Important Signs
(i) Blood Relation From Paternal Side (i) ' ' is used for husband & wife
This type of blood relation can be further subdivided (ii) ' ' is used for brother & sister
into three types: (iii) ' ' is used for parents (Father & mother). Parents
(a) Past generations of father
Examples: Great grandfather, great grandmother, are put on the top while children are put at the
grandfather, grandmother etc. bottom
(b) Parallel generations of father (iv) '-' or minus sign is used for female
Examples: Uncles (Brothers of father), aunts (Sisters (v) '+' or plus sign is used for male
of father) etc. Now adopting and using the above given symbols
(c) Future generations of father we can make a family tree . Let us see :-
Examples: Sons, daughters, grandsons, grand
daughters etc.
(ii) Blood Relation From Maternal Side
This type of blood relations can also be subdivided
into three types:- (A, C, Q = 1st generation people)
(a) Past generations of mother (R, D = 2nd generation people)
Examples: Maternal great grandfather, maternal great As per the given diagram Q is the brother of C and
grandmother, maternal grandfather, C is the sister of Q. Where '-' sign above C makes
maternal grandmother etc. it clear that C is a female and '+' sign above 'Q'
(b) Parallel generations of mother makes it clear that Q is a male. Similarly, R is the
brother of D and D is the sister of R. Further,
Exmples: Maternal uncles, maternal aunts etc.
according to the given informations, A and C are
(c) Future generations of mother
having a husband and wife relationship and hence
Examples: Sons, daughters, grandsons, grand this has been presented as (A + C -) . Lastly,,
daughters etc. the vertical line gives father and son relationship
Keep in Mind
(1) Without the information of gender, no relationship
can be established between two people. For ex- and has been presented as . Here, A is father
ample, if given that R is the child of P & Q, then
we can only say that P & Q are the parents of R.
and hence it is put on top while R is put at bottom
But we cannot find out the following.
because R is the son of A. If minus (-) sign is put
(i) R is the son of P & Q or R is the Daughter of P & for R, then R will be daughter of A. Now, from
Q. this family diagram it becomes clear that C is the
(ii) Who is the mother of R and who is the father of R. mother of R and D and as Q is the brother of C
But if it is given that P is a male, Q is a female
SSC
then Q will definitely be the maternal uncle of R
& D.
Formats of the questions
Example  1:  Pointing to a person, a man said to a
woman,'His mother is the daughter of your father'.
How was the woman related to the person?
(a) Sister (b) Uncle Example 3: If ‘P  Q’ means ‘P is the brother of Q’;
(c) Father (d) Aunt ‘P   Q’ means ‘Q is the mother of P’; ‘P - Q’
(e) None of these means ‘P’ is the father of ‘Q’ and ‘P + Q’ means
Explanation: Option (d) is correct because daughter ‘Q is the sister of P’, then which of the following
of your father your sister; the person's mother means ‘M is the daughter of T’?
is the woman's sister. Woman is person's aunt. (a) M + N J - T (b) T - J R + M
Example 2: If E is the mother of C; S is the husband of (c) M - J T K (d) M + W R T
E; A is the brother of B; B is the sister of M; C is (e) None of these
the mother of B and D is the brother of C then, S is Explanation: Option (b) is correct as (a) and (d) are
the Paternal grandfather of ........... ruled out because the gender of M is not known.
(a) D (b) E (c) is ruled out as M is a male. But (b) is correct
(c) C (d) A, B and M which shows the family tree given below:-
(e) None of these
Explanation:  Option (d) is correct because in such
problems a family tree is required.Let us see the
tree below:
Clearly, M is the daughter of T.
Some important blood relations
1. Son of father or mother Brother
2. Daugther of father or mother Sister
3. Brother of father Uncle
4. Brother of mother Maternal uncle
5. Sister of father Aunt
6. Sister of Mother Aunt
7. Father of father Grandfather
8. Father of father of father Great grandfather
9. Father of grandfather Great grandfather
10. Mother of father Grandmother
11. Mother of mother of father Great grandmother
12. Mother of grandmother Great grand mother
13. Father of mother Maternal grand father
14. Father of father of mother Great maternal grandfather
15. Father of maternal grandmother Great maternal grandfather
16. Mother of mother Maternal grandmother
17. Mother of mother of mother Great maternal grandmother
18. Mother of maternal grandmother Great maternal grandmother
19. Wife of father Mother
20. Husband of mother Father
21. Wife of grandfather Grandmother
22. Husband of grandmother Grandfather
23. Wife of son Daughter in law
24. Husband of daughter Son in law
25. Brother of husband Brother in law
SSC
26. Brother of wife Brother in law
27. Sister of Husband Sister in law
28. Sister of wife Sister in law
29. Son of brother Nephew
30. Daughter of brother Niece
31. Wife of brother Sister in law
32. Husband of sister Brother in law
33. Son of sister Nephew
34. Daughter of sister Niece
35. Wife of uncle Aunt
36. Wife of maternal uncle Aunt
37. Son/daughter of uncle/aunt Cousin
38. Son/daughter of maternal uncle/
maternal aunt Cousin
39. Son/daughter of sister of father Cousin
40. Son/daughter of sister of mother Cousin
41. Only son of grandfather Father
42. Only daughter of maternal
grandfather Mother
43. Daughter of grandfather Aunt
44. Sons of grandfather other
than father Uncle
45. Son of maternal grand father Maternal uncle
46. Only daughter in law of
grandfather/grandmother Mother
47. Daughter in law of
grandfather/grandmother Aunt
48. Daughter in law of maternal
grandfather/maternal grandmother Aunt
49. Neither brother nor sister Self

Exercise

1. Pointing towards a picture, Anjali says, 4. Pointing towards Puja, Vinod said that her
“He is the son of my grandfather’s only father is the son of my mother’s sister. How
son”. How is the boy's picture related to is Puja related to Vinod?
Anjali? (a) Niece (b) Brother
(a) Brother (b) Uncle (c) Cousin (d) Grandfather
(c) Son (d) Nephew 5. Pointing towards a man in photograph, a
2. Kamala told Mohan that your mother is woman said, “His brother’s father is the
the only daughter of my mother. How is only son of my grandfather”. How is the
Kamala related to Mohan? women related to the man in photograph?
(a) Sister (b) Mother (a) Mother (b) Aunt
(c) Daughter (d) Aunt (c) Sister (d) Daughter
3. Arun said, “This girl is the wife of my 6. Pointing towards a boy in the photograph,
mother’s grandson”. How is Arun related Reena says, “He is the only son of my
to this girl? grandfather’s only son”. How is Reena
(a) Father (b) Grandfather related to boy's photograph?
(c) Husband (d) Father-in-law (a) Mother (b) Sister
(c) Aunt (d) None
SSC
7. Introducing a woman, a man said, “Her 14. Pointing towards a photograph a man
only brother is the only son of my father”. says, “I do not have any brother or sisters
How is the woman related to the father of but the father of this man is son of my
that man? father”. Whose picture is this?
(a) Wife (b) Sister (a) Self/ His own (b) His Son
(c) Daughter (d) Mother (c) His father (d) His Nephew
8. Pointing towards a man, a woman said, 15. Indicating to wards a picture (photograph)
“His mother is the only daughter-in-law a lady says that the sister of this man’s
of my mother-in-law”. How is the man son is my mother-in-law, then what is the
related to the woman? relation between that lady’s husband and
(a) Husband (b) Son the picture.
(c) Father (d) Brother (a) Son in law (b) Grandson
9. Pointing towards a woman in picture, a (c) Son (d) Nephew
man said, “Her sister’s father is the only 16. Introducing a lady, a man says, Sister of
son of my grandfather”. How is that man Her mother’s husband is my aunt how is
related to the woman in picture? that related to the man.
(a) Uncle(b) Father (a) Aunt
(c) Son (d) Brother (b) Mother
10. Neetu said that Anju’s anut is the only (c) Maternal cousin
daughter of my maternal grandmother, (d) Paternal Cousin
then how is Anju related to Neetu? 17. Indicating towards Namita Mohan says,
(a) maternal brother “the brother of your husband is mother is
(b) maternal sister the maternal uncle of my father”. How is
(c) Aunt Mohan related to Namita.
(d) maternal uncle (a) Brother (b) Husband
11. Pointing towards a man, woman said, “His (c) Uncle (d) Son
mother is the only daughter of my mother”. 18. Ravina told Karishma, “your sister is the
How is that woman related to the man? daughter of my mother’s brother." How
(a) Mother Karishma is related to brother of Ravina’s
(b) Daughter mother.
(c) Sister (a) Daughter
(d) maternal grandmother (b) Sister
12. Pointing towards a picture, Mihir says, (c) Nephew
“His sister is mother of my brother’s son (d) Daughter-in-law
Tushar”. How is Mihir related to Tushar? 19. Walking with his friend, Mohan meets a
(a) Brother (b) Uncle man whose mother is the wife of only son
(c) Nephew (d) Brother-in-law of Mohan’s father. How is that man related
to Mohan?
13. Pointing towards a photograph, a man
says to his friend, “This woman is the (a) Son (b) Uncle
granddaughter of my father ’s elder (c) Nephew (d) Brother
brother”. How is woman in photograph 20. Pointing toward a lady, Neelam said, “Her
related to that man? son’s father is my son-in-law”. Then, how
(a) Niece (b) Daughter is Neelam related to that lady?
(c) Aunt (d) Sister-in-law (a) Sister (b) Niece
SSC
(c) Daughter (d) Mother (a) Son
21. There are two character in a film. Out of (b) Maternal Nephew
them one is the father of other’s son. How (c) Paternal Nephew
are they related with each other? (d) None
(a) Grandfather and Grandson 28. A man married his daughter to his
(b) Grandfather and Son maternal anut’s son. What did son-in-law
(c) Husband and wife call that man before marriage?
(d) Father and son (a) Uncle
22. Sunita, daughter of Rakhi, says to Seema, (b) Brother
“your mother is younger sister of my father (c) Paternal Brother
who is the third daughter of Prem Kumar, (d) Aunt
find out the relationship of PremKumar
with Seema. 29. M is the son of P. Q is the grand daughter of O
who is the husband of P. How is M related to O ?
(a) Maternal grandfather
(b) Father (a) Son (b) Daughter
(c) Uncle (c) Mother (d) Father
(d) Brother 30. P is the brother of Q. R is the mother of Q.
23. Introducing Suchendra, Naman said, “She S is the father of R. T is the mother of S.
is the wife my only brother’s nephew”. How Then, how is P related to T?
is Suchendra related to Naman? (a) Granddaughter
(a) Wife (b) Sister (b) Great grandson
(c) Sister-in-law (d) None (c) Grandson
24. Introducing a man, a lady said, “Father of (d) Grandmother
his father-in-law is my father-in-law”. How 31. X is the sister of Y. Y is the daughter of K.
is that man is related to lady? K is the husband of L. How is L related to
(a) Father (b) Husband Y?
(c) Son (d) Son-in-law (a) Brother (b) Sister
25. Pointing towards a girl in picture, Umesh (c) Father (d) Mother
said, “Her mother’s brother is the only son 32. A is the daughter of B. B is the mother of
of my mother’s father”. How is girl’s C. D is the brother of C. Then, how is D
mother related to Umesh? related to A?
(a) Mother (a) Father (b) Grandfather
(b) Maternal aunt (c) Brother (d) Son
(c) Mother/maternal aunt 33. X is brother of Y. Y is wife of Z. Z is son of
(d) None W. W is wife of V. How is V related to Y?
26. Pointing towards Neeru, Asha says, “I am (a) Mother-in-law (b) Brother-in-law
the only daughter of her mother’s son”. (c) Father-in-law (d) Son-in-law
How is Neeru related to Asha? 34. E is sister of B. A is father of C. B is son of
(a) Paternal Aunt (b) Paternal Sister C, then, how is A related to E?
(c) Niece (d) Mother (a) Grandfather
27. Pointing towards Kedaar, Beena said, “His (b) Granddaughter
mother’s brother is the father of my son (c) Father
Nitin”. How is Kedaar related to Beena.
(d) Great Grandfather
SSC
35. Grandmother, father, mother, five son and Direction (Q. 41-42) - Study the following information
their wives. Two sons and one daughter of and answer the questions given below
each son are presented in a party. How (i) ‘P ÷ Q’ means ‘P, is sister of Q’.
many members are there in the party? (ii) ‘P × Q’ means ‘P, is brother of Q’.
(a) 12 (b) 16 (iii) ‘P – Q’ means ‘P, is mother of Q’.
(c) 24 (d) 28 (iv) ‘P + Q’ means ‘P, is father of Q’.
36. There are a man, his wife, four son and 41. Which of the following means ‘M is maternal
their wives. Each son has three boy and uncle of T’ ?
one girl in the family. How many male
(1) M ÷ K – T (2) M × K + T
members are there in the family?
(3) M × K – T (4) M ÷ K – T
(a) 1 (b) 12
(5) None of the above
(c) 4 (d) 17
42. Which of the following means ‘H is maternal uncle
37. Neeraj’s father has two brother. Elder
of T’ ?
brother has two son and three daughter
and younger brother has two son and two (1) H + J + T (2) T × K + H
daughter. All male members are married (3) H + J × T (4) H – J + T
and Neeraj’s all paternal brother have two (5) None of the these
daughters each. They all live in a family. Directions (Q. 43-46) – These questions are based on
Tell, how many members are there in the the following information. Study it carefully and an-
family while all daughters are unmarried? swer the questions.
(a) 37 (b) 48 (i) ‘A × B’ means ‘A is father of B’.
(c) 29 (d) 31 (ii) ‘A ÷ B’ means ‘A is daughter of B’.
38. Mr. and Mrs. Gopal have three daughters. (iii) ‘A + B’ means ‘A is sister of B’.
Each daughter has a brother too. Then, (iv) ‘A – B’ means ‘A is husband of B’.
how many members are there in the
43. Which of the following indicates ‘N is mother of
family?
K’?
(a) 5 (b) 6
(1) K + L ÷ N × F (2) K + L ÷ N – M
(c) 7 (d) 8
(3) H × K ÷ N (4) N × F + K
39. Santosh was travelling in a train. He met
(5) None of these
a man and five women. Each woman was
holding a baby in her arms. How many 44. In F ÷ R × H – L, how is H related to F ?
members were there in the railway coach? (1) Father (2) Brother
(a) 9 (b) 10 (3) Sister (4) Cannot be determined
(c) 11 (d) 12 (5) None of these
40. There was a group of some siblings 45. In G × T + Q ÷ M, how is M related to G ?
(brother-sister). The number of Ram’s (1) Brother (2) Sister
sister was double to his brother – but (3) Sister-in-law (4) Cannot be determined
according to Sita the number of Ram’s (5) None of these
sisters and brothers were equal. Then, how
46. In F – R + H ÷ T, how is F related to T ?
many brother sisters were there in the
group? (1) Son-in-law (2) Daughter-in-law
(a) 3 brother & 3 sister (3) Son (4) Daughter
(b) 4 brother & 3 sister (5) None of these
(c) 3 brother & 4 sister Direction (Q. 47-48) – These questions are based on
the following information.
(d) 2 brother & 2 sister
SSC
(i) ‘A * B’ means ‘B is father of A’. (2) Mother-in-law/Father-in-law
(ii) ‘A $ B’ means ‘A is the sister of B’. (3) Grandmother
(iii) ‘A @ B’ means ‘A is mother of B’. (4) Grandmother/Grandfather
(iv) ‘A = B’ means ‘B is brother of A’. (5) None of these
47. If R @ Q = L * M = P, how is R related to P ? 53. How is R related to Q, if ‘P – Q + R ÷ T’ ?
(1) Cousin sister (2) Mother-in-law (1) Brother/Sister
(3) Aunt (4) Data inadequate (2) Niece
(5) None of these (3) Sister
48. Which of the following means L is the aunt of Q ? (4) Nephew/Niece
(1) L @ P $ M = Q (2) L $ M @ N = Q (5) None of these
(3) L = N * R = Q (4) L $ M * N = Q 54. How is T related to P if ‘P × Q – T + R’ ?
(5) None of these (1) Mother
Directions (Q. 49-51) – These questions are (2) Father-in-law
based on the following information. (3) Mother-in-law
(i) ‘P × Q’ means ‘P is brother of P’. (4) Mother-in-law/Father-in-law
(ii) ‘P ÷ Q’ means ‘Q is mother of P’. (5) None of the above
(iii) ‘P – Q’ means ‘P is mother of Q’. Directions (Q. 55-60) – Read the following informa-
(iv) ‘P + Q’ means ‘Q is brother of P’. tion carefully and answer the questions which follow :
49. Which of the following means ‘M is the daughter (i) ‘A × B’ means ‘A is father of B’.
of T’? (ii) ‘A + B’ means ‘A is daughter of B’.
(1) M + N ÷ J – T (2) T – J × R + M (iii) ‘A ÷ B’ means ‘A is mother of B’.
(3) M – J × T ÷ K (4) M + W × R ÷ T (iv) ‘A – B’ means ‘A is brother of B’.
(5) None of these 55. If ‘ P ÷ R – Q × T’ how is P related to T ?
50. How is K related to R in the expression : (1) Grandmother (2) Mother-in-law
R÷T+K? (3) Sister (4) Grandfather
(1) Daughter (2) Sister (5) None of these
(3) Niece (4) Cannot be
56. If ‘ P ÷ Q + R × T’ how is T related to Q ?
determine
(1) Aunt (2) Sister
(5) None of these
(3) Brother (4) Grandson
51. Which of the following means ‘D is the
grandfather of W’ ? (5) None of these

(1) D K×T W (2) D ÷ K × T ÷ W 57. Which of the following means that R is wife of
P?
(3) D K × T ÷ W (4) D ÷ K × T W
(1) P × R – Q – T (2) P ÷ T + R – Q
(5) None of these
(3) P ÷ R – Q + T (4) P × T – Q + R
Directions (Q. 52-54) – Read the following information
(5) None of these
carefully and answer the questions which follow :
58. If ‘ R – P ÷ J × Q’ how is J related to R ?
(i) If ‘A × B’ means ‘A is wife of B’.
(1) Son/Daughter(2) Nephew
(ii) If ‘A + B’ means ‘A is brother of B’.
(3) Niece (4) Grandson
(iii) If ‘A ÷ B’ means ‘A is daughter of B’.
(5) None of these
(iv) If ‘A – B’ means ‘A is son of B’.
52. How is Q related to P if ‘P ÷ R × T – Q’ ?
(1) Granddaughter
SSC
59. If ‘ P + Q – R ÷ T’, how is T related to P ? 60. If ‘ P × T ÷ Q + R’ how is R related to P ?
(1) Aunt (2) Aunt/Uncle (1) Daughter
(3) Father (4) Grandmother (2) Husband
(5) None of these (3) Son-in-law
(4) Son-in-law/Daughter-in-law
(5) None of these

Answers
1. a 2. b 3. d 4. a 5. c 6. b
7. c 8. b 9. d 10. b 11. a 12. b
13. a 14. b 15. b 16. d 17. b 18. a
19. a 20. d 21. c 22. a 23. a 24. d
25. c 26. a 27. b 28. c 29. a 30. a
31. d 32. c 33. c 34. a 35. d 36. d
37. c 38. b 39. d 40. c 41. C 42. e
43. C 44. b 45. e 46. a 47. e 48. b
49. e 50. e 51. a 52. d 53. e 54. b
55. a 56. e 57. d 58. b 59. e 60. c
SSC
14. LOGICAL VENN DIAGRAM
What is Venn Diagram? B = Minute
Venn diagram is the presentation of certain group of C = Hour
items through geometrical figures like circles, triangles, Diagram 3:
rectangles etc.
Questions based on venn diagram analyse a
candidate’s ability to relate a certain group of items
and illustrate them diagrametically.
How to present Venn Diagram?
Diagram 1:

The given diagram shows that two groups A and C are


partly related to the third group B and are themselves
independent of each other.
Example: Dogs, Pets, cats
Explanation:It is clear, that some dogs and some cats
are pets. But all pets are not dogs or cats. Also dogs
and cats are completely different groups and hence,
The given diagram represents three different groups not related to each other.
A, B and C having no relationship to one another. It In the given diagram
means no item is common in A, B and C. A = Dogs
Example: Teachers, Lawyers, Doctors B = Pets
Explanation: These three professionals bear no C = Cats
relationship to one another as all the given groups are Diagram 4:
associated with three different professions.
Diagram 2:

The given diagram shows that two different items A


The diagram shows that B completely includes A but and B belong to the group of third such that some items
A does not completely include B. of each of these two groups are common.
and Example: Females, Mothers, Sisters
C completely includes B but B does not completely Explanation: As we know, some mothers may be
include C. sisters or some sisters may be mohters. Also, both
mothers and sisters are females.
Example:Seconds, Minutes, Hours
In the given diagram
Explanation: Second is part of minute and minute
is part of hour. A = Mothers
Clearly, in diagram. B = Sisters
A = Second C = Females
SSC
Diagram 5: B = Women
C = Teachers
Diagram 7:

The given diagram shows that two independent groups


A and B belong to the third group C. Also, A includes
some items of C and B includes some other items of C.
Example: Table, Chair, Furniture
The given diagram shows that B completely includes
Explanation: No table can be chair and also no chair A while A does not completely include B
can be table. But both table and chair represent the
and
group of furniture. In the given diagram
C is completely different from A and B.
A = Table
Example: Doctors, Human Being, Cats
B = Chair
Explanation: All doctors come under the group of
C = Furniture
human  beings but all human  beings do not come
under the group of doctors
Diagram 6: and
Cats are entirely different from doctors and human
beings.
In the given diagram
A = Doctors
B = Human beings

The given diagram shows that A, B and C are partly C = Cats


related to each other as shaded part is common in all Diagram 8:
the three.
Also
 A and B partly include each other
 B and C partly include each other
 A and C partly include each other
Example: Married, Women, Teachers
Explanation: Some married are women. The given diagram shows that B completely includes
Some married are teachers. A while C partly belongs to A & B.
Some teachers are women. Example: Males, Father, Doctors
Some women are teachers. Explanation: As we know that all fathers are males
but some males and some fathers can be doctors.
In the given diagram
In the given diagram
A = Married
SSC
A = Fathers Example: Teachers, Authors, Birds
B = Males Explanation: Some teachers are authors.  Some
C = Doctors authors are teachers
Diagram 9: But
Birds are entirely different from teachers and authros.
In the given diagram
A = Teachers
B = Authors
C = Birds

Diagram 11:

The given diagram shows that B completely includes


A but C partly belongs to B.
The given diagram shows that A and B completely in-
Example: Females,Mothers, Children
clude C but they partly represent each other.
Explanation: All mothers  represent the group of
Example: Men, Married, Husband
females while all females do not represent the group
of mothers. Explanation: It is clear, that
and • All husbands are married.
Some children represent females but some children • All husbands are men.
do not represent females. • Some men are husbands
In the given diagram • Some married are husbands (some married can be
A = Mothers wives also)
B = Females • Some men are married.
C = Children In the given diagram
A = Men
Diagram 10: B = Husbands
C = Married
Note:
(i) In place of circle, the diagrams may be represented
by other geometrical figures like triangles
rectangles, squares etc.
(ii) In the venn diagram more than one geometrical
figure can be used collectively.
The given diagram shows that A and B are partly re-
lated to each other while C is entirely different from A
and B.
SSC
Exercise
1. P and Q are two independent groups. Which
of the following options does not represent
them? (a) (b)

(a) (b)

(c) (d)

(c) (d)
(e) None of these
Directions (Q 5-9): In the following questions,
three  classes  are  given  out  of  the  five  figures
(e) None of these that follow, you have to indicate which figure
will best represent the relationship amongst the
2. Which of the following is correct about the three classes.
diagram given below?

(a) (b)

(a) All R are T (b) All P are R


(c) (d)
(c) All T are P (d) All P are T
(e) None of these
3. Some part of a certain group M is related to
another group T while a group P is also partly
related to T but M and P are completely (e)
independent of each other. Which of the
following can be its possible representation? 5. Monkey, Wolves, Animals
6. Mammals, Cows, Parrots
7. Rajasthan, Himachal, India
(a) (b)
8. Authors, Teachers, Women
9. Girls, Students, Players
Directions  (Q  10-14):  Given  below  are  five
possible Venn Diagrams. In each case, mark the
(c) (d) one  you  feel  appropriate  description  of  the
three listed classes.
(e) None of these
4. If two groups X and M are partly releated to
each other while a third group L completely
r e pr es ent s X and M, t hen whi ch of t he (a) (b)
following can be its diagram?
SSC
(d) Horse, Mare, Male
18. (a) Oxygen, Air, Water
(c) (d) (b) Director, Engineer, Musician
(c) Fruit, Pear, Grass
(d) Apple, Orange, Mango
(e) 19. (a) Women, Doctor, Maid
(b) Mars, Moon, Star
10. Peon, Doctor, People (c) Swimmer, Carpenter, Singer
(d) Professor, Scholar, Politician
11. Musicians, Instrumentalists, Guitarists
12. Watermelon, Brinjals, Fruits Directions  (Q  20  -  23):  Study  the  following
diagram to answer the questions asked.
13. History, Ancient India, Medieval India
14. Mountains, Forests, Earth
D i re c t i o n s   ( Q   1 5 - 1 9 ) :   E a c h   o f   t h e   f o u r
alternatives in each of the following questions,
three alternatives are such that the three words
in each are related among themselves in one of
the five ways represented by (i), (ii), (iii), (iv)
and (v) below while none of these relationships
is applicable to the remaining alternative. That
is your answer.
20. Find out the square of the number that lies
inside all the figures.
(i) (ii) (a) 4 (b) 16
(c) 64 (d) 9
(e) None of these
21. What is the sum of the numbers that lie inside
(iii) (iv) any two figures?
(a) 15 (b) 18
(c) 25 (d) 9
(e) None of these
22. H ow ma ny nu mbe r s l i e o nl y i n si de t h e
(v) triangle?
(a) 9 (b) 2
15. (a) Book, Page, Paragraph (c) 5 (d) 1
(b) Girl, Athlete, Singer (e) None of these
(c) Bed, Ward, Hospital 23. Find the product of the numbers lying only
inside circle and only inside rectangle.
(d) Copper, Zink, Iron
(a) 75 (b) 25
16. (a) Tiger, Animal, Carnivourous
(c) 60 (d) 35
(b) Mouth, Ear, Body
(e) None of these
(c) Bed, Ward, Nurse
24. A result of a survery of 1000 persons with
(d) Cat, Dog, Hen
r e s p e c t t o t h e i r kn o w l e d ge o f H i s t o r y,
17. (a) Red, Blue, Colour Economics and Social Science is given below.
(b) Cow, Mammal, Animal On the basis of this survey, find the ratio of
(c) Merchant, Cloth, colour those who know all the three subjects to those
SSC
who do not know Social Science. students.

(a) (b)
27. The number of students taking any three of
the above subjects are:
(c) (d)
(a) 63 (b) 65
(e) None of these (c) 68 (d) 67
Directions (Q 25 - 26): The following questions (e) None of these
are based on the diagram given below: 28. Find the total number of students taking Hindi
or Math or Sociology.
(a) 190 (b) 196
(c) 183 (d) 185
(e) None of these
29. The number of students taking both Hindi and
English among other subjects are
(a) 63 (b) 68
• Square = Clerks
(c) 88 (d) 78
• Rectanle = Government Employees
(e) None of these
• Triangle = Urban People 30. Which subject has been taken by the largest
• Circle - Graduates number of students?
25. Take out the correct statement (a) Maths (b) Sociology
(a) Some clerks are government employees. (c) English (d) Hindi
(b) No clerks is urban. (e) None of these
(c) All graduates are urban Directions  (Q  31  -  35):  Each  of  the  quesions
(d) All graduates are government employees given below contains three groups of items. You
(e) None of these h a v e   t o   c h o o s e   f ro m   t h e   f o l l o w i n g   f i v e
26. Select the correct statement from the options numbered diagrams, the diagram that depicts
given below. the correct relationship among the three groups
of items in each of the question.
(a) All government employees are clerks.
(b) Some government employees are graduates
as well as clerks. (a) (b)
(c) All government employees are graduates.
(d) All clerks are government employees but not
graduates.
(e) None of these
Directions (Q 27 - 30): The following diagram
represents  the  different  subject  taken  by  the
SSC
diagram carefully to answer the questions that
follow.
(c) (d)

(e)

31. Classical Dance, Bharatnatayam, Kuchipuri


32. Rice, Fruit, Birds
39. According to the above diagram which one of
33. Asia, Pakistan, Islamabad the following statement is true?
34. Parrots, Birds, Lions (a) All philosophers are magicians but no
35. Sisters, Fatty, Brothers magician is a historian.
Directions  (Q  36  -38):  Study  the  diagaram (b) All magicians are either historians or
given below to answer the questions asked on philosophers.
the basis of that. (c) Some philosophers are historians and some
magicians are philosophers.
(d) Some historians, who are philosophers are
magicians too.
(e) None of these
40. According to the above diagram which one of
= Men
the following statements is not true?
= Women (a) Some magicians are philosophers
(b) No magicians is historian.
= Educated
(c) Magicians, who are philosophers are
= Employed historian also.
(d) Some philosophers are magicians
36. What does number 8 depicts?
(e) None of these
(a) Educated unemployed women.
41. The following diagram represents the students
(b) Uneducated unemployed women.
who are singers, dancers and poets.
(c) Uneducated employed women.
(d) Educated employed women.
(e) None of these
37. Uneducated employed men are depicted by the
number
(a) 4 (b) 3
(c) 2 (d) 1
(e) None of these
38. Which number depicts educated unemployed Study the diagram and identify the region
men? which represents the students who are both
(a) 2 (b) 1 poets and singers but not dancers.
(c) 4 (d) 3 (a) B + C + D (b) M
(e) None of these (c) Q + R (d) R + X
Di rect ions  ( Q  39  -  40) :  See  t he  f oll ow i ng (e) None of these
SSC
Directions  (Q  42  -  45):  See  the  diagram  and 44. Find out the number of families having all the
answer the questions given below: four things mentioned in the diagram given
above
(a) 25 (b) 50
(c) 20 (d) 15
(e) None of these
45. W h i c h n u mb e r i s c o mmo n t o a l l t h e
geometrical figures in the diagram given
below?

42. Find out the number of families having cars


only.
(a) 30 (b) 50 (a) 2 (b) 5
(c) 40 (d) 15 (c) 4 (d) 5
(e) None of these (e) None of these
43. Find out number of families which have bikes?
(a) 190 (b) 187
(c) 191 (d) 188
(e) None of these

Answers
1. c 2. c 3. b 4. b 5. c 6. d
7. c 8. e 9. e 10. e 11. e 12. a
13. c 14. d 15. d 16. d 17. c 18. d
19. b 20. b 21. a 22. d 23. c 24. a
25. a 26. b 27. e 28. c 29. a 30. b
31. a 32. b 33. d 34. e 35. c 36. a
37. b 38. c 39. c 40. c 41. b 42. e
43. d 44. c 45. a
SSC
15. SYLLOGISM
Syl logi sm i s a Greek wor d t hat does mean other words a categorical proposition has no
'i nf er ence' or 'deduct i on'. T he pr obl ems of condition attached with it and it makes direct
syllogism is based on two parts: assertion. It is different from no-categorical
(1) Proposition/ propositions proposition which is in the format
(2) Conclusion/ conclusions drawn from given "If M then P"
proposition/ propositions. Types of categorical proposition
What is a Proposition?
Just consider the sentences given below:-
(i) "All are

(ii) "No is

(iii) " Some are

(iv) "Some are not Therefore, it is clear, that universal propositions


either completely include the subject (A type) or
completely exclude it (E type). On the other hand,
All the sentences mentioned above give a particular propositions either only partly include
relation between subject & predicate. Here, it is the subject (I type) or only partly exclude the
clear from the sentences that a subject is the part subject (O type).
of sentence something is said about, while a
N o w w e c a n s u mma r i ze t h e f o u r t yp e s o f
predicate is the term in a sentence which is related
proposition to be used while solving the problems
to the subject.
of syllogism:-
Now, let us define the proposition:
Format Type
A Proposition is a sentence that makes a
All M are P - A
statement giving a relation between two terms. It
has three parts:- No M are P - E
(a) The subject Some M are P - I
(b) The predicate Some M are not P - O
(c) The relation between subject & predicate How to Identify Hidden Proposition?
What is a Categorical Proposition? (i) A Type:-
Let us see the sentences given below: Apart from 'all' it starts with every, each and any.
"All M are P" Example:-
"No M are P" Every girl is beautiful
"Some M are P" [All girl are beautiful]
"Some M are not P" Each of them is healthy
What we notice in all above mentioned sentences [All (of them) are healthy]
t hat they are condi ti on fr ee. These type of Any one could kill the lion.
sentences are called categorical propositions. In [All can kill the lion]
SSC
further, let us see the sentences given below:- (III) I Type:
Apart from some it also start with word such no
should be awarded often frequently, almost, generally, mostly, a few,
most etc.
Example:-
is a actor Almost all the girls are beautiful.
[Some girls are beautiful]
Most of the garments are handmade.
Thus a positive sentence with a particular person [Some of the garments are handmade.]
as its subject is A type. Usually girls are beautiful.
Also, a sentence in the following format is A [Some girls are beautiful]
type:- A few money are left in my wallet.
"All girls except are healthy." [Some money are left in my wallet.]
Further, let us see the sentences given below:-
Few girls are not studious.
(ii) E Type:- [Some girls are studious.]
Apart from 'no' this type of propositions starts Rarely is a girl not beautiful.
form 'no one', 'none', 'not a single' etc. [Some girls are beautiful]
Example:- Seldom are women not housewife.
No one (student) is studious. [Some women are housewife.]
[No student is studious] It is clear from the above examples that negative
sentences beginning with words like 'few', 'rarely',
None of the girl is beautiful 'seldom', etc. (also 'hardly', 'scarcely', 'little' etc.)
[No girl is beautiful] are to be reduced to I type.
Not a single girl is healthy. Just see the other formats given below:-
[No girl is healthy]
Further, let us see the sentences given below:-
does not deserve
All girls except are beautiful.
[Some girls are beautiful]
is not a actor

Thus, a negative sentence with a particular person


on its subject is E type proposition. All girls except have passed
Also, sentences in following formats are E type:-
[Some girls have passed]
"No student except has failed."
Therefore, a positive proposition with an indefi-
nite exception is reduced to I type.
"Is there any truth left in the world". (iv) O Type:-
[No truth is left in the world.] Apart from " some....not" this type of statements
start with words like 'all', 'every', 'any', 'each', etc.
SSC
Example:- T h e r e f o r e , a n e ga t i ve p r o p o si t i o n w i t h a n
All girls are not beautiful. indefinite exception, is reduced to O type.
[Some girls are not beautiful] Identifying Exclusive Propositions
Every boy is not present. Such propositions start with 'only', 'alone', 'none
[Some boys are not present] but', etc. and they can be reduced to either A or E
or I format.
Further, let us see the following sentences:-
E xa mp l e : O n l y gr a d u a t e a r e p r o b a t i o n a r y
Poor are usually not healthy.
officers.
[Some poor are not healthy]
No non-graduate is probationary officer (E
Almost all the girls are not beautiful. type)
[Some girls are not beautiful] All probationary officers are graduate (A
Most of the garments are not handmade. type)
[Some of the garments are not handmade] Some graduates are probationary officers (I
Girls are not frequently short tempered. type)
[Some girls are not short tempered] G e n e r a l f o r ma t o f s e n t e n c e s gi ve n i n t h e
Now, i t i s cl ear f r om t he above ment i oned examinations:-
examples that negative propositions with words All M are p (A type)
such as 'almost', 'frequently', 'most', 'mostly', 'a
few', 'generally', etc. are to be reduced to the O No M are P (E type)
type propositions.
Again positive propositions starting with
Some M are P (I type)
words likes 'few', 'scarcely', 'rarely', 'little',
'seldom' etc. are said to be O type.
Example:- Some M are not P (O type)
Seldom are women jealous Note:-
[Some women are not jealous] General format given above are frequently asked
formats in the examinations. But students must
Few girls are beautiful.
be ready for other hidden formats of A, E, I and
[Some girls are beautiful] O types of propositions as problems in hidden
Rarely is a wealthy person worried. formats can also be given in question papers.
[Some wealthy person not worried] Conversion of Propositions
Also see the following formats:- Before solving the problems of syllogism it is
must to know the conversion rules of all A, E, O
and I types of proposition:_
Conversion of A type:-

No girls except are beautiful.

[Some girls are not beautiful]


All are (A type)
after conversion it becomes

No women except are housewife. "Some are (I type)

[Some women are not housewife] Therefore, it is clear that A type of propositions
gets converted into I type.
SSC
Conversion of E type: of subject and predicate does not place. This
method is also used for drawing immediate
inference.
"No are (E type) Example 1: All M are P. (A)

after conversion it becomes Some M are P. (I)


Example 2: No M are P. (E)
Some M are P. (O)
"No are (E type) Rule to Draw Conclusion
Therefore, E gets converted into E A conclusion drawn from a single proposition is
j u s t a c o n ve r s i o n o r i m p l i c a t i o n o f t h a t
Conversion of I type:
proposition while to get conclusion from two
propositions a certain table is used that tells us
what type of conclusion ( in form of proposition)
"Some are (I type) we get out of two propositions . To understand it
after conversion it becomes let us see the following conclusion table:-
Proposition I  Proposition II   Conclusion
A A A
"Some are (I type)
A E E
Therefore, I gets converted into I.
Conversion of O type:
E A (O) R
O type of proposition can't be converted.
Note:-
E I (O) R
In each conversion, subject becomes predicate and
medicate become subject.
I A I
In f a c t , c o n ve r s i o n i s a n i mme d i a t e
inference that is drawn from two propositions are
called mediate inference. I E O
Now we can make a short type table of conversion
to remember. Note:-
(a) Apart from above 6 pairs of propositions, no
Conversion Table other pair will give any conclusion.
Type of proposition      Gets converted into
(b) The conclusion drawn out of two propositions
A -------------- I is itself a proposition and its subject is the sub-
ject of the 1st statement while its predicate is the
predicate of the 2nd statement. The common term
E -------------- E get disappeared.

I -------------- I (c) (O) R does mean that the conclusion is O type


but is in reverse order. In this case, the subject of
O -------------- Never gets converted the inference or conclusion is the predicate of the
Implication 2nd proposition and the predicate of the conclu-
Through implication A is converted into I and E sion is the subject of the 1st sentence or state-
is converted into O but in this case interchange ment.
SSC
(d) The conclusion table gives correct conclusions change the order of sentences. In another words
or inference if and only if the two proposition we put I sentence in place of II and II in place of
are aligned properly. I:
What is Aligning?
Let us see the following examples:- II. Some cats are .
Ex.1.:
I. Some are chairs.
Statements: I. All are beautiful.
Let us consider another pair of statements
II. Some are Indian I. All bats are chair.
Ex.2.: II. All bats are cats.
Statements: I. No is chair.. Then how to align it? Infact, in such cases
we do alignment in two ways:-
II. Some tabes are . (a) by converting statement I as
Ex.3.: I. Some chair are .
Statements: I. Some women are . II. All are cats.
II. No is chair.. and
In all the above mentioned example, we notice (b) by changing the orders of the sentences and
that in two statements of every example, there is then converting the statement II.
a common term. In example 1 the word 'girl' is Now 1st change the order as:
common; in example 2 the word 'pen' is common; II. All bats are cats.
while in example 3 the word 'men' is common. I. All bats are chair.
N o w, t h e a l i gn i n g o f t h e t w o s t a t e me n t s again we do conversion for II and the aligned pair
(propositions) does mean that the pair of takes the form as
statements must be written in such a way that the
common term is the predicate of the 1st sentence II. Some cats are .
and the subject of the 2nd.
I. All are chair..

Just think over the following examples:- Therefor, as per the requirement and nature of the
sentence the alignment is done.
Statements: I. Some girls are . (i) only by changing the order of sentences.
or
II. All are tall.
(ii) only by converting one of the sentence.
Hence, the common term cute is the predicate of
or
the I statement and subject of the 2nd statement.
by changing the order of the statements and then
Therefore, the two statements (I & II) are properly
converting one of the sentences.
aligned.
I E A  Rule:-
But see another example
Alignment must be done in I E A order. It does
Statements: I. Some are chairs. mean that if the two statements are I & E then the
conversion must be done for I and for E & A, it
II. Some cats are . will be done for E.
Her e, t h e sen t ence s ar e not a l i gne d as t he After discussing all the minute things
predicate of the 1st statement is not the subject about this chapter, now we have come at the
of the 2nd. position of solving the problems of syllogism.
Then how to align it? In such type of cases, we How to Solve Problems?
SSC
This method has two main steps.
(a) Aligning the pair of sentences. (i) Some cats are rats. 
 I - O pair
(b) Using conclusion table to draw conclusion. (ii) Some cats are not rats.
For example:- (i) All cats are rats. 
Statements: I. All rats are cats.  A - O pair
(ii) Some cats are not rats.
II. All rats are men.
(i) Some cats are rats. 
When aligned it takes the form as  I - E pair
(ii) No cats are rats. 
I. Some cats are . [I type]
A pa r t f r o m I-O, A -O a nd I-E p ai r t h e t w o
II. All are men. [A type] sentences must have same object and predicate
Now we use the conclusion table given in the as are the above mentioned pairs. For these pairs
chapter that says we write the form
I + A = I type of conclusion. Either (i) or (ii) follows
Therefore, the drawn conclusion must be. For example, see the following format:-
"Some cats are men." Statements: I. Some dogs are cats.
It is clear that the conclusion drawn "Some cats II. Some cats are rats.
are men" is a mediate inference as it is the result Conclusions: (i) Some cats are dogs.
of t wo proposit ions. But in act ual probl em (ii) Some rats are cats.
immediate inferences are also given in conclusion (iii) All cats are rats.
part and that format is given below:- (iv) Some dogs are rats.
Statement: I. All rats are cats. (v) Some dogs are not rats.
II. All rats are men. Answer options:
Conclusion: (i) Some cats are men. (a) All follow
(ii) Some men are cats. (b) Only (i) follows
(iii) Some rats are cats. (c) Only (ii) and (iii) follow
(iv) Some cats are rats. (d) Either (iv) or (v) and (i) & (ii) follow
(v) Some rats are men. Here, option (D) is correct because conclusion (i)
(vi) Some men are rats. i s t he i mmedi at e i nf er ence (conver sion) of
Answer option: statements I while conclusion (ii) is the immediate
(a) Only (iii) follows inference of II. Conclusion (iv) & (v) make
(b) Only (i), (ii), and (iii) follow complementary pair of I-O type. Conclusion (iii)
(c) Only (iv) follows is not correct because I and II are I type of
(d) All follow statements and I + I does not give any conclusion.
(e) None of these Further, A type of conclusion cannot be find from
Here, the correct option is D. the immediate inference (conversion) of I type of
conclusion (i) follows because it is the mediate statements as I & II are.
inference of statement I & II. Conclusion (ii) is Now, the complete process of solving syllogism
the conversion of conclusion (i). Conclusion (iii) problems can be summarised as below:-
i s t he i mmedi at e i nf er ence (conver sion) of (A) 1st step is aligning the sentence.
statement I while conclusion (iv) is the conversion (B) 2nd step is using conclusion table.
o f c o n c l u s i o n ( i i i ) . C o n c l u s i o n ( v) i s t h e (C) 3rd step is checking immediate inference &
immediate inference (conversion) of statement II mediate inference.
whi l e c oncl usi o n ( vi ) i s t h e co nver si on of (D) 4th step is checking through the conversion
conclusion (v). of immediate inference & mediate inference.
Further, in some problems complementary pairs (E) checking the complementary pairs.
are also seen in the conclusion part in the forms
of sentences given below:-
SSC
Exercise
Directions (Q. 1-100): In each question below is given Conclusions: I. Some cows are not elephants
a statement followed by two conclusions numbered I II. Some elephants are not cows.
and II. You have to assume everything in the state- 8. Statements: All elephants are foxes.
ment to be true, then consider the two conclusions No fox is a hen.
together and decide which of them logically follows
Conclusions: I. Some foxes are elephants.
beyond a reasonable doubt from the information
given in the statement. II. Some foxes are not elephant.
Given answer 9. Statements: All books are pencils.
(a) If only conclusion I follows No pencil is a table
(b) If only conclusion II follows Conclusions: I. No book is a table,
(c) If either I or II follows II. Some books are table.
(d) If neither I nor II follows 10. Statements: All staplers are pens.
(e) If both I and II follow No pen is a book
1. Statements: All books are copies. Conclusions: I. Some pens are books
All copies are cakes. II. Some books are pens
Conclusion: I. All books are cakes. 11. Statements: No pen is a potato
II. All cakes are books. All potatoes are tomatoes
2. Statements: All girls are honest person. Conclusions: I. Some pens are tomatoes
All II. Some pens are not tomatoes.
honest persons are intelligent. 12. Statements: All potatoes are tomatoes.
Conclusions: I. Some girls are intelligent. No tomatoes are onions.
II. Some intelligent (persons) are Conclusions: I. No potatoes are onions.
girl. II. Some potatoes are onions.
3. Statements: All staplers are ball. 13. Statements: All skirts are shirts.
All balls are bat. All keys are skirts.
Conclusions: I. Some bats are ball. Conclusions: I. All keys are shirts.
II. Some staplers are ball. II. All shirts are keys.
4. Statements: All cakes are chocolates. 14. Statements: All plates are tumblers.
All chocolates are biscuits. All spoons are tumblers.
Conclusions: I. Some biscuits are cakes. Conclusions: I. Some plates are tumblers.
II. Some cakes are biscuits. II. Some tumblers are spoons.
5. Statements: All elephants are dog. 15. Statements: All plates are tumblers.
No dog is a cow. All spoons are tumblers.
Conclusions: I. No cow is a dog. Conclusions: I. Some plates are spoons.
II. No elephant is a cow. II. Some plates are not spoons.
6. Statement: All elephants are dog. 16. Statements: All plates are tumbler.
No dog is a cow. All spoons are tumbler.
Conclusions: I. No cows are elephants. Conclusions: I. All plates are spoon.
II. Some dogs are not cows. II. Some plates are not spoon.
7. Statements: All elephants are dog. 17. Statements: No room is house.
No dog is a cow. All houses are villages.
SSC
Conclusions: I. Some rooms are villages. All jeeps are scooters.
II. Some rooms are not villages. Conclusions: I. All cars are scooters.
18. Statements: No doctor is an engineer. II. All scooters are jeeps.
No engineer is a lecturer. 28. Statements: No car is a jeep.
Conclusions: I. Some doctors are lecturers. All jeeps are scooters.
II. Some doctors are not lectur- Conclusios: I. All cars are scooters.
ers. II. Some cars are not scooters.
19. Statements: No mango is an apple. 29. Statements: No auto-rickshaw is a scooter.
No apple is a guava. Some scooters are motorcyles.
Conclusions: I. Some mangoes are guavas. Conclusions: I. Some motorcycles are not
II. Some guavas are mangoes. auto-rickshaw.
20. Statements: No book is bat II. Some auto-rickshaw is not a
No book is cow. motorcycle.
Conclusions: I. No bat is a book. 30. Statements: No bus is a scooter.
II. No cow is a book. Some scooters are jeeps.
21. Statements: No book is a rocket. Conclusions: I. Some buses are jeeps.
No bus is a rocket. II. Some buses are not jeeps.
Conclusions: I. Some books are bus. 31. Statements: No helicopter is an aeroplane.
II. All books are bus. Some aeroplanes are ships.
22. Statements: No bus is a jeep. Conclusions: I. Some ships are helicopters.
No car is a jeep. II. Some ships are not helicop-
Conclusions: I. Some buses are not car. ters.
II. Some buses are car. 32. Statemetns: No helicopters is an
aeroplane.
23. Statements: No book is a pen.
Some aeroplanes are ship.
All pens are white dress.
Conclusions: I. Some aeroplanes are helicop-
Conlusions: I. Some white dress are pen.
ters.
II. No pen is a book.
II. Some aeroplanes are not heli-
24. Statemetns: No book is a pen. copters.
All pens are white dress. 33. Statemetns: Some houses are city.
Conclusions: I. All white dresses are book All cities are homes.
II. All white dresses are pen Conclusions: I. Some houses are homes.
25. Statements: No bulb is a television II. Some homes are houses.
All televisions are radios 34. Statements: Some dogs are cats.
Conclusions: I. Some radios are not bulbs All cats are elephants.
II. Some televisions are not bulbs Conclusions: I. Some cats are dogs.
26. Statements: No rickshaw is a bicycle. II. Some dogs are not elephants.
All bicycles are motorcycles. 35. Statements: Some roads are dams.
Conclusions: I. Some rickshaws are not mo- All dams are bridges.
torcycles.
Conclusions: I. Some bridges are not dams.
II. Some motorcycles are not
II. Some dams are not roads.
rickshaws.
36. Statements: Some staplers are fevicol
tubes.
27. Statements: No car is a jeep.
SSC
All fevicol tubes are pens. Conclusions: I. All dogs are elephants.
Conclusions: I. All pens are fevicol tubes II. All elephants are dogs.
II. All pens are not fevicol tubes. 46. Statements: Only books are pens.
37. Statements: All charis are tables. All books are dusters.
Some tables are benches. Conclusions: I. Some books are pens.
Conclusions: I. Some chairs are benches. II. Some pens are books.
II. Some benches are chairs. 47. Statements: Only toys are dogs.
38. Statements: All buckets are baskets. All toys are oxen.
Some baskets are jars. Conclusions: I. Some dogs are oxen.
Conclusions: I. Some jars are baskets. II. Some oxen are dogs.
II. Some jars are not baskets. 48. Statements: Only toys are dogs.
39. Statements: All girls are dishonest per- Only oxen are toys.
sons. Conclusions: I. Only dogs are oxen.
Some dishonest persons are II. Only oxen are dogs.
responsible persons. 49. Statements: Only books are cars.
Conclusions: I. All girls are responsible per- Only buses are books.
sons.
Conclusions: I. Only buses are cars.
II. All responsible persons are
II. Only cars are buses.
girl
50. Statements: Only jokers are stars.
40. Statements: Some boys are honest persons.
Only actors are jokers.
No honest person is a student.
Conclusions: I. Only actors are stars.
Conclusions: I. Some boys are not honest per-
son. II. Only stars are actors.
II. Some boys are not students. 51. Statement:
41. Statements: Some doctors are quack. (a) Some rats are cats.
No quack is an engineer. (b) All cats are dogs.
Conclusions: I. Some engineers are doctors. (c) No dog is a horse.
II. Some engineers are not doc- (d) All horses are bulls.
tors. Conclusion:
42. Statements: No book is a pencil. I. No dogs is a bull.
Some pencils are not copies. II. Some dogs are bulls.
Conclusions: I. Some pencils are not Books. III. Some dogs are rats.
II. Some copies are not pencils. IV. Some bulls are rats.
43. Statements: No book is a pencil. (1) None follows
Some pencils are not copies. (2) All follow
Conclusion: I. No copies are pencils. (3) Only either I or II and III follow
II. No copy is a book (4) Only I and III follow
44. Statements: No book. is pencil. (5) Only I, II and III follow
Some pencils are not copies. 52. Statements:
Conclusions: I. No pencil is book. (a) Some umbrellas are sticks.
II. Some pencils are copies. (b) Some sticks are balls.
45. Statements: Only cats are dogs. (c) Some balls are bats.
All cats are elephants. (d) All bats are guns.
SSC
Conclusion: (b) Some CDs are keyboards.
I. Some balls are umbrellas. (c) Some keyboards are monitors.
II. Some guns are bats. (d) Some computers are floppies.
III. Some sticks are guns. Conclusion:
IV. Some balls are guns. I. Some monitors are floppies.
(1) Only I, II and IV follow II. No floppy is a monitor.
(2) Only II and IV follow III. Some computers are CDs.
(3) Only II, III and IV follow IV. Some keyboards are floppies.
(4) Only I, II and III follow (1) Only I follows
(5) None of these (2) Only I, III and IV follow
53. Statements: (3) Only II follow
(a) Some books are notebooks. (4) Only either I or II follow
(b) Some notebooks are dictionaries (5) None of these
(c) Some dictionaries are files. 56. Statements:
(d) All files are envelopes. (a) Some tents are buildings.
Conclusion: (b) Some buildings are chairs.
I. Some envelopes are notebooks. (c) Some chairs are windows.
II. Some files are books. Conclusion:
III. Some books are dictionaries. I. Some windows are buildings.
IV. No book is an envelope. II. Some windows are tents.
(1) None follow III. Some chairs are tents.
(2) Only I, II and III follow (1) None follows
(3) Only II and III follow (2) Only I and II follow
(4) Only III and IV follow (3) Only II and III follow
(5) None of these (4) Only I and III follow
54. Statements: (5) All follow.
(a) Some keys are chains. 57. Statements:
(b) All chains are locks. (a) All tables are boards.
(c) All locks are numbers. (b) All pens are boards.
(d) No number is a digit. (c) All boards are papers.
Conclusion: Conclusion:
I. Some keys are numbers. I. Some pens are tables.
II. All chains are numbers. II. Some papers are pens.
III. Some locks are keys. III. No pen is a table.
IV. No digit is a chain. (1) All follow
(1) Only I and II follow (2) Only either I or III follows
(2) Only I, II and III follow (3) Only II follows
(3) Only II and III follow (4) Only either I or III and II follows
(4) Only II, III and IV follow (5) None of these
(5) All follow 58. Statements:
55. Statements: (a) All chains are cycles.
(a) Some floppies are CDs. (b) Some cycles are wheels.
(c) All wheels are mirrors.
SSC
Conclusion: (1) Only I follows
I. Some cycles are mirrors. (2) Only II follows
II. Some wheels are chains. (3) Only III follows
III. Some mirrors are chains. (4) All follow
(1) None follows (5) None of these
(2) Only I follows 62. Statements:
(3) Only II follows (a) Some cameras are binoculars.
(4) Only III follows (b) All binoculars are spoons.
(5) All follow (c) Some spoons are cups.
59. Statements: Conclusion:
(a) Some boxes are hammers. I. Some cups are binoculars.
(b) Some hammers are beads. II. Some spoons are cameras.
(c) All beads are rings. III. Some cups are cameras.
Conclusion: (1) Only I follows
I. Some rings are hammers. (2) Only II follows
II. Some hammers are boxes. (3) Only III follows
III. Some rings are beads. (4) Only either I or III follows
(1) None follows (5) None of these.
(2) Only I follows 63. Statements:
(3) Only II and III follows (a) All books are pins.
(4) Only I and III follows (b) Some pins are tablets.
(5) All follows (c) All tablets are needles.
60. Statements: (d) Some needles are threads.
(a) All hotels are airports. Conclusion:
(b) All airports are belts. I. Some needles are pins.
(c) Some belts are hooks. II. Some pins are books.
Conclusion: III. Some threads are needles.
I. Some belts are hotels. IV. Some needles are tablets.
II. Some airports are hotels. (1) None follows
III. Some hooks are belts. (2) Only I and II follow
(1) Only I and II follows (3) Only I, II and III follow
(2) Only I and III follows (4) Only II, III and IV follow
(3) Only II and III follows (5) All follows
(4) Only III follows 64. Statements:
(5) All follow (a) All jungles are bins.
61. Statements: (b) All bins are petals.
(a) All jackets are trousers. (c) No petal is root.
(b) No trouser is a shirt. (d) All roots are flowers.
(c) Some shirts are caps. Conclusion:
Conclusion: I. No flowers is bin.
I. Some caps are jackets. II. No jungle is root.
II. Some shirts are jackets. III. All jungles are petals.
III. Some trousers are jackets.
SSC
IV. All flowers are roots. I. Some tubes are beads.
(1) Only I and II follow II. Some cycles are houses.
(2) Only II and III follow III. No beads is tube.
(3) Only I, II and III follow IV. Some rains are cycles.
(4) All follow (1) Only I follow
(5) None of these (2) Only either I or III follows
65. Statements: (3) Only I and II follow
(a) All poles are fans. (4) Only either I or III and IV follow
(b) All fans are stands. (5) None of these
(c) Some stands are pens. 68. Statements:
(d) Some pens are boxes. (a) All clouds are storms.
Conclusion: (b) Some storms are cyclones.
I. Some boxes are poles. (c) All cyclones are thunders.
II. Some fans are boxes. (d) Some thunders are cyclones.
III. Some pens are poles. Conclusion:
IV. Some pens are fans. I. Some lightning are cyclones.
(1) None follow II. No lightning is a cyclone.
(2) Only I and II follow III. Some cyclones are clouds.
(3) Only II and IV follow (1) Only I follows
(4) Only III and IV follow (2) Only II follows
(5) Only I, II and IV follow (3) Only III follows
66. Statements: (4) Only either I or II follows
(a) Some scales are weights. (5) None of these
(b) All weights are metals. 69. Statements:
(c) Some metals are rings. (a) Some pins are needles.
(d) All rings are bands. (b) Some needles are handles.
Conclusion: (c) Some handles are locks.
I. Some bands are scales. (d) Some locks are keys.
II. Some weights are bands. Conclusion:
III. Some rings are scales. I. Some keys are handles.
IV. Some metals are scales. II. Some handles are pins.
(1) Only I and III follow III. Some pins are keys.
(2) Only I and II follow (1) None follows
(3) Only II and III follow (2) Only I and II follow
(4) Only II and IV follow (3) Only II and III follow
(5) None of these (4) Only I and III follow
67. Statements: (5) All follow
(a) Some houses are beads. 70. Statements:
(b) Some beads are cycles. (a) All hills are mountains.
(c) Some cycles are tubes. (b) All mountains are dams.
(d) Some tubes are rains. (c) Some dams are rivers.
Conclusion: (d) All rivers are lakes.
SSC
Conclusion: I. Some readers are teachers.
I. Some hills are lakes. II. Some rectors are teachers.
II. Some dams are lakes. III. All rectors are readers.
III. Some dams are hills. (1) None follows
(1) Only I and II follow (2) Only I follows
(2) Only II and III follow (3) Only II follows
(3) Only I and III follow (4) Only III follow
(4) All follow (5) All follow
(5) None of these 74. Statements:
71. Statements: (a) Some papayas are guavas.
(a) Some receipts are challans. (b) Some guavas are carrots.
(b) Some challans are papers. (c) Some carrots are mangoes.
(c) Some papers are books. Conclusion:
(d) All books are files. I. Some mangoes are papayas.
Conclusion: II. Some carrots are papayas.
I. Some papers are files. III. No papayas is a mango.
II. Some books are receipts. (1) Only either I or II follows
III. No book is a receipt. (2) Only either I or III follows
(1) Only I follow (3) Only II follows
(2) Only I and III follow (4) Only III follows
(3) Only I and either II or III follow (5) None of these
(4) Only I and III follow 75. Statements:
(5) None of these (a) All bottles are glasses.
72. Statements: (b) All drums are bottles.
(a) All bottles are jars. (c) Some cups are bottles.
(b) All jars are containers. Conclusion:
(c) All containers are jars. I. Some glasses are cups.
(d) Some lids are jars. II. All drums are glasses.
Conclusion: III. Some bottles are drums.
I. All bottles are lids. (1) None follow
II. All containers are jars. (2) Only I and II follow
III. Some lids are jars. (3) Only II and III follow
(1) Only I and II follow (4) Only I and III follow
(2) Only II and III follow (5) All follow
(3) Only I and III follow 76. Statements:
(4) None follow (a) All tablas are sitars.
(5) All follow (b) All sitars are harmoniums.
73. Statements: (c) All harmoniums are violins.
(a) Some teachers are professors. Conclusion:
(b) Some professors are readers. I. Some violins are tablas.
(c) All readers are rectors. II. Some violins are sitars.
Conclusion: III. Some harmoniums are sitars.
SSC
(1) All follow (4) Only III follows
(2) Only II follow (5) None of these
(3) Only I follows 80. Statements:
(4) Only I and II follow (a) All dolls are windows.
(5) None follows (b) All bottles are windows.
77. Statements: (c) All cars are bottles.
(a) Some stations are ports. Conclusion:
(b) All shops are stores. I. All cars are windows.
(c) No port is a store. II. Some cars are dolls.
Conclusion: III. Some windows are cars.
I. Some station are shops. (1) Only I and II follow
II. Some stations are store. (2) Only II and III follow
III. No shop is a port. (3) Only I and III follow
(1) Only I follows (4) All follow
(2) Only II follows (5) None of these
(3) Only III follows 81. Statements:
(4) None follow (a) Some benches are beads.
(5) None of these (b) All beads are flowers.
78. Statements : (c) No tree is a flower.
(a) All halls are tyres. Conclusion:
(b) Some tyres are wheels. I. Some trees are benches.
(c) All wheels are cars. II. Some trees are beads.
Conclusion: III. No trees is a bead.
I. Some cars are wheels. (1) Only I follows
II. Some cars are tyres. (2) Only either I or III follows
III. Some wheels are halls. (3) Only either II or III follows
(1) None follows (4) Only III follows
(2) Only I follows (5) None of these
(3) Only I and II follow 82. Statements:
(4) Only III follows (a) All breads are tables.
(5) Only II and III follow (b) Some tables are brushes.
79. Statements: (c) All brushes are paints.
(a) Some blades are hammers. Conclusion:
(b) Some hammers are knives. I. Some paints are breads.
(c) Some knives are axes. II. Some brushes are breads.
Conclusion: III. Some paints are tables.
I. Some axes are hammers. (1) None follows
II. Some knives are blades. (2) Only II follows
III. Some axes are blades. (3) Only III follows
(1) None follows (4) Only I and II follow
(2) Only I follows (5) Only II and III follow
(3) Only II follows
SSC
83. Statements: 86. Statements:
(a) Some picture are frames. (a) Some caps are umbrellas.
(b) Some frames are idols. (b) Some umbrellas are raincoats.
(c) All idols are curtains. (c) All raincoats are trousers.
Conclusion: (d) All trousers are jackets.
I. Some curtains are pictures. Conclusion:
II. Some curtains are frames. I. Some raincoats are caps.
III. Some idols are frames. II. Some trousers are umbrellas.
(1) Only I and II follow III. All raincoats are umbrellas.
(2) Only II and III follow (1) None follows
(3) Only I and III follow (2) Only I & II follow
(4) All follow (3) Only II & III follow
(5) None of these (4) Only I & III follow
84. Statements: (5) None of these
(a) Some leaves are flowers. 87. Statements:
(b) No flower is fruit. (a) Some fans are coolers.
(c) Some fruits are branches. (b) Some coolers are machines.
(d) Some branches are stems. (c) Some machines are computers.
Conclusion: (d) All computers are televisions.
I. Some leaves are stems. Conclusion:
II. All leaves are either stems or fruits. I. Some televisions are machines.
III. All stems are either branches or fruits. II. Some machines are fans.
(1) Only I follows III. No machine is a fan.
(2) Only II & III follow (1) None follows
(3) Only III follows (2) Only I follows
(4) All follow (3) Only either II or III follows
(5) None of these (4) Only I and either II or III follow
85. Statements: (5) All follow
(a) All lions are tigers. 88. Statements:
(b) All tigers are leopards. (a) All keys are staplers.
(c) Some leopards are wolves. (b) All staples are blades.
(d) No wolf is elephant. (c) Some blades are erasers.
Conclusion: (d) Some erasers are sharpeners.
I. No elephant is lion. Conclusion:
II. Some wolves are lions. I. Some sharpeners are keys.
III. Some leopards are lions. II. All keys are blades.
(1) Only I follows III. Some erasers are keys.
(2) Only II follows (1) Only I & II follow
(3) Only III follows (2) Only I follow
(4) Only I & II follow (3) Only II follows
(5) All follow (4) All follow
(5) None of these
SSC
89. Statements: 92. Statements:
(a) All grapes are plums. (a) Some flats are houses.
(b) All plums are oranges. (b) Some houses are bungalows.
(c) Some oranges are apples. (c) No bungalow is hotel.
(d) Some apples are guavas. (d) All hotels are restaurants.
Conclusion: Conclusion:
I. Some oranges are grapes. I. No bungalow is restaurant.
II. Some guavas are oranges. II. Some houses are hotels.
III. Some apples are plums. III. Some restaurants are hotels.
(1) Only I follows (1) Only I follow
(2) Only I & II follow (2) Only III follows
(3) Only I & III follow (3) Only II & III follows
(4) All I, II & III follow (4) None follows
(5) None of these (5) None o these
90. Statements: 93. Statements:
(a) Some pets are dogs. (a) Some lemons are chillies.
(b) Some dogs are cats. (b) No chilly is brinjal.
(c) Some dogs are rats. (c) All brinjals are sweets.
(d) No rat is goat. (d) Some sweets are desserts.
Conclusion: Conclusion:
I. Some cats are rats. I. No chilly is sweet.
II. No cat is rat. II. Some lemon are desserts.
III. No goat is dog. III. Some brinjals are desserts.
(1) None follows (1) Only I follows
(2) Only I follows (2) Only II follows
(3) Only II follows (3) Only III follows
(4) Only III follows (4) None follows
(5) Only either I or II follows (5) None of these
91. Statements: 94. Statements:
(a) Some roses are jasmines. (a) Some pens are sticks.
(b) Some jasmines are lilies. (b) Some sticks are canes.
(c) All lilies are marigolds. (c) All canes are scales.
(d) All marigolds are sunflowers. (d) No scales is weight.
Conclusion: Conclusion:
I. All lilies are sunflowers. I. Some sticks are scales.
II. Some jasmines are marigolds. II. No stick is scale.
III. Some jasmines are sunflowers. III. No cane is weight.
(1) All follow (1) Only either I or II follows
(2) Only I follow (2) Only I & III follows
(3) Only II & III follow (3) Only either I or II and III follow
(4) Only I & II follow (4) All I, II and III follow
(5) None of these (5) None of these
SSC
95. Statements: 98. Statements:
(a) Some folders are boxes. (a) All locks are keys.
(b) Some boxes are bags. (b) All keys are doors.
(c) All bags are containers. (c) Some doors are windows.
(d) Some bags are sacks. (d) Some windows are floors.
Conclusion: Conclusion:
I. No folder is bag. I. Some keys are windows.
II. Some boxes are containers. II. No floor is door.
III. Some sacks are containers. III. No lock is window.
(1) Only I & II follow (1) None follows
(2) Only II & III follow (2) Only I follows
(3) Only I & III follow (3) Only I & II follow
(4) All follow (4) Only III follows
(5) None of these (5) None of these
96. Statements: 99. Statements:
(a) Some insects are pests. (a) All books are novels.
(b) All pests are birds. (b) Some novels are poems.
(c) No bird is plants. (c) Some poems are stories.
(d) All plants are animals. (d) No story is a movie.
Conclusion: Conclusion:
I. No animal is bird. I. Some books are poems.
II. Some insect are birds. II. Some movies are novels.
III. No pests are plants. III. No movies is a novel.
(1) Only I follows (1) Only I follows
(2) Only II follows (2) Only I & II follows
(3) Only III follows (3) Only either II or III follows
(4) Only II & III follow (4) None follows
(5) None of these (5) None of these
97. Statements: 100. Statements:
(a) Some paints are colours. (a) Some bags are suits.
(b) All colours are solutions. (b) All suits are trousers.
(c) Some solutions are liquids. (c) Some trousers are shirts.
(d) All liquid are solids. (d) All shirts are coats.
Conclusion: Conclusion:
I. Some paints are solution. I. Some trousers are coats.
II. Some colours are liquids. II. Some bags are trousers.
III. Some solution are solids. III. Some suits are coats.
(1) Only I & II follow (1) Only I & II follow
(2) Only II & III follow (2) Only I follows
(3) All I, II & III follow (3) Only II & III follows
(4) None follows (4) All follow
(5) None of these (5) None of these
SSC
Answers

1. A 35. D 51. 3 68. 4 85. 3


18. C
2. E 36. D 52. 2 69. 1 86. 5
19. D
3. E 37. D 53. 1 70. 2 87. 4
20. E
4. E 38. A 54. 2 71. 3 88. 3
21. D
5. E 39. D 55. 4 72. 2 89. 1
22. C
6. E 40. B 56. 1 73. 1 90. 5
23. E
7. E 41. C 57. 4 74. 2 91. 1
24. D
8. A 42. A 58. 2 75. 5 92. 2
25. E
9. A 43. D 59. 5 76. 1 93. 4
26. B
10. D 44. A 60. 5 77. 3 94. 2
27. D
11. C 45. A 61. 3 78. 3 95. 2
28. C
12. A 46. E 62. 2 79. 1 96. 4
29. A
13. A 47. E 63. 5 80. 3 97. 5
30. C
14. E 48. B 64. 2 81. 4 98. 1
31. B
15. C 49. A 65. 1 82. 3 99. 3
32. B
16. C 50. A 66. 5 83. 2 100. 1
33. E
17. C 67. 2 84. 5
34. A
16. NON-VERBAL REASONING
Non-verbal reasoning Actual Figure Water Image
Figure based reasoning is non verbal. It has no language (i)
at all. To solve non verbal problems one has to find out
the pattern of pictorial presentation in the given figure. (ii)
To get the more clear concept about non verbal Explanation:
reasoning. Let us see the types of problems coming
before you
Types of Problems (i)Actual Figure:
(A) Problems Based on Mirror Image
In a mirror image, left part of an object becomes right
part and right part becomes left part. Remember the
Water Image:
rule given below.
(L.H.S.) (R.H.S.)
Example: Clearly, top becomes bottom and bottom becomes top.
Actual Figure Mirror Image

(i) (ii) Actual Figure:

(ii)
Explanation:
Water Image:
(i) Actual Figure:

Clearly, top becomes bottom and bottom becomes top.


Mirror Image:
(C) Problems Based on Figure Series
Clearly, L.H.S. becomes R.H.S. and R.H.S. becomes A figure series is a sequence of many elements made
L.H.S. of figures. Let us discuss some examples that make the
(ii) Actual Figure: M i r r o r concept more clear about figure series.
Image: Ex. 1:

Clearly, L.H.S. becomes R.H.S. and R.H.S. becomes


L.H.S.
(B) Problems Based on Water Image.
In water image, the actual figure gets inverted vertically. Explanation: Figure 1 has two triangles.
In other words, in water image top becomes bottom Figure 2 has three triangles.
and bottom becomes top but there is no change in L.H.S. Figure 3 has four triangles.
and R.H.S. like mirror image. L.H.S. remains L.H.S.
Figure 4 has five triangles.
and R.H.S. remains R.H.S.
Clearly, every next figure has one more triangle than
Remember the rule given below.
the previous figure has.
Ex. 2:

Example:
Explanation: Figures 1 and 2 have triangles and 2nd
figure has two triangles more than the 1st figure has. Explanation: gets inverted in every next step.
Figures 3 and 4 have dark circles and 4th figure has 2 Ex. 8:
dark circles more than the 3rd figure has. Following
the same pattern figures 5 and 6 have arrows and 6th
figure has two arrows more than the 5th figure has.
Ex. 3:
Explanation: One + sign gets disappeared in every
Explanation: Figure next figure.
Ex. 9:
rotates one step anticlockwise in every next figure.
Remember

Explanation: A new element is added in every next


figure.

Ex. 10:
Ex. 4:
Explanation: One element gets inverted in every next
figure and the process starts from extreme right.
Note: The process may also start from extreme left.
Ex. 11:
Explanation: rotates one step clockwise in every
next figure.
Ex. 5:

Explanation: In every next figure, one element


becomes the mirror image of itself. In other words, in
every next figure, the L.H.S. of one element becomes
R.H.S. and R.H.S. becomes L.H.S. This process starts
Explanation: rotates half step anti clockwise in
from top.
every next figure. Note: The process may also start from bottom.
Ex. 6:

Ex. 12:

Explanation: rotates half step and one step Explanation: In 1st to 2nd figure, one element from
extreme left gets inverted vertically. In 2nd to 3rd figure,
alternately clockwise.
the one element from extreme right gets inverted
Ex. 7: vertically. From 3rd to 4th figure, the 2nd element from
left gets inverted and from 4th to 5th figure, the 2nd
element from right gets inverted. The same process
takes place in 6th and 7th figure.
Note: In other series, (series like example 11) same
process may take place in top and bottom sequence. (D) Problems Based on Analogy
Ex. 13: Students must have concept about analogy. Analogy
does mean similarity. When we talk of analogy in terms
of reasoning then we say that analogy is a similar
relationship in two or more things. In chapter 1, we
discussed about the analogy of words. Following the
Explanation: 8 moves one step clockwise in every same pattern, here, we are discussing about the analogy
next figure. of figures. Analogy of figures does mean similar
4 moves one step anticlockwise in every next figure. relationship in two or more figures. In the given
A new element comes at the centre in every next figure. problem, if two figures in a pair have certain
Ex. 14: relationship, then the same relationship will be
presented by the figures of another pair.
Ex. 1:

Explanation: In the
Explanation: , and move one step
anticlockwise in every next figure and there is no
change in element at the centre. 1st pair figure rotates one step clockwise and 2nd pair
Ex. 15: follow the same pattern. Hence, the given pairs of
figures exibit analogy.
Ex. 2:

Explanation: N, M, T and move one step clockwise


in every next step while the figure at the centre rotates
one step anticlockwise in every next figure. Explanation: In the 1st pair, the 2nd figure has 2
Note: In some other series, the figure at the centre may triangles more than the 1st figure has. In the same way
rotate clockwise. the 2nd figure in the 2nd pair has 2 triangles more than
the 1st figure has. Hence, the two pairs of figures exibit
analogy.

Exercise - 1
1. Choose the correct image of the figure when seen in
water from the rsponses given below:
Question Figure:
(a) (b) (c) (d)
× 3. Which of the answer figures is exactly the mirror im-
age of the given figure when the mirror is held at MN?
Answer Figures: Question Figure:

×
× ×
M N
(a) (b) (c) (d) Answer Figures:
2. Choose the correct water image of the figure from re-
sponses given below?
Question Figure:
(a) (b) (c) (d)
4. Which of the answer figures is exactly the mirror im-
age of the given figure when mirror is held on xy?
Answer Figures:
Question Figure: 8. If a mirror is placed on the northern side then which of
x the answer figures is the right image of the given ques-
tion figure.
Question Figure:
y
Answer Figures:

Answer Figures:

(a) (b) (c) (d)


Directions (5): In the following question, which of the an-
swer figures is exactly the mirror image of the question (a) (b) (c) (d)
figure when the mirror is held at XY? 9. Question Figure:
5. Question Figure: N
x
M
Answer Figures:
Y
Answer Figures:

(a) (b) (c) (d)


(a) (b) (c) (d) 10. Question Figure:
6. Which answer figure is the exact mirror image of the
given question figure when the mirror is held from the
right at PQ?
Question Figure: Answer Figures:
P

(a) (b) (c) (d)


Q
Answer Figures: 11. Question Figure:
N

(a) (b) (c) (d) Answer Figures:


7. If the mirror is placed on the line LM, then which of the . . .
answer figures is the right image of the given question
figure? (a) (b) (c) (d)
Question Figure:
12. Question Figure:
N
.

L M
Answer Figures: M
Answer Figures:
. . . .
(a) (b)

(a) (b) (c) (d)

(c) (d)
13. Which of the answer figures is exactly the mirror im- Answer Figures:
age of the question figure when the mirror is held at
MX.
Question Figure:
(a) (b) (c) (d)
18. Choose the correct water-image of the figure from re-
M X
sponses given below:
Answer Figures: Question Figure:

(a) (b) (c) (d) Answer Figures:

14. Which of the following figure is exactly the mirror im-


age of the question figure when the mirror is held at
AB? (a) (b) (c) (d)
Question Figure: 19. Which of the answer figures is exactly the mirror im-
age of the given figure when the mirror is held at XY?
Question Figure:
A B
X
Answer Figures:
Y
Answer Figures:
(a) (b) (c) (d)
15. Which of the answer figures is exactly figure, when
mirror is held at MN? (a) (b) (c) (d)
Question Figure: 20. Which of the Answer Figures is exactly the mirror im-
M
age of the given problem figure when one mirror is held
at, left side?
N Question Figure:
Answer Figures:

(a) (b) (c) (d) Answer Figures:


16. Which of the answer figures is exactly the mirror im-
age of the given figure when the mirror is held at MN?
Question Figure:
M (a) (b) (c) (d)
21. Which one of the Answer Figures is exactly the mirror
N image of the given figure when the mirror is held at
Answer Figures: AB?
Question Figure:

(a) (b) (c) (d) A B

17. Which of the answer figures is exactly the mirror im- Answer Figures:
age of the given figure which the mirror is held at MN?
Question Figure:
M

(a) (b) (c) (d)


N
22. Question Figure: 28. Question Figure:
A

W
IN
A B

Answer Figures:
B
Answer Figures:

(a) (b) (c) (d)


23. Question Figure: (a) (b) (c) (d)
29. Question Figure:
A
A B
Answer Figures:

B
(a) (b) (c) (d) Answer Figures:

24. Question Figure:

(a) (b) (c) (d)


P Q
Answer Figures: 30. Choose the correct water image of the question figure
out of the four answer figures given:
Question Figure:
(a) (b) (c) (d)
25. Question Figure:
Q Answer Figures:

P
Answer Figures:
(a) (b) (c) (d)
31. Which of the answer figures is exactly the mirror im-
(a) (b) (c) (d) age of the given figure, when the mirror is held at AB?
Question Figure:
26. Question Figure: A
Q

B
P Answer Figures:
Answer Figures:

(a) (b) (c) (d) (a) (b) (c) (d)

27. Question Figure: 32. Choose the correct mirror image of the figure, when
the mirror is held at AB:
Question Figure:
A
Answer Figures:
B
Answer Figures:

(a) (b) (c) (d)

(a) (b) (c) (d)


33. Choose the correct water image of the question figure Question Figure:
out of four answer figures given:
Question Figure:
M N
Answer Figures:
Answer Figures:

(a) (b) (c) (d)


(a) (b) (c) (d)
38. If mirror is placed on the line LM, then which of the
34. If a mirror is placed on the line LM, then which of the answer figures is the right image of the given question
answer figures is the right image of the given question figure?
figure? Question Figure:
Question Figure: L

M
Answer Figures:
Answer Figures:

(a) (b)

(a) (b) (c) (d)


(c) (d)
35. If a mirror is placed on the line MN, then which of the
answer figures is the right image of the given question
39. Which of the answer figures is exactly the mirror im-
figure?
age of the question figure, when the mirror is held on
Question Figure:
N
the line PQ?
Question Figure:
P

Q
M
Answer Figures: Answer Figures:

(a) (b) (c) (d) (a) (b) (c) (d)


36. If a mirror is placed on the line MN, then which of the 40. Which of the answer figure is exactly the mirror image
answer figures is the right image of the given question of the given figure, when the mirror is held on the line
figure? AB?
Question Figure: Question Figure:
M

A
N

B
Answer Figures: Answer Figures:

(a) (b)
(a) (b) (c) (d)
37. If a mirror is placed on the line MN, then which of the
(c) (d)
answer figures is the correct image of the given ques-
tion figure?
41. Which of the answer figures is exactly the mirror im- Answer Figures:
age of the question figure, when the mirror is held on
the line AB?
Question Figure:
(a) (b) (c) (d)
A
46. Which of the answer figures is exactly the mirror im-
age of the given figure, when the mirror is held on the
B
line MN?
Answer Figures: Question Figure:
M

(a) (b) (c) (d) N


Answer Figures:
42. Which of the answer figure is exactly the mirror image
of the given figure, when the mirror is held on the lines
AB?
Question Figure: (a) (b) (c) (d)
47. Which of the answer figure is exactly the mirror image
of the given figure, when the mirror is held on the line
Answer Figures: MN?
Question Figure:
M

(a) (b) (c) (d)


43. Which of the answer figure is exactly the mirror image
of the given figure, when the mirror is held on the line N
AB? Answer Figures:
Question Figure:
A

(a) (b) (c) (d)


B
Answer Figures: 48. Question Figure:
X

FROWNING
(a) (b) (c) (d)
Y
44. Which of the answer figure is exactly the mirror image Answer Figures:
of the given figure, when the mirror is held on the line
A B?
Question Figure:
R A
E
A
D B 49. Question Figure:
Answer Figures:

(a) (b) (c) (d)


45. Which of the answer figure is exactly the mirror image
of the given figure, when the mirror is held on the line Answer Figures:
A B?
Question Figure:
(a) (b)
A

B
53. Question Figure:
(c) (d) x

50. y
Answer Figures:

(a) (b) (c) (d)


54.
x
SEPTEMBER
51.
y
M

APRIL

55.

52. If a mirror is placed on the line MN, then which of the


answer words is the right image of the given word?

Answer
1. (b) 2. (b) 3. (c) 4. (b) 5. (c) 6. (b)
7. (c) 8. (b) 9. (b) 10. (c) 11. (c) 12. (c)
13. (c) 14. (d) 15. (c) 16. (a) 17. (a) 18. (a)
19. (a) 20. (c) 21. (a) 22. (d) 23. (a) 24. (c)
25. (a) 26. (a) 27. (d) 28. (d) 29. (d) 30. (d)
31. (a) 32. (a) 33. (d) 34. (a) 35. (c) 36. (d)
37. (b) 38. (b) 39. (b) 40. (a) 41. (a) 42. (b)
43. (d) 44. (d) 45. (d) 46. (a) 47. (a) 48. (a)
49. (a) 50. (d) 51. (c) 52. (d) 53. (d) 54. (d)
55. (c) 56. (c) 57. (b)
Exercise – 2
1. A circular paper is folded and punched as shown be-
low. How will it appear when opened?
Question Figure:
(a) (b) (c) (d)
6. Find out from the given answer figures how would the
Answer Figures: papers appear after folding punching and reopening?
Question Figure:

(a) (b) (c) (d)


Answer Figures:
Directions (2): A paper is folded as shown in the given fig-
ures and a hole is made. When opened how will it ap-
pear. Choose from the given responses.
2. Question Figure: (a) (b) (c) (d)
7. A square papers is folded in particular manner and
punches are made. When unfolded the paper appears
as given below. Find out the manner in which the paper
Answer Figures: was folded and punches were made.
Question Figure:

(a) (b) (c) (d)


3. A plece of circular paper is folded and cut as shown Answer Figures:
below. How will it appear when opened? Select your
responses from answer figures.

(a) (b) (c) (d)


8. Question Figure:
Answer Figures:

(a) (b) (c) (d) Answer Figures:


4. A triangular piece of paper is folded and punched as
shown in the question figure. How will it appear when
opened? Select your responses from answer figures. (a) (b) (c) (d)
9. Question Figure:

Answer Figures:

Answer Figures:
(a) (b) (c) (d)
5. A circular papers is folded and punched as shown be-
low. How will it appear when opened? (a) (b) (c) (d)
Question Figure:
10. Question Figure:

Answer Figures:
Answer Figures:
(a) (b) (c) (d)
(a) (b) (c) (d)
11. Question Figure:
16. Problem Figure:

Answer Figures:

Answer Figures:
(a) (b) (c) (d)
12. A piece of paper is folded and punched as shown be-
low. From the given responses, indicate how ot will
appear when opened. (a) (b) (c) (d)
Question Figure: 17. A piece of paper is folded and a cut is made a shown
below. From the given responses indicate how it will
appear when opened.
Problem Figure:
Answer Figures:

Answer Figures:
(a) (b) (c) (d)
13. A piece of paper is folded and punched as shown be-
low. From the given responeses indicate how it will (a) (b) (c) (d)
appear when opened.
Question Figure: 18. Question Figure:

Answer Figures:
Answer Figures:

(a) (b) (c) (d)


(a) (b) (c) (d)
19. Among the four answer figures, which figure can be
14. Among the four answer figures, which one can be
formed from the cutpieces given below in the question
formed from the cut out pieces given below in the ques-
figure?
tion figure?
Question Figure:
Question Figure:

Answer Figures:
Answer Figures:

(a) (b)

(a) (b) (c) (d)


15. Question Figure: (c) (d)

20. A rectangular piece of paper is folded and punched as


shown below in the question figures. From the given
Answer Figures: answer figures indicate how it will appear when opened
Question Figure:

Answer Figures:
Answer Figures:

(a) (b) (c) (d)


(a) (b) (c) (d) 26. Question Figure:
21. Question Figure:

Answer Figures:

Answer Figures:

(a) (b) (c) (d)


27. Question Figure:
(a) (b) (c) (d)
22. Question Figure:

Answer Figures:

Answer Figures:

(a) (b) (c) (d)


28. Question Figure:
(a) (b) (c) (d)
23. Question Figure:

Answer Figures:

Answer Figures:

(a) (b) (c) (d)


29. A square sheet of paper has been folded twice and
(a) (b) (c) (d) punched and then unfolded. The pattern of holes on the
24. A square transparent sheet with a pattern is given. Fig- sheet of paper has been shown as in the question figure.
ures out from amongst the four alternatives as to how Find out the punched hole pattern when the question
the pattern would appear when the transparent sheet is figure is folded twice.
folded at the middle line shown. Question Figure:
Question Figure:

Answer Figures:
Answer Figures:

(a) (b) (c) (d)


(a) (b) (c) (d)
30. Question Figure:
25. Question Figure:
Answer Figures:

Answer Figures:
(a) (b) (c) (d)
31. A square sheet of paper has been folded twice and
punched and then unfolded. The pattern of holes on the
(a) (b) (c) (d)
sheet of paper has been shown as in the question figure.
Find out the punched hole pattern when the question 36. A piece of paper is folded and punched as shown be-
figure is folded twice. low. From the given Answer Figures indicate how it
Question Figure: will appear when opented.
Question Figure:

Answer Figures:
Answer Figures:

(a) (b) (c) (d)


32. A paper is folded and cut as shown in the given ques- (a) (b) (c) (d)
tion. When opened, how will it appear?
37. Question Figure:
Question Figure:

Answer Figures:
Answer Figures:

(a) (b) (c) (d)


(a) (b) (c) (d) 38. How will it look like when a transparent paper with
33. Question Figure: given figure is folded on the dotted line as shown?
Question Figure:

Answer Figures:
Answer Figures:

(a) (b) (c) (d)


(a) (b) (c) (d)
34. Question Figure:
39. A rectangular paper is folded in a particular manner
and a punch is made. When unfolded, the paper ap-
pears as given belw.
Answer Figures: Question Figure:

(a) (b) (c) (d) Answer Figures:

35. A square-sheet of paper is four-folded (i.e..4 layer of


paper are made of the sheet). Then it is cut on one edge
of the triangular fold. Find the figure pattern of sheet (a) (b) (c) (d)
on unfolding from the answer figures.
40. What figure will be formed when a transparent sheet
Question Figure:
with the given figures is folded on the dotted line? Answer Figures:
Question Figure:

(a) (b) (c) (d)


Answer Figures: 45. A piece of paper is folded and cut as shown below in
the question figures. From the given answer figgures,
indicate how it will pear when opened?
Question Figure:
(a) (b) (c) (d)
41. A piece of paper is folded and punched as shown be-
low. From the given responses indicate how it willo
Answer Figures:
appear when opened.
Question Figure:

(a) (b) (c) (d)


Answer Figures:
46. A piece of paper is folded and punched as shown be-
low in the question figures. From the given answer fig-
ures, indicate how it will appear when opened?
(a) (b) (c) (d) Question Figure:
42. A piece of paper is folded and cut as shown below in
the question figures. From the given answer figures.
From the given answer figures, indicate how it will ap-
Answer Figures:
pear when opened?
Question Figure:

(a) (b) (c) (d)


47. A triangular sheet of paper has been folded and punched
Answer Figures: shown in the following series of figures. How will it
appear when opened?
Question Figure:

(a) (b) (c) (d)


43. A piece of paper is folded and cut as shown below in
the questions figures. From the given answer figures, Answer Figures:
indicate how it will appear when opened?
Question Figure:

(a) (b) (c) (d)


Answer Figures: 48. If the following pattern is drawn in a transparent square
sheet and folded along the dotted lines, how does it
appear?
Question Figure:
(a) (b) (c) (d)
44. A piece of paper is folded and cut as shown below in
the question figures. From the given answer figures,
indicate how it will appear when opened? Answer Figures:
Question Figure:
(a) (b) (c) (d) 52. Question Figure:
49. If the following patten is drawn in a transparent square
sheet and folded along the dotted lines, how does it
appear?
Question Figure: Answer Figures:

Answer Figures: (a) (b) (c) (d)


53. Question Figure:

(a) (b) (c) (d)


50. A square sheet of paper has been folded and punched Answer Figures:
as shown in the following series of figures. How will it
appear when opened?
Question Figure:
(a) (b) (c) (d)
54. Question Figure:

Answer Figures:

Answer Figures:

(a) (b) (c) (d)


51. Question Figure: (a) (b) (c) (d)
55. A circular sheet of paper has been folded and punched
as shown in the following series of figures. How will it
appear when opened?
Question Figure:
Answer Figures:

Answer Figures:
(a) (b) (c) (d)

(a) (b) (c) (d)


Answer
1. (b) 2. (b) 3. (c) 4. (c) 5. (c) 6. (d)
7. (a) 8. (b) 9. (c) 10. (a) 11. (a) 12. (a)
13. (c) 14. (a) 15. (c) 16. (a) 17. (d) 18. (c)
19. (a) 20. (c) 21. (b) 22. (b) 23. (d) 24. (d)
25. (b) 26. (a) 27. (d) 28. (c) 29. (b) 30. (d)
31. (d) 32. (b) 33. (c) 34. (a) 35. (a) 36. (d)
37. (d) 38. (b) 39. (c) 40. (a) 41. (d) 42. (a)
43. (a) 44. (d) 45. (b) 46. (c) 47. (b) 48. (d)
49. (d) 50. (b) 51. (a) 52. (d) 53. (b) 54. (b)
55. (b)
Exercise – 3
Directions (1-2): Which answer figure will complete the
question figure?
Question Figure:
(a) (b) (c) (d)
5. Select the answer figure which sompletes the question
figure.
Question Figure:
Answer Figures:

(a) (b) (c) (d) Answer Figures:


2. Which answer figure will complete the question fig-
ure?
Question Figure:
(a) (b) (c) (d)
6. Which answer figures will complete the question fig-
ure?
Question Figure:

Answer Figures:
?
(a) (b) (c) (d)
Answer Figures: 7. Directions (7): Which one of the answer figures shall
complete the given question figure?
Question Figure:
(a) (b) (c) (d) ?
Directions (3-4): Which answer figure will complete the
question figure?
3. Question Figure: Answer Figures:

(a) (b) (c) (d)


8. Which one of the answer figures shall complete the
given questions figure?
Answer Figures: Question Figure:

(a) (b) (c) (d)


4. Question Figure:
Answer Figures:

(a) (b) (c) (d)


9. Which one of the answer figures shall complete the
Answer Figures:
given question figure?
Question Figure: (a) (b) (c) (d)
13. Which one of the answer figures shall complete the
given question figure?
Question Figure:
? * *
Answer Figures:
*
Answer Figures:

(a) (b) (c) (d)


10. Which one of the answer figures shall complete the
*
* *
given question figure?
Question Figure: (a) (b) (c) (d)
14. Which answer figure is the part of the following ques-
? tion figure?
Question Figure:
Answer Figures:

Answer Figures:
(a) (b) (c) (d)
11. Which answer figure will complete the question fig-
ure?
Question Figure:
(a) (b) (c) (d)
15. Which one of the answer figures shall complete the
given question figure?
Question Figure:

Answer Figures:

Answer Figures:
(a) (b)

(a) (b) (c) (d)


16. Which one of the answer figures shall complete the
(c) (d) given question figure?
Question Figure:
Directions (12): In question which answer figure will com-
plete the question figure?
12.
Question Figure:

Answer Figures:

(a) (b) (c) (d)


Answer Figures: 17. Which one of the answer figures shall complete the
given question figure?
Question Figure:
Answer Figures:

Answer Figures:
(a) (b) (c) (d)
22. Which answer figure will complete the pattern in the
question figure?
(a) (b) (c) (d) Question Figure:
18. Which Answer figure will complete the Question fig-
ure?
Question Figure:

Answer Figures:

(a) (b) (c) (d)


Answer Figures:
23. Question Figure:

(a) (b) (c) (d)


19. Which answer figure will complete the qestion figure? Answer Figures:
Question Figure:

(a) (b) (c) (d)


24. Question Figure:
Answer Figures:

Answer Figures:
(a) (b) (c) (d)
20. Which answer figure will complete the question fig-
ure?
Question Figure:
(a) (b) (c) (d)
25. Question Figure:

Answer Figures:

Answer Figures:

(a) (b) (c) (d)


21. Which answer figure will complete the questions fig- (a) (b) (c) (d)
ure? 26. Question Figure:
Question Figure:

Answer Figures:
(a) (b) (c) (d)
27. Question Figure: Answer Figures:

?
Answer Figures: (a) (b) (c) (d)
32. Question Figure:

(a) (b) (c) (d)


28. Question Figure: Answer Figures:

Answer Figures: (a) (b) (c) (d)


33. Question Figure:

(a) (b) (c) (d)


29. Question Figure: Answer Figures:

(a) (b) (c) (d)


34. In the given question, a part of the figure is missing.
Answer Figures: Select one of the response figures which exactly fits
into the missing part of the question figure.
Question Figure:

(a) (b) (c) (d)


30. Question Figure:
Answer Figures:

Answer Figures: (a) (b) (c) (d)


35. Question Figure:

(a) (b) (c) (d)


31. Which one of the answer figures shall complete the
given question figure? Answer Figures:
Question Figure:

(a) (b) (c) (d)


36. Question Figure:
?

Answer Figures:
Answer Figures:

(a) (b) (c) (d) (a) (b) (c) (d)


37. Question Figure: 42. Question Figure:

Answer Figures: Answer Figures:

(a) (b) (c) (d) (a) (b) (c) (d)


38. Question Figure: 43. Question Figure:

Answer Figures:

Answer Figures:

(a) (b) (c) (d)


39. Question Figure:
(a) (b) (c) (d)
44. Question Figure:

Answer Figures:

Answer Figures:
(a) (b) (c) (d)
40. Question Figure:

? (a) (b) (c) (d)


45. Question Figure:

Answer Figures:
?

(a) (b) (c) (d) Answer Figures:


41. Question Figure:

(a) (b) (c) (d)


46. Question Figure:

?
?
Answer Figures:
Answer Figures:
.
.. .. ..

(a) (b) (c) (d)


47. Question Figure:
(a) (b) (c) (d)
? 52. Question Figure:

Answer Figures:
?

Answer Figures:
(a) (b) (c) (d)
48. Question Figure:

(a) (b) (c) (d)


?
53. Question Figure:
Answer Figures:
?

Answer Figures:
(a) (b) (c) (d)
49. Question Figure:

(a) (b) (c) (d)


? 54. Question Figure:
Answer Figures:
?
Answer Figures:

(a) (b) (c) (d)


50. Question Figure:

(a) (b) (c) (d)


? 55. Question Figure:

Answer Figures:

(a) (b) (c) (d) Answer Figures:


51. Which answer figure will complete the patternin the
question figure?
Question Figure:

(a) (b) (c) (d)


56. Which answer figure will complete the question fig-
ure?
Question Figure:

Answer Figures:

Answer Figures: (a) (b)

(c) (d)
(a) (b) (c) (d)
57. Question Figure:
61. Which answer figure will complete the pattern in the
question figure?
Question Figure:

Answer Figures:

Answer Figures:

(a) (b) (c) (d)


58. Which one of the following figures will complete the
given question figure?
Question Figure: (a) (b) (c) (d)
62. Which answer figure will complete the pattern in the
question figure?
Question Figure:
Answer Figures:

Answer Figures:
(a) (b) (c) (d)
59. Which answer figure will complete the pattern in the
question figure?
Question Figure: (a) (b) (c) (d)
63. Which answer figure will complete the pattern in the
question figure?
Question Figure:

Answer Figures:

(a) (b) (c) (d) Answer Figures:


60. Which answer figure will complete the pattern in the
question figure?
Question Figure:
(a) (b) (c) (d)
64. Which answer figure will complete the pattern in the 65. Which answer figure will complete the pattern in the
the question figure? question figure?
Question Figure: Question Figure:

* *
Answer Figures: *
Answer Figures:

(a) (b) (c) (d)


* * *
(a) (b) (c) (d)

Answer
1. (a) 2. (c) 3. (b) 4. (b) 5. (d) 6. (d)
7. (d) 8. (a) 9. (a) 10. (d) 11. (a) 12. (a)
13. (b) 14. (d) 15. (a) 16. (a) 17. (c) 18. (b)
19. (d) 20. (a) 21. (c) 22. (b) 23. (a) 24. (a)
25. (b) 26. (c) 27. (a) 28. (b) 29. (b) 30. (c)
31. (b) 32. (c) 33. (d) 34. (c) 35. (a) 36. (c)
37. (d) 38. (d) 39. (c) 40. (c) 41. (b) 42. (c)
43. (a) 44. (c) 45. (a) 46. (c) 47. (d) 48. (d)
49. (a) 50. (c) 51. (c) 52. (d) 53. (a) 54. (b)
55. (a) 56. (c) 57. (a) 58. (c) 59. (d) 60. (b)
61. (b) 62. (a) 63. (b) 64. (a) 65. (d)
Exercise – 4
Directions (1-2): Find the missing figure in the series from (a) (b) (c) (d)
the given answer figures. 7. Questions Figures:
1. Questions Figures:
?
? Answer Figures:

Answer Figures:

(a) (b) (c) (d)


Directions (8-11): Find the missing figure in the series from
the given answer figures.
(a) (b) (c) (d)
8. Questions Figures:
2. Question Figures

? ?
Answer Figures:
Answer Figures

(a) (b) (c) (d)


(a) (b) (c) (d)
9. Questions Figures:
Directions (3-4): Complete the series of figures, by select-
ing correct answer figure from the given responses.
?
3. Questions Figures: Answer Figures:

?
Answer Figures: (a) (b) (c) (d)
10. Questions Figures:

4.
(a) (b) (c)
Questions Figures:
(d)
?
Answer Figures:
?
Answer Figures:

(a) (b) (c) (d)


(a) (b) (c) (d)
11. Questions Figures:
5. Questions Figures:
×
× × + + ?
?
– – – Answer Figures:
Answer Figures:
+
+
– –×
× ×– ×– (a) (b) (c) (d)
(a) (b) (c) (d) 12. Which of the following answer figures can complete
6. Questions Figures: the given series?
× × Questions Figures:
× ×
?
Answer Figures:
?
× Answer Figures:
×
× ×
(a) (b) (c) (d) (a) (b) (c) (d)
13. Find the missing figure in the series from the given an- 19. Questions Figures:
swer figures. C
Questions Figures:
C
C
C
?
Answer Figures:
? C C
Answer Figures: C
C
(a) (b) (c) (d)
20. Questions Figures:
....
....
(a) (b) (c) (d) ..
14. Questions Figures:
.....
.....
?
Answer Figures:
? .....
... .....
...
.... ....
.
Answer Figures:
(a) (b) (c) (d)
21. Questions Figures:
(a) (b) (c) (d)
15. Questions Figures: ?
+ -
- - Answer Figures:
+
+ ?
Answer Figures:
+ +
- (a) (b) (c) (d)
- - + + - 22. Questions Figures:
(a) (b) (c) (d)
16. Questions Figures: ?
? Answer Figures:

Answer Figures:

(a) (b) (c) (d)


23. Questions Figures:
(a) (b) (c) (d)
17. Questions Figures:
?
? Answer Figures:

Answer Figures:
(a) (b) (c) (d)
24. Questions Figures:
(a) (b) (c) (d) × ×
18. Questions Figures: × ?
Answer Figures:
? ×
Answer Figures: × × ×
(a) (b) (c) (d)
25. Questions Figures:
+ +
? + +

Answer Figures: (a) (b) (c) (d)


32. Questions Figures:

(a) (b) (c)


26. Questions Figures:
(d) ?
Answer Figures:
?
Answer Figures:
(a) (b) (c) (d)
33. Questions Figures:

(a) (b) (c)


27. Questions Figures:
(d)
?
Answer Figures:
?
Answer Figures:
(a) (b) (c) (d)
34. Questions Figures:
(a) (b) (c) (d)
28. Questions Figures: ?
+ + Answer Figures:
?
+ +
Answer Figures:
+ + (a) (b) (c) (d)
35. Questions Figures:
+ +
(a) (b) (c) (d)
29. Questions Figures: ?
? Answer Figures:

Answer Figures:

(a) (b) (c) (d)


36. Questions Figures:
(a) (b) (c) (d)
30. Questions Figures:
?
? Answer Figures:
Answer Figures:

(a) (b) (c) (d)


(a) (b) (c) (d) 37. Questions Figures:
31. Questions Figures:
?
W
W

+
+
+
+
? W

Answer Figures:
Answer Figures:
44. Questions Figures:

(a) (b) (c) (d)


?
38. Questions Figures: Answer Figures:

?
Answer Figures: (a) (b) (c) (d)
45. Questions Figures:

(a) (b) (c) (d)


?
39. Questions Figures: Answer Figures:

?
Answer Figures:
(a) (b) (c) (d)
46. Questions Figures:

(a) (b) (c)


40. Questions Figures:
(d) ?
Answer Figures:
++++
+ ++
+++
+
++++
?
Answer Figures:
+
++++ (a) (b) (c) (d)
++++
+++

++
++
++

47. Questions Figures:


×
(a) (b) (c)
41. Questions Figures:
(d) × × ?
Answer Figures:

? × ×
× ×

Answer Figures: (a) (b) (c) (d)


48. Questions Figures:

(a) (b) (c) (d)


42. Questions Figures: Answer Figures:

? (a) (b) (c) (d)


Answer Figures:
49. Questions Figures:
× ×
×
?
(a) (b) (c) (d)
43. Questions Figures: Answer Figures:
× ×
? ×
×
×
Answer Figures: (a) (b) (c) (d)
50. Questions Figures:

(a) (b) (c) (d)


+ + +
? ×
(a) (b) (c) (d)
Answer Figures:
57. Questions Figures:

(a) (b) (c) (d)


?
51. Questions Figures: Answer Figures:

?
Answer Figures:
(a) (b) (c) (d)
58. Questions Figures:
×
(a) (b) (c)
52. Questions Figures:
(d)
× × ?
Answer Figures:
?
Answer Figures:
× ××
(a) (b) (c) (d)
59. Questions Figures:
(a) (b) (c) (d)
53. Questions Figures:
+
s +
s
+
s
s
+ ?
? Answer Figures:
s s s s
+ + + +
Answer Figures:

(a) (b) (c) (d)


60. Questions Figures:
(a) (b) (c) (d)
54. Questions Figures:
?
? Answer Figures:
Answer Figures:

(a) (b) (c) (d)


(a) (b) (c) (d) 61. Questions Figures:
55. Questions Figures:

? ?
Answer Figures:
Answer Figures:

(a) (b) (c) (d) (a) (b) (c) (d)


56. Questions Figures: 62. Questions Figures:
+
× × ? ?
Answer Figures:
Answer Figures:
(a) (b) (c) (d) (a) (b) (c) (d)
63. Questions Figures: 69. Questions Figures:

? ?
Answer Figures:
Answer Figures:

(a) (b) (c) (d)


(a) (b) (c) (d)
70. Questions Figures:
64. Questions Figures:

? ?
Answer Figures:
Answer Figures:

(a) (b) (c) (d)


(a) (b) (c) (d) 71. Questions Figures:
65. Questions Figures:

?
?
Answer Figures:
Answer Figures:

(a) (b) (c) (d)


(a) (b) (c) (d) 72. Questions Figures:
66. Questions Figures:

? ?
Answer Figures: Answer Figures:

(a) (b) (c) (d) (a) (b) (c) (d)


67. Questions Figures: 73. Questions Figures:

? ?
Answer Figures: Answer Figures:

(a) (b) (c) (d) (a) (b) (c) (d)


68. Questions Figures: 74. Questions Figures:

?
Answer Figures: Answer Figures:
×
×× ××
× ××
(a) (b) (c) (d) (a) (b) (c) (d)
75. Questions Figures: 81. Questions Figures:

? ?
Answer Figures: Answer Figures:

(a) (b) (c) (d) (a) (b) (c) (d)


76. Questions Figures: 82. Questions Figures:

?
? Answer Figures:
Answer Figures:

(a) (b) (c) (d)


(a) (b) (c) (d) 83. Questions Figures:
77. Questions Figures:

? Answer Figures:
Answer Figures:

(a) (b) (c) (d)


84. Questions Figures:
(a) (b) (c) (d)
78. Questions Figures: ?

Answer Figures:
?
Answer Figures:
(a) (b) (c) (d)
85. Questions Figures:

(a) (b) (c) (d)


79. Questions Figures: Answer Figures:

?
Answer Figures: (a) (b) (c) (d)
86. Questions Figures:

?
(a) (b) (c) (d) Answer Figures:
80. Questions Figures:
×
××
× ××
? (a) (b) (c) (d)
87. Questions Figures:
Answer Figures:
89. Questions Figures:
?
Answer Figures:
?
Answer Figures:

(a) (b) (c) (d)


(a) (b) (c) (d)
88. Questions Figures:
90. Questions Figures:

? ?
Answer Figures:
Answer Figures:

(a) (b) (c) (d)


(a) (b) (c) (d)

Answer
1. (b) 2. (c) 3. (c) 4. (d) 5. (a) 6. (a)
7. (c) 8. (a) 9. (b) 10. (b) 11. (c) 12. (b)
13. (d) 14. (d) 15. (d) 16. (a) 17. (a) 18. (b)
19. (a) 20. (d) 21. (b) 22. (c) 23. (b) 24. (a)
25. (c) 26. (d) 27. (d) 28. (a) 29. (a) 30. (b)
31. (c) 32. (a) 33. (a) 34. (c) 35. (b) 36. (c)
37. (a) 38. (a) 39. (d) 40. (a) 41. (a) 42. (d)
43. (c) 44. (d) 45. (a) 46. (b) 47. (c) 48. (a)
49. (a) 50. (d) 51. (a) 52. (b) 53. (c) 54. (b)
55. (d) 56. (a) 57. (c) 58. (d) 59. (a) 60. (a)
61. (a) 62. (a) 63. (c) 64. (c) 65. (c) 66. (b)
67. (c) 68. (c) 69. (c) 70. (a) 71. (a) 72. (a)
73. (c) 74. (d) 75. (d) 76. (b) 77. (c) 78. (d)
79. (c) 80. (c) 81. (b) 82. (c) 83. (c) 84. (c)
85. (d) 86. (d) 87. (d) 88. (d) 89. (b) 90. (b)
Exercise – 5
1. Questions Figures:
.
....
..... ....
... ?
4 5 6
Answer Figures:
(a) 1, 3 (b) 3, 4
(c) 3, 5 (d) 4, 6
(a) (b) (c) (d)
8. Questions Figures:
2. Questions Figures:

? ?
Answer Figures:
Answer Figures:

(a) (b) (c) (d) (a) (b) (c) (d)


3. Questions Figures: 9. Questions Figures:

? ?
Answer Figures: Answer Figures:

(a) (b) (c) (d)


(a) (b) (c) (d)
4. Questions Figures:
10. Questions Figures:
? ?
Answer Figures:
Answer Figures:

(a) (b) (c) (d) (a) (b) (c) (d)


5. Identify the two figures which are similar in all respects. 11. Questions Figures:

?
Answer Figures:
(1) (2) (3) (4) (5) (6)
(a) 3 - 6 (b) 1 - 4
(c) 2 - 4 (d) 2 - 5
6. Questions Figures: (a) (b) (c) (d)

? 12. Questions Figures:

Answer Figures:

1 2 3 4 5 6
(a) (b) (c) (d) (a) 1 -3 (b) 4 - 6

7. Questions Figures: (c) 2-4 (d) 3 - 5


13. Questions Figures:

1 2 3 Answer Figures:
(a) (b) (c) (d)
1 2 3 4
14. Questions Figures:

? 5 6 7 8
(a) 1–5 (b) 3 – 6
Answer Figures:
(c) 5–8 (d) 4 – 7
21. Questions Figures:
(a) (b) (c) (d)
15. Questions Figures:
?
Answer Figures:
?
Answer Figures:

(a) (b) (c) (d)


(a) (b) (c) (d) 22. Questions Figures:
16. Questions Figures:

?
?
Answer Figures:
Answer Figures:

(a) (b) (c) (d)


(a) (b) (c) (d)
17. Questions Figures:
23. Questions Figures:

Answer Figures:
?
Answer Figures:

(a) (b) (c) (d)


18. Questions Figures: (a) (b) (c) (d)
? 24. Four pairs of candles with stands are given below.
Among them only one pair is similar in all respects.
Answer Figures:
Identify the pair numbers which represent that pair.

(a) (b) (c) (d)


19. Questions Figures: 1 2 3 4

Answer Figures: 5 6 7 8
(a) 6-7 (b) 3 - 6
(a) (b) (c) (d) (c) 3-7 (d) 4 - 6
20. Four pairs of flower pots are given below. Among them 25. Questions Figures:
only one pair is similar in all respects. Identify the pair
numbers which represent that pair. ?
Answer Figures:
32. Questions Figures:

(a) (b) (c) (d)


?
Answer Figures:
26. Questions Figures:

? (a) (b) (c) (d)


Answer Figures: 33. Questions Figures:

?
(a) (b) (c) (d) Answer Figures:
27. Questions Figures:

?
Answer Figures: (a) (b) (c) (d)
34. Questions Figures:

(a) (b) (c) (d) ?


28. Questions Figures: Answer Figures:

?
Answer Figures:
(a) (b) (c) (d)
35. Questions Figures:
(a) (b) (c) (d)
29. Questions Figures: ?
Answer Figures:
?
Answer Figures:

(a) (b) (c) (d)

(a) (b) (c) (d) 36. Questions Figures:

30. Questions Figures:


?
Answer Figures:
Answer Figures:

(a) (b) (c) (d)

(a) (b) (c) (d) 37. Questions Figures:

31. Questions Figures:

Answer Figures:
Answer Figures:

(a) (b) (c) (d)


(a) (b) (c) (d)
38. Questions Figures: × ×× ×× ×
(c) ×× × (d) × ××
××× ×× ×
44. Questions Figures:
Answer Figures:
?
Answer Figures:
(a) (b) (c) (d)
39. Questions Figures:
(a) (b) (c) (d)
++ +
? 45. Questions Figures:
Answer Figures:
?
Answer Figures:
(a) (b) (c) (d)
40. Questions Figures:

Answer Figures:
? 46.
(a) (b)
Questions Figures:
(c) (d)

+ + ?
(a) (b) (c) (d)
Answer Figures:
41. Questions Figures:

?
Answer Figures:
(a) (b) (c) (d)
47. Questions Figures:

(a) (b) (c) (d)


42. Questions Figures: ?
Answer Figures:
?
Answer Figures:

(a) (b) (c) (d)


48. Questions Figures:
(a) (b) (c) (d)
43. Questions Figures:
?
× ×
× ×
× ×
? Answer Figures:

Answer Figures:
× × × ×
(a) ×××× (b) × × ×× (a) (b) (c) (d)
× × × ×
49. Questions Figures: (a) (b) (c) (d)
50. Questions Figures:
?
Answer Figures:
?
Answer Figures:

(a) (b) (c) (d)

Answer
1. (c) 2. (d) 3. (d) 4. (d) 5. (b) 6. (b)
7. (c) 8. (b) 9. (d) 10. (d) 11. (d) 12. (c)
13. (c) 14. (d) 15. (b) 16. (b) 17. (a) 18. (d)
19. (c) 20. (b) 21. (c) 22. (a) 23. (c) 24. (c)
25. (b) 26. (b) 27. (b) 28. (d) 29. (b) 30. (b)
31. (a) 32. (b) 33. (d) 34. (b) 35. (b) 36. (a)
37. (b) 38. (b) 39. (b) 40. (a) 41. (c) 42. (a)
43. (a) 44. (b) 45. (c) 46. (a) 47. (a) 48. (d)
49. (b) 50. (d)
17. PUZZLE
Directions (1 to 5) : Study the following information immediate right of C. A is on the immediate left of
carefully and answer the question given below: D.
Eight students A, B, C, D, E, F, G & H study section X, 06. Who is seated opposite to A?
Y, Z of class. In each section, there are atleast two but not (1) G (2) D
more then three students and each of them likes one of the
(3) C (4) A
games, namely Cricket, Volleyball, Basketball, Football,
Tabletennis, Hockey, Carrom and Lawn-Tennis but not (4) None of these
necessarily in same order. D likes Volleyball & study in 07. Who is next to E in Clockwise direction?
section Y with those students who like football & Lawn- (1) H (2) B
Tennis. B likes Cricket & does not study in Section Z. Student (3) F (4) A
who likes Carrom studies with F in his section. E and H (4) None of these
neither study in Section Y nor in Section Z. C neither like
08. Which of the folloing pairs of two persons, both perosn
Carrom nor Lawn-Tennis, G does not study in Section Z.
are facing same direction and first person is on the right
who likes Basketball does not study in Section X. H does
of second person?
not like Hockey.
(1) DF (2) CB
1. Which game does G like?
(3) FC (4) AG
(1) Table Tennis (2) Basketball
(4) None of these
(3) Lawn-Tennis (4) Data inadequate
09. Who is sitting opposite to E?
(5) None of these
(1) D (2) A
2. In which section does A study?
(3) F (4) A or D
(1) X (2) Z
(5) None of these
(3) Y (4) Y and Z
10. Which one of the following statement given below is
(4) None of these
definitely true?
3. Which of the following groups of students do study in
(1) A is facing towards North
section Y?
(2) E is seated opposite to F
(1) BEH (2) FA
(3) F is seated on the left of G
(3) DPA (4) DCG
(4) C is seated on the left of A
(4) None of these
(5) None of these
4. Which game does H like?
Direction (11 to 15) : Study the following information
(1) Table - Tennis (2) Basketball
carefully and answer the questions given below:
(3) Lawn - Tennis (4) Data inadequate
Seven friends P, F, R, T, Q, N & D study the seven
(4) None of these different subjects, namely IT, Civil, HR, Marketing, Finance,
5. Which game does 'C' like? Journalism & Pharmacy but not necessarily in same order.
(1) Hockey (2) Carram Each of them likes one of the colours, namely Red, Blue,
(3) Lawn Tennis (4) Data inadequate Green, Yellow, Pink, Orange, Grey but not necessarily in given
(5) None of these oredr. Out of them, three are girls. P does not study IT or HR
but likes yellow colour. A girl studies Civil & likes grey colour,
Direction (6 to 10) : Study the following information
Q, sister of N, studies marketing & likes Pink colour. D's
carefully and answer the question given below:
subject is Pharmacy & likes red colour. R's wife N studies
Eight Person A, B, C, D, E, F, G& H are seated HR & likes green colour. F like grey & R likes orange colour.
around a rectangular table, two on each side, facing Person who likes blue colour, studies finance.
the centre and just opposite to are another. D is seated
11. Who studies civil ?
opposite to H & facing towards North. E is on the
immediate left of H. A & G are seated facing same (1) P (2) T
direction. G is just opposite to B who is on the (3) F
(4) Cannot be determined 19. In the following five options, four are like on the basis
(5) None of these of their place, so they form a group. Find the group
12. Which one of the following groups belongs to girls? which is different from them?

(1) F, D, N (2) F, Q, N (1) TP (2) RV

(3) Q, N, P (3) QP (4) QV

(4) Cannot be determined (5) MW

(5) None of these 20. In which of the following pairs of person, second perosn
is to the immediate left of the first person ?
13. Which subject does 'R' study?
(1) UP (2) VP
(1) Civil
(3) QM (4) RT
(2) Finance
(4) None of these
(3) Journalism
Direction (21 to 25) : Study the following information
(4) Cannot be determined
carefully and answer the question given below:
(5) None of these
P, Q, R, S, T, V, W and Z are eight friends studying in
14. Who studies Journalism? three different engineering colleges - A, B and C in three
(1) P (2) Q disciplines - Mechanical, Electrical and Electronics with not
(3) R less than two and not more than three in any college. Not
(4) Cannot be determined more than three of them study in any of the three disciplines.
(5) None of these W studies Electrical in college B with only T, who studies
Mechanical. P and Z do not study in college C and study in
15. Which one of the following options is correct sequence
the same discipline but not Electrical. R studies Mechanical
of preson, colour & subjects?
in college C with V, who studies Electrical. S studies
(1) Blue -T- Marketing (2) Pink - N- HR
Mechanical and does not study in the same college where R
(3) Orange - R- Civil (4) Blue- T- Finance studies. Q does not study Electronics.
(5) None of these 21. Which of the following combinations of college-
Direction (16 to 20) : Study the following information student-specialisation is correct?
carefully and answer the questions given below: (1) C-R Electronics (2) A-Z Electrical
Eight friends P, M, R, T, Q, U, V & W are seated in two (3) B-W-Electronics (4) B-W- Electrical
rows facing one another. Four persons are in each row. P is in
(5) B-Z-Electronics
between U &V & facing towards North. Q who is to the
22. In which of the following colleges do two students study
immediate left of M and opposite to W. R is between T & M.
in Electrical discipline?
W is on the immediate right of V.
(1) A only
16. Who is sitting opposite to R?
(2) B only
(1) U (2) Q
(3) C only
(3) V (4) P
(4) Cannot be determined
(5) None of these
(5) None of these
17. Who is Immediate right of R?
23. In which discipline does Q study?
(1) M (2) U
(1) Electrical
(3) M or T
(2) Mechanical
(4) Cannot be determined
(3) Electrical or Mechanical
(5) None of these
(4) Data inadequate
18. In which of the following pairs of people one person is
sitting opposite to another? (5) None of these
(1) MV (2) RV 24. In which of the colleges at least one student studies in
Mechanical discipline?
(3) TV (4) UR
(1) A only (2) B only
(5) None of these
(3) C only (4) Both A and B
(5) All A, B and C A, M, D, P, R, T, B & H are sitting around a circle,
25. S studies in which college? facing the centre. M is third to the left of A who is second to
(1) A (2) B the left of T. D is second to right of H who is second to the
right to T. R is second to right of B who is not a neighbour
(3) A or B (4) Data inadequate
of T.
(5) None of thes
31. In which of the following compositions, shows that first
Direction (26-30) : Study the following information & second are to the left of B?
carefully and answer the question given below:
(1) MD (2) DH
A, B, C, D, E, F, G and H are eight employees working
(3) AM (4) AR
in three departments - Marketing, Finance and Production -
in an organisation with at least two of them in any department. (5) DM
Each of them has a different choice of TV channels from 32. Who is third to the right of T?
STAR, Zee, ESPN, DD, Sony, NDTV, AAJ TAK and BBC (1) D (2) B
not necessarily in the same order. (3) H (4) M
D likes ESPN and he works in Production department (5) None of these
with only G. B and F do not work in the same department. H 33. Who is on the immediate left of H ?
likes DD and does not work in Finance department. A does
(1) P (2) M
not work in the same department with either F or C, who
does not work in Marketing department. E works with C in (3) T (4) R
the same department and likes Star. B likes Aaj Tak and none (5) Data inadequate
of his colleagues in the department likes either Sony or NDTV. 34. Who is second to the left of B?
G likes Zee. F does not like Sony. (1) D (2) H
26. Which channel does A like? (3) M (4) Date indaquate
(1) DD (2) NDTV (5) None of these
(3) BBC (4) Data inadequate 35. In which of the following compositions the third per-
(5) None of these son is second to the left of second person?
27. In which department does F work? (1) BAR (2) DBM
(1) Marketing (2) Production (3) TPH (4) PMH
(3) Finance (5) None of these
(4) Marketing or Finance Directions (36 to 40) : Study the following informa-
(5) None of these tion carefully and answer the question given below :
28. Which channel does F like? P, Q, R, S, T, V and W are seven students of a school.
(1) ZEE (2) NDTV Each of them studies in a different standard—from Standard
IV to Standard X—not necessarily in the same order. Each of
(3) BBC (4) Data inadequate
them has a favourite subject form English, Science, History,
(5) None of these Geography, Mathematics, Hindi and Sanskrit, not necessarily
29. Which of the following groups work in Marketing in the same order.
department? Q studies in VII Standard and does not like either Math-
(1) BAC (2) BGC ematics or Geography. R likes English and does not study
(3) HBE (4) BAH either in V or in IX. T studies in VIII Standard and likes
(5) None of these Hindi. The one who likes Science Studies in X Standard. The
30. In which department does B work? one who likes Geography studies in V Standard. W likes San-
skrit, P does not study in X Standard. S studies in IV Stan-
(1) Marketing (2) Production
dard.
(3) Finance (4) Data inadequate
36. In which standard does W study?
(5) None of these
(1) VII (2) IX
Direction (31 to 35) : Study the following informa-
(3) X (4) Data inadequate
tion carefully and answer the question given below:
(5) None of these
37. Which subject does P like? who will be third in the following?
(1) Geography (2) Mathematics (1) B (2) F
(3) English (4) History (3) G (4) B or G
(5) None of these (5) Cannot be determined
38. Which subject does S like? 44. How many students of them are shorter than F?
(1) History (2) Geography (1) Three (2) Four
(3) Mathematics (4) Data inadequate (3) Five (4) Data inadequate
(5) None of these (5) None of these
39. In which standard does P study? 45. Which of the following is unnecessary to answer the all
(1) IV (2) VII above question?
(3) IX (4) X (1) Only (iii) (2) Only (ii) & (iii)
(5) None of these (3) Only (iii) and (iv) (4) Only (ii) & (v)
40. Which of the following combinations of student-stan- (5) All are compulsory to answer the all questions.
dard-subject is correct? Direction (41 to 45) : Study the following informa-
(1) T - VIII - Mathematics tion carefully and answer the question given below :
(2) W - VII - Sanskrit Eight friends A, B, C, D, E, F, G & H are seated around
(3) Q - VII - Geography a circular table facing the centre. A is second to left of D who
is third to the left of E. C is third to the right of G who is not
(4) V - X - Science
a neighbour of E. H is third to the right of B who is second to
(5) None of these the right of G.
Direction (41 to 45) : Study the following informa- 46. Who is sitting in between D & C?
tion carefully and answer the question given below:
(1) Only B (2) Only C & A
(i) Eight students A, B, C, D, E, F, G & H are of each
(3) Only G (4) Only E
different height.
(5) Only G & E
(ii) D is shorter than A but taller than G.
47. Who is second to the right of E?
(iii) E is taller than H but shorter than C.
(1) B (2) F
(iv) B is shorter than D but taller than F.
(3) E (4) C
(v) C is shorter than G
(5) None of these
(vi) G is not so tall as F.
48. Which option is correct about 'C' in reference to F
41. Which one of the following options is not definitely
correct? (1) Third to the left

(1) G is shorter than F (2) Third to the left


(2) C is shorter that F (3) Third to the right
(3) F is shorter than C (4) Second to the right

(4) H is shorter than E (5) None of these

(5) All are correct 49. In the following five, four are like on the basis of their
sitting arrangement & so from a group. Which of the
42. If one more student 'T' is also added to this group of
following does not belong to the group?
student who is taller than E but shorter than G, then
which of the followig options would be correct? (1) CH (2) FG
(1) C &T are of same height (3) DA (4) BE

(2) T is shorter than D (5) GB


(3) T is shorter than H 50. In which of the folloing pairs second person is to the
immediate left of the first?
(4) T is taller than A
(1) DB (2) EH
(5) None of these
(3) FA (4) GD
43. If all eight students are arranged in descending order,
(5) None of these
Direction (51 to 55) : Study the following informa- (3) S (4) T
tion carefully and answer the question given below: 58. Who is Q' s son?
P, Q, R, S, T and M are six students of a school, one (1) A (2) D
each studies in Class I-VI. Each of them has a favourite colour (3) B (4) E
from red, black, blue, yellow, pink and green, not necessarily
59. Who is P's son?
in the same order.
(1) A (2) C
Q likes black and does not study in Class IV or V. The
one who studies in Class IV does not like green. P studies in (3) D (4) E
Class II. M likes blue and does not study in Class IV. The one 60. Who is R's Son ?
who likes yellow studies in Class VI. S likes pink and studies (1) E (2) A
in Class I. R does not study in Class VI. (3) B (4) D
51. In which class does R study? Directions (Q. 61-66) - Study the following information
(1) V (2) III carefully and answer the questions.
(3) IV (4) Data inadequate A, B, C, D, E, F, G and H are sitting around a circle,
(5) None of these facing the centre. A sits fourth to the right of H while
52. Which colur does R like? second to the left of F. C is not the neighbor of F and B,
D sits third to the right of C. H never sits next to G.
(1) Black (2) Yellow
61. Who amongst the following sits between B and D ?
(3) Red (4) Blue
(1) G (2) F
(5) None of these
(3) H (4) A
53. Which colour does P like?
(5) C
(1) Green (2) Yellow
62. Which of the following pairs sit between H and G ?
(3) Red (4) Data inadequate
(1) BH (2) EF
(5) None of these
(3) CE (4) DB
54. Which of the following combinations is correct?
(5) None of the these
(1) P - II - Yellow (2) Q - III - Green
63. Four of the following are alike in a certain way based
(3) S - I - Black (4) T - V - Yellow
on their questions in the seating arrangement and so
(5) None of these form a group. Which is the one that does not belong to
55. In which class does M study? the group ?
(1) IV (2) III (1) AE (2) HF
(3) II (4) V (3) BD (4) GE
(5) None of these (5) CH
Direction (56 to 60) : Study the following informa- 64. Who is immediate right of A?
tion carefully and answer the question given below: (1) C (2) D
A, B, C, D & E are the son of P, Q, R, S & T but not in (3) G (4) Date inadequate
this order. Made the correct combination of mother and son on
(5) None of these
the basis of the information givne below.
65. Who sits second to the right of B ?
Q is not the morher of B and C. T is not the mother of A
and E. C is not R's son. E is not the son of Q and S and 'R' is (1) A (2) C
not the initial letter of her mohter's name.Q and R is not A's (3) D (4) E
mother. (5) None of these
56. Who is S's son? 66. Which is the position of B with respect to C ?
(1) A (2) B I. Second of the right
(3) C (4) D II. Sixth to the left
57. Who is C's mother ? III. Third to the left
(1) P (2) R IV. Fifth to the right
(1) Only II (2) Only II and III (4) Third to the left
(3) Only I and IV (4) Date inadequate (5) None of these
(5) Both III and IV 73. Who sits between A and D ?
Directions (Q. 67-71) - Study the following information (1) F (2) E
carefully and answer the questions. (3) G (4) B
A, B, C, D, E, F, G and H are sitting around a circle, (5) H
facing the centre. E and G always sit next to each other. 74. What in E’s position with respect to C ?
D sits third to the right of C. F sits second to the left of
(1) To the immediate right
H. C never sits next to A, while D never sits next to G. H
is not the neighbor of D and C. (2) To the immediate left

67. Who sits to immediate right of F ? (3) Second to the right

(1) D (2) C (4) Cannot be determined

(3) B (4) A (5) None of these

(5) None of these 75. Which of the following pairs has the first person to the
immediate left of second person ?
68. Four of the following are alike in a certain way based
on their positions in the seating arrangement and so form (1) GB (2) A F
a group. Which is the one that does not belong to that (3) CE (4) H D
group. (5) None of these
(1) CH (2) BA 76. Who is second to the right of B ?
(3) FE (4) DG (1) F (2) A
(5) AC (3) H (4) D
69. Which of the following pairs sits between B and F ? (5) None of these
(1) HB (2) FD Directions (Q. 77-81) - Study the following information
(3) BG (4) CG carefully and answer the questions given below.
(5) AH A, B, C, D, E, F, G and H are sitting around a circle,
70. Who sits second to the left of B ? facing the centre. F is second to the right of A and third
to the left of C, B is second to the left of C and fourth to
(1) F (2) G
the right of H. D is second to the right of G.
(3) A (4) E
77. Who is to the immediate left of D ?
(5) None of these
(1) H (2) C
71. Who sits between A and D ?
(3) G (4) Date inadequate
(1) B (2) F
(5) None of these
(3) C (4) E
78. Who is second to the left of E ?
(5) None of these
(1) B (2) G
Directions (Q. 72-76) - Study the following information
(3) H (4) Data inadequate
carefully and answer the questions given below it.
(5) None of these
Eight friends A, B, C, D, E, F, G and H are sitting around
a circle, facing the centre. E is third to the left of G who 79. Who is third to the left of B ?
is to the immediate right of B who is third to the left of (1) E (2) H
A. H is second to the right of F who is not an immediate (3) F (4) Date inadequate
neighbor of E. D is not an immediate neighbor of B. (5) None of these
72. Which of the following is the correct position of B with 80. What is the position of G with respect to A ?
respect to D ? (1) Third to the left (2) Third to the right
(1) Second to the right (3) Fifth to the left (4) Fourth to the right
(2) Second to the left (5) Fifth to the right
(3) Third to the right
81. In which of one following combinations is the third (3) Saturday (4) Data inadequate
person sitting in between the first and the second (5) None of these
persons ? 86. Which of the folowing is R's day off ?
(1) BGC (2) EFB (1) Friday (2) Thursday
(3) DAH (4) AEF (3) Tuesday (4) Data inadequate
(5) GCD (5) None of these
Directions (Q. 82-84) - Study the following information 87. Which of the following groups of friends work in shift
carefully and answer the questions given below it. II ?
Five plays A, B, C, D and E were organised in a week (1) RP (2) RV
from Monday to Saturday with one play each day and
(3) QWS (4) Data inadequate
no play was organised on one of these days. Play D was
organised before Thursday but after Monday. Play E (5) None of these
was organised on Saturday. Play C was not organised 88. Which of the following is Q's day off ?
on the first day. Play B was organised on the next day (1) Friday (2) Wednesday
on which Play C was organized. A was organised on (3) Thursday (4) Monday
Tuesday. (5) None of these
82. On which day was Play B organised ? 89. Which of the following groups of friends work in shift
(1) Thursday (2) Friday I?
(3) Wednesday (4) Data inadequate (1) RV (2) RP
(5) None of these (3) QWS (4) Data inadequate
83. On which day no Play was organised ? (5) None of these
(1) Monday (2) Wednesday Directions (Q. 90-94) : Study the following
(3) Thursday (4) Data inadequate information carefully and answer the questions given below;
(5) None of these One of the seven subjects, viz Maths, Zoology, Botany,
84. Which play was organised on Wednesday ? Chemistry, Physics, English and Statistics, is taught on
(1) A (2) C one day in a week starting from Monday and ending on
Sunday. Chemistry is taught on Thursday. English is
(3) D (4) Data inadequate
taught the day immediately next to the daty when Zool-
(5) None of these ogy is taught. English is taught neither on Tuesday nor
Directions (Q. 85-89) : Study the following information on Saturday. Only one lecture is held between Chemis-
carefully and answer the questions given below ; try and Botany. Two letures are scheduled between
P, Q, R, S, T, V and W are seven friends working in maths and Zoology. Statistics is taught neither on Mon-
acall centre. Each of them has different day offs in a day nor on Sunday.
week from Monday to Sunday not necessarily in the 90. On which of the following days in Physics taught?
same order. They work in three different shifts I, II and (1) Monday (2) Tuesday
III with at least two of them in each shift.
(3) Wednesday (4) Thursday
R works in shift II and his day off is not Sunday. P's day
(5) Friday
off is Tuesday and he does not work in the same shift
with either Q or W. None of those who work in shift I 91. How many subjects are taught between Botany and
has day off either on Wednesday or on Friday. V works Zoology ?
with only T in shift III. S' day off is Sunday. V's day off (1) None (2) One
is immediate next day of that of R's day off. T's day off (3) Two (4) Three
is not on Wedensday. W's day off is not on the previous (5) Four
day of P's day off. S works in shift I. Q does not work in 92. Which of the following subjects is taught on Sat-
the same shift with R and his day off is not on Thurs- urday ?
day.
(1) Botany (2) Statistics
85. Which of the following ins W's day off ?
(3) Zoology (4) Maths
(1) Tuesday (2) Monday
(5) Physics 96. Which item does D plays ?
93. On which of the following days in Statistics taught ? (1) Mimicri - Monday
(1) Tuesday (2) Wednesday (2) Music - Tuesday
(3) Thursday (4) Friday (3) Drama - Wednesday
(5) Cannot be determined (4) Speech - Monday
94. If Statistics is related to Zoology and Physics is related (5) None of these
to Botany in a certain way, then which of he following 97. What day of the week is A performance ?
would Chemistry be related to, following the same pat- (1) Tuesday (2) Wednesday
tern ?
(3) Friday (4) Saturday
(1) Maths (2) Statistics
(5) None of these
(3) Physics (4) English
98. What day of the week is G performance ?
(5) Cannot be determined
(1) Wednesday (2) Saturday
Directions (Q. 95-99) : Study the following
(3) Thesday
information carefully and answer the questions given below;
(4) Cannot be determined
Seven Friends A, B, C, D, E, F & C, from Monday to
Sunday Fun stage show exhibiting different, but not (5) None of these
necessarily in the same order, each shows a different
item. Such as music, dance, Mimicri, drama, debate, 99. Who will perfrom debate ?
speech and monologue, but not necessarily in that or- (1) B (2) D
der. (3) F (4) C
B performs Drama on Thursday and E perfroms Music (5) None of these
on Sunday. G performs Mimicri and doest not play ei-
ther Tuesday or Saturday. C performs after the next day
100. What day of the week is F performance ?
of G. D perfroms on Monday but does not play either
Dance or Debate. A performs Monologue who is next (1) Wednesday (2) Saturday
to the speech. Dance does not perform on Saturday. (3) Tuesday
95. Who perform the dance ? (4) Cannot be determined
(1) C (2) F (5) None of these
(3) D (4) C or D
(5) None of these

Answer
1. (3) 2. (2) 3. (4) 4. (1) 5. (5) 6. (3) 7. (2)
8. (4) 9. (3) 10. (2) 11. (3) 12. (4) 13. (5) 14. (1)
15. (4) 16. (4) 17. (5) 18. (1) 19. (3) 20. (2) 21. (4)
22. (3) 23. (1) 24. (5) 25. (1) 26. (3) 27. (3) 28. (2)
29. (4) 30. (1) 31. (5) 32. (4) 33. (1) 34. (3) 35. (2)
36. (2) 37. (1) 38. (3) 39. (5) 40. (4) 41. (2) 42. (2)
43. (1) 44. (2) 45. (5) 46. (1) 47. (2) 48. (1) 49. (3)
50. (5) 51. (3) 52. (3) 53. (1) 54. (5) 55. (4) 56. (1)
57. (4) 58. (2) 59. (4) 60. (3) 61. (2) 62. (3) 63. (1)
64. (2) 65. (4) 66. (5) 67. (4) 68. (5) 69. (5) 70. (3)
71. (2) 72. (3) 73. (5) 74. (2) 75. (4) 76. (1) 77. (2)
78. (3) 79. (5) 80. (4) 81. (3) 82. (1) 83. (1) 84. (3)
85. (3) 86. (4) 87. (1) 88. (4) 89. (3) 90. (1) 91. (4)
92. (3) 93. (4) 94. (2) 95. (2) 96. (4) 97. (1) 98. (5)
99. (4) 100. (1)
18. CODED INEQUALITIES
Explanations: Option (a) is correct. Let us see:
What is coded inequalities?
H < Q ..... (i); Q < F ..... (ii); L > F ..... (iii)
This segment of reasoning deals with the statements
involving ‘less than’, ‘greater than’, ‘equal to’, ‘not Combining all, we get H < Q < F < L. Hence, L >
equal to’ etc. are represented by unusual symbols. To H. Thus conclusion I follows. But H < F. Hence
solve the questions, such unusual symbols are substi- conclusion II (H < F) does not follow.
tuted by the real (usual) from given statements. For 2. Statements:
example, the usual symbol for ‘greater then is’ ‘>’ but J$T, T@V, V#M
in inequalities it can be coded as P, , O, or any other Conclusion:
unsual symbol. I. T#M II. J©M
Important symbols Explanations: Option (d) is correct. Let us see:
> = Geater than J > T ..... (i); T < V ..... (ii); V < M ..... (iii)
< = Less than from (ii) and (iii), we get T < M. Hence, conclu-
> = Greater than or equal to sion I (T < M) does not follow. J and M just can’t
< = Less than or equal to be compared. Hence conclusion II does not follow.
= = Equal to 3. Statements:
Formats of the question U#D, D@R, R©T
Directions (Q. 1 - 5 ): In these questions the Conclusion:
symbols @, #, $ and © are used with different I. U@R II. T$D
meaning as follows: Explanations: Option (e) is correct. Let us see:
‘A@B’ means ‘A is smaller than B’. U < D ..... (i); D < R ..... (ii); R = T ..... (iii)
‘A#B’ means ‘A is either smaller than or equal to B’. Combining all, we get U < D < R = T. Hence, U <
R. Thus conclusion II follows.
‘A$B’ means ‘A is greater than B’. 4. Statements:
‘A B’ means ‘A is either greater than or equal to B’. M L, L$K, K@R
‘A©B’ means ‘A is neither greater than nor smaller Conclusion:
than B’.
I. M R II. M@R
In each question, three statements showing
Explanations: Option (c) is correct. Let us see:
relationships have been given, which are followed
by two conclusions I & II . Assuming that the given M > L ..... (i); L > K ..... (ii); K < R ..... (iii)
statements are true, find out which conclusion(s) from (ii) and (iii), we get L and R can’t be com-
is/are definitely true. pared. Consequently, M and R can also not be com-
Give answer a) if only conclusion I is true pared. But either conclusion I (M > R) or
conclusion II (M < R) follows.
Give answer b) if only conclusion II is true.
5. Statements:
Give answer c) if either conclusion I or conclu-
sion II is true. J@N, N©W, W$V
Give answer d) if neither conclusion I nor conclu- Conclusion:
sion II is true. I. J©V II. J@W
Give answer e) if both conclusion I and II are true. Explanations: Option (b) is correct. Let us see:
1. Statements: J < N ..... (i); N = W ..... (ii); W > V ..... (iii)
H#Q, Q@F, L$F from I and II, we get J < W ..... (iv). Hence, con-
Conclusion: clusion II follows. But (iii) and (iv) do not lead us
to a relationship between J and V. Hence conclu-
I. L$H II. H#F
sion I does not follow.
Exercise
Directions (Q. 1 - 5 ): In the following questions, I. R K II. M@K
the symbols , , #, © and @ are used with the fol- Directions (Q. 6 - 10 ): Symbols @, %, , $ and #
lowing meaning as illustrated below: are used with different meanings as explained be-
‘P Q’ means ‘P is not greater than Q’. low:
‘P Q’ means ‘P is neither greater than nor equal to Q’. ‘A$B’ means ‘A is not greater than B’.
‘A#B’ means ‘A is not smaller than B’.
‘P#Q’ means ‘P is not smaller than Q’. ‘A@B’ means ‘A is neither greater than not equal to
‘P©Q’ means ‘P is neither smaller than nor greater than B’.
Q’. ‘A%B’ means ‘A is neither smaller than nor equal B’.
‘P@Q’ means ‘P is neither smaller than nor equal to ‘A B’ means ‘A is neither smaller than nor greater than
Q’. B’.
Now in each of the following questions assuming the In each question, three statements showing relation-
given statements to be true, find which of the two ships have been given, which are followed by two
conclusions I and II given below them is/are defi- conclusions I & II . Assuming that the given state-
nitely true? ments are true, find out which conclusion(s) is/are
Give answer a) if only conclusion I is true definitely true.
Give answer b) if only conclusion II is true. Give answer a) if only conclusion I is true
Give answer c) if either conclusion I or conclu- Give answer b) if only conclusion II is true.
sion II is true. Give answer c) if either conclusion I or conclu-
Give answer d) if neither conclusion I nor conclu- sion II is true.
sion II is true. Give answer d) if neither conclusion I nor conclu-
Give answer e) if both conclusions I and II are sion II is true.
true. Give answer e) if both conclusion I and II are true.
1. Statements: 6. Statements:
B N, N K, K©D H%L, L G, G#M
Conclusion: Conclusion:
I. D@B II. B K I. H#M II. H%G
2. Statements: 7. Statements:
F@R, R W, W#D A$J, J%R, P@R
Conclusion: Conclusion:
I. D R II. F@W I. J%P II. P A
3. Statements: 8. Statements:
T V, V@N, N J K%N, N#U, U$M
Conclusion: Conclusion:
I. J@V II. J V I. N M II. N%M
4. Statements: 9. Statements:
M©T, T@R, R#J E%I, I K, K@J
Conclusion: Conclusion:
I. J T II. R M I. K%E II. J%I
5. Statements: 10. Statements:
K©B, B M, M@R B S, S@D, U#D
Conclusion: Conclusion:
I. S@U II. D%B @, #, %, $ and © are used with different meanings
Directions (Q. 11 - 15 ): In these questions symbols as follows:
@, #, $, % and © are used with different meanings ‘A@B’ means ‘A is not smaller than B’.
as follows: ‘A#B’ means ‘A is neither smaller than nor equal to B’.
< ‘A@B’ means ‘A is smaller than B’.
> ‘A#B’ means ‘A is greater than B’. ‘A%B’ means ‘A is not greater than B’.
< ‘A$B’ means ‘A is either smaller than or equal to B’. ‘A$B’ means ‘A is neither greater than nor equal to B’.
‘A©B’ means ‘A is neither smaller than nor greater than
> ‘A%B’ means ‘A is either greater than or equal to B’.
B’. 16. Statements:
= ‘A©B’ means ‘A is neither greater than nor smaller J#H, H©T, T$R, R%F
than B’. Conclusion:
In each question, three statements showing relation- I. J#R II. R#F
ships have been given, which are followed by two
III. J#T
conclusions I & II . Assuming that the given state-
ments are true, find out which conclusion(s) is/are (a) Only I is true
definitely true. (b) Only I & II are true
Mark answer a) if only conclusion I is true (c) Only III is true (d) All are true
Mark answer b) if only conclusion II is true. (e) None of these
Mark answer c) if either conclusion I or II is true. 17. Statements:
Mark answer d) if neither I nor II is true. E$P, P%H, H@I, I#K
Mark answer e) if both conclusion I and II are Conclusion:
true. I. P©I II. I%E
11. Statements: III. H%K
L$K, K@M, J%M (a) Only I is true (b) Only II is true
Conclusion: (c) Only III is true
I. L@M II. K@J (d) Only I and II are true
12. Statements: (e) None is true
E$W, W@Q, Q%H 18. Statements:
Conclusion: L@K, K#R, R$H, H%N
I. E$Q II. E©H Conclusion:
13. Statements: I. L#R II. N#R
J©T, T#W, W%R III. L©N
Conclusion: (a) Only I and II are true
I. J#R II. T%R (b) Only either I or II is true
14. Statements: (c) Only II and III are true
T#R, R%H, H@F (d) Only III is true
Conclusion: (e) None of these
I. H@T II. F©T 19. Statements:
15. Statements: T$V, V%K, K©L, L%H
F©D, D#V, V@P Conclusion:
Conclusion: I. V%H II. T$L
I. F©P II. V#F III. T$H
Directions (Q. 16 - 20 ): In these questions, symbols (a) Only I and II are true
(b) Only II and III are true I. V#D II. F#D
(c) Only I and III are true III. V©F
(d) All are true (a) Only I is true
(e) None of these (b) Only II is true
20. Statements: (c) Only either I or II is true
V@W, W#D, D$M, M%F (d) Both I and II are true
Conclusion: (e) None of these
Answer
1. a 2. d 3. c 4. e 5. b
6. b 7. a 8. d 9. b 10. e
11. e 12. d 13. a 14. a 15. b 16. c
17. e 18. a 19. d 20. d

Answers with Explanation


1. (a) Let us see: 6. (b) Let us see:
B < N ..... (i); N < K ..... (ii); K = D ..... (iii). H > L ..... (i); L = G ..... (ii); G > M ..... (iii).
Combining these, we get B < N < K = D. Combining all, we get H > L = G > M. Hence,
Hence, D > B and Conclusion I is true. But B H > M. This does not lead us to conclusion I.
< K. Hence conclusion II (B < K) is not true. And H > G. This is conclusion II.
2. (d) Let us see: 7. (a) Let us see:
F > R ..... (i); R < W ..... (ii); W > D ..... (iii). A < J ..... (i); J < R ..... (ii); P < R ..... (iii).
from (ii) and (iii), no relationship can be es- Combining, (ii) and (iii), we get J > P. Hence,
tablished between D and R. Hence conclu- conclusion I is true. But A and P can’t be com-
sion I does not follow. pared from (i) and I. Hence II is not true.
Nor do (i) and (ii) lead us to conclude any re- 8. (d) Let us see:
lationship between F and W. Hence conclusion K > N ..... (i); N > U ..... (ii); U < M ..... (iii). N
II does not follow. and M can’t be compared from (ii) and (iii).
3. (c) Let us see: Nor are I and II exhaustive because they are
T < V ..... (i); V > N ..... (ii); N < J ..... (iii). V silent on the possibility N < M. Hence neither
and J can’t be compared on the basis of (ii) is definitely ture.
and (iii). But the two conclusion (J > V and J 9. (b) Let us see:
< V) are complementary. Hence either I or II E > I ..... (i); I = K ..... (ii); K < J ..... (iii).
is true. Combining (i) and (ii), we get E > I = K or E >
4. (e) Let us see: K. Hence, conclusion I (K > E) is not true.
M = T ..... (i); T > R ..... (ii); R > J ..... (iii). Combining (ii) and (iii) we get, I = K < J or I <
Combining these, we get M = T > R > J. Hence, J. Hence conclusion II (J > I) is true.
J < T and conclusion I follows. Also R < M 10. (e) Let us see:
and conclusion II follows. B = S ..... (i); S < D ..... (ii); U > D ..... (iii).
5. (b) Let us see: Combining (ii) and (iii), we get S < D < U or S
K = B ..... (i); B < M ..... (ii); M > R ..... (iii). < U. Hence, conclusion I is true. Combining
from (ii) and (iii) we can’t establish any rela- (ii) and (iii) we get, B = S < D or B < D. Hence
tionship between R and B, and therefore be- conclusion II (D > B) is true.
tween R and K. Hence conclusion I does not 11. (e) Let us see:
follow. From (i) and (ii) conclusion II follows. L < K ..... (i); K < M ..... (ii); J > M ..... (iii).
Combining all, we get L < K < M < J. Hence, < R < F. Obiviously, R and J can’t be com-
L < M and I follows. pared. Hence, conclusion I does not follow.
Again, K < J and II follows. Again, R < F. Hence II (R > F) is false. But J >
12. (d) Let us see: T. Hence III follows.
E < W ..... (i); W < Q ..... (ii); Q > H ..... (iii). 17. (e) Let us see:
From (i) and (ii), we get E < W < Q or E < Q E < P ..... (i); P < H ..... (ii); H > I ..... (iii); I >
...... (iv). Hence, I does not follow. Again, from K ..... (iv). Combining these, we get E < P < H
(iv) and (iii), E and H can’t be compatred. > I > K. Hence, P and I can’t be compared.
Hence II does not follow. Nor can I and E. Thus I and II do not follow.
13. (a) Let us see: Also, H > K. Hence III (H < K) is false.
J = T ..... (i); T > W ..... (ii); W > R ..... (iii). 18. (a) Let us see:
Combining all, we get J = T > W > R. Hence, J L > K ..... (i); K > R ..... (ii); R < H ..... (iii); H
> R and so I follows. Also, T > R and so II < N ..... (iv). Combining these, we get L > K >
does not follows. R < H < N. Hence, L > R and I follows. Also,
14. (a) Let us see: N > R and II follows. But L and N can’t be
compared. Hence III does not follow.
T > R ..... (i); R > H ..... (ii); H < F ..... (iii).
From (i) and (ii), we get T > R > H or H < T. 19. (d) Let us see:
Hence, conclusion I follows. But form I and T < V ..... (i); V < K ..... (ii); K = L ..... (iii); L
(iii) F and T can’t be compared. Hence II does < H ..... (iv). Combining these, we get T < V <
not follows. K = L < H. Hence, V < H and I is true. Again,
15. (b) Let us see: T < L and II is true. Also, T < H and III is true.
F = D ..... (i); D > V ..... (ii); V < P ..... (iii). 20. (d) Let us see:
From (i) and (ii), we get F = D > V or V < F. V > W ..... (i); W > D ..... (ii); D < M ..... (iii);
Hence, conclusion II is follows. But from II M < F ..... (iv). Combining these, we get V >
and (iii) F and P can’t be compared. W > D < M < F. Hence, V > D and I is true.
16. (c) Let us see: Again, F > D and II is true. But V and F can’t
be compared. Hence III does not follow.
J > H ..... (i); H = T ..... (ii); T < R ..... (iii); R <
F ....... (iv). Combining these, we get J > H = T

Self Evaluation Test


Directions (Q. 1 - 5 ): In the following questions, ingly.
the symbols @, $, , # and are used with the fol- 1. Statements:
lowing meaning as illustrated below: H@T, T#F, F E, E V
‘P$Q’ means ‘P is not smaller than Q’. Conclusion:
‘P@Q’ means ‘P is neither smaller than nor equal to I. V$F II. E@T
Q’.
III. H@V IV. T#V
‘P#Q’ means ‘P is neither greater than nor equal to Q’.
(a) Only I, II and III are true
(b) Only I, II and IV are true
‘P Q’ means ‘P is neither greater than nor smaller than
(c) Only II, III and IV are true
Q’.
(d) Only I, III and IV are true
‘P Q’ means ‘P is not greater than Q’.
(e) All I, II, III and IV are true
Now in each of the following questions, assuming
the given statements to be true, find which of the 2. Statements:
four conclusions I, II, III and IV given below them D#R, R K, K@F, F$J
is/are definitely true and give your answer accord- Conclusion:
I. J#R II. J#K
III. R#F IV. K@D than Q’.
(a) Only I, II and III are true 6. Statements:
(b) Only II, III and IV are true J©M, M K, K%D
(c) Only I, III and IV are true Conclusion:
(d) All I, II, III and IV are true I. J©D II. D M
(e) None of these III. K©J
3. Statements: (a) None is true (b) Only I is true
N B, B$W, W#H, H M (c) Only II is true (d) Only III is true
Conclusion: (e) Only I and II are true
I. M@W II. H@N 7. Statements:
III. W N IV. W@N R@K, K$F, F N
(a) Only I is true (b) Only III is true Conclusion:
(c) Only IV is true I. N©R II. F@R
(d) Only III or IV is true III. F©R
(e) Only either III or IV and I are true (a) Only I is true
4. Statements: (b) Only either II or III is true true
R D, D$J, J#M, M@K (c) Only I and either II or III are true
Conclusion: (d) Only III is true
I. K#J II. D@M (e) None of these
III. R#M IV. D@K 8. Statements:
(a) None is true (b) Only I is true R$D, D@N, N©F
(c) Only II is true (d) Only III is true Conclusion:
(e) Only IV is true I. F D II. F R
5. Statements: III. N%R
M$K, K@N, N R, R#W (a) Only I and II are true
Conclusion: (b) Only I and III are true
I. W@K II. M$R (c) Only II and III are true
III. K@W IV. M@N (d) All are true
(a) Only I and II are true (e) None of these
(b) Only I, II and III are true 9. Statements:
(c) Only III and IV are true H T, T$B, B©R
(d) Only II, III and IV are true Conclusion:
(e) None of these I. R©H II. B©H
Directions (Q. 6 - 10 ): In the following questions, III. T H
the symbols $, %, @, © and are used with the (a) Only I is true
following meanings as illustrated belows: (b) Only I and II are true
‘P%Q’ means ‘P is not smaller than Q’. (c) Only I and III are true
‘P$Q’ means ‘P is not greater than Q’. (d) Only II and III are true
‘P Q’ means ‘P is neither greater than nor equal to Q’. (e) None of these
10. Statements:
‘P©Q’ means ‘P is neither smaller than nor equal to M%D, D K, K$N
Q’.
Conclusion:
‘P@Q’ means ‘P is neither greater than nor smaller
I. K©M II. N©D sion II is true.
III. M©N Give answer e) if both conclusion I and II are true.
(a) Only I is true (b) Only II is true 11. Statements:
(c) Only III is true F#H, H%K, K$R, R©M
(d) Only I and II are true Conclusion:
(e) None of these I. F%R II. F#M
Directions (Q. 11 -15 ): The symbols @, #, $, % and 12. Statements:
© are used with the different meaning as follows: L@D, D©P, P#V, V%G
‘A@B’ means ‘A is smaller than B’. Conclusion:
‘A#B’ means ‘A is not smaller than B’. I. P%L II. G@P
‘A$B’ means ‘A is neither smaller than nor greater than 13. Statements:
B’. E%W, W©Q, Q$T, T@H
‘A%B’ means ‘A is greater than B’. Conclusion:
‘A©B’ means ‘A is not greater than B’. I. H#W II. H#E
In each of the following questions assuming the 14. Statements:
given statements to be true, find out which of the J©T, T@H, H%I, I$L
two conclusions I and II given below them is/are Conclusion:
definitely true. I. L©H II. J©I
Give answer a) if only conclusion I is true 15. Statements:
Give answer b) if only conclusion II is true. R@Q, Q%P, P©V, V#M
Give answer c) if either conclusion I or conclu- Conclusion:
sion II is true. I. R@P II. R#P
Give answer d) if neither conclusion I nor conclu-

Answer
1. b 2. e 3. e 4. a 5. e
6. a 7. c 8. b 9. e 10. b
11. a 12. e 13. d 14. d 15. c
19. BINARY NUMBERS
Before getting concept of binary numbers, we
must understand what is decimal numbers. How to convert binary numbers to decimal
numbers?
Decimal numbers
In the above explanation we have already con-
Such numbers are represented by using ten digits
verted a binary number 101 into decimal number , i.e.,
0, 1, 2, 3, 4, 5, 6, 7, 8 and 9
5. Now we will convert the same binary number into
For example 187 is a decimal number. We can
decimal number through shortcut method. Let us see:
write 187 in the following manner: 187 = (1 102) +
Step 1: We write 1, 2, 4, 8......... below the digits, from
(8 101) + (7 100) = 100 + 80 + 7 = 187
the right end.
Note:
100 = 1 as the value is always equal to 1 if any
number is raised to the power of zero.
The writting pattern of 187 makes it clear that to Step 2: Only that digits/numbers will be added
get the value of any decimal number we follow the which fall below 1. The addition will be our decimal
following rules: number.
• The 1st digit from right end is multiplied by 100. Hence, the required decimal number for 101
• The 2nd digit from right end is multiplied by 101. = 4 + 1 = 5.
• The third digit from right end is multiplied by 102. How to convert decimal number to binary number?
In similar manner if the number has n digits then Shortcut method: A decimal number is converted into
the nth digit from right end will be multiplied by 10n - 1. binary by the method of successive divisions in the
following way:
Finally, we add all these
(i) Each time the dividend is divided by 2.
Binary Numbers
(ii) The remainder is noted and the quotient becomes
Unlike the decimal number the binary numbers
the next dividend.
are represented by only two digits 0 & 1 and digits of a
number are multiplied by power of 2 and not by the (iii) The quotient (the next dividend) again divided by
power of 10 as in the case of decimal numbers. 2 and this process is repeated utill no more divi-
sion is possible.
For example 101 will be written as follows:
Let us see the conversion of 17 into binary num-
101 = (1 22) + (0 21) + (1 20)
ber:
=4+0+1=5
Now it is clear that 101 in the binary system rep-
resents the number five which is represented as 5 in
the decimal number system.
The writting pattern of 101 makes it clear that to
get the value of any binary number we follow the rule Required binary number for 17 = 10001
given below: Formats of the questions
• The 1st digit from right end is multiplied by 20 (= Directions (Q. 1 - 2 ): Study the following in-
1). formation carefully and answer the questions given
• The 2nd digit from right end is multiplied by 21(= below:
2). In a certain code, the symbol for 0 (zero) is and
• The third digit from right end is multiplied by 22(= that for 1 is . There are no other symbols for all
4). other number greater than one. The number greater than
In similar manner if the number has n digits then one are to be written only by using the two symbols
the nth digit from right end will be multiplied by 2n - 1. given above. The value of the symbol for 1 doubles
itself every time it shifts one place to the left. Study
Finally, we add all these
the following examples: 1. Option (a) is the correct option. Let us see:
0 is written as
11101 =
1 is written as
2 is written as Adding nos. below 1s, 16 + 8 + 4 +1 = 29
3 is written as
4 is written as and so on. 2. Option (c) is correct. Let us see:
1. Which of the following numbers will be repre- HCF of 10, 20 and 15 = 5.
sented by ? 5 will be represented as
(a) 29 (b) 30
(c) 22 (d) 21
2. Which of the following will represent the HCF
of 10, 20 and 15? 101 =
(a) (b) Note:
(c) (d) In place of and , symbols may be , $, , or
Sol: (1 - 2) anything else.

Exercise
Directions (Q. 1 - 5 ): Study the following (a) 25 (b) 23
information carefully and answer the question given (c) 22 (d) 18
below: 2. Which of the following is the HCF of ,
In a certain code, the symbol for 0 (zero) is and ?
and that for 1 is . There are no other symbols for all (a) (b)
other number greater than one. The number greater than
(c) (d)
one are to be written only by using the tow symbols
given above. The value of the symbol for 1 doubles 3. Find the value of divided by
itself every time it shifts one place to the left. Study the (a) (b)
following examples: (c) (d)
0 is written as 4. How does one write 14 + 2 - 7 in this code?
1 is written as (a) (b)
2 is written as (c) (d)
3 is written as 5. Find the value of 4 5 + 23 - 6 6 + 4?
4 is written as and so on. (a) (b)
1. Which of the following is represented by (c) (d)
?

Answer
1. a 2. c 3. c 4. a 5. b
Answers with Explanation
decimal numbers we get 4, 6 and 10 respec-
1. (a) tively. The HCF of 4, 6 and 10 = 2. 2 is repre-
Adding the numbers below 1 the required an- sented in binary representation as 10 or
swer = 16 + 8 + 1 = 25 3. (c) = 1111
111
2. (c) = 100, = 110,
10, = 11
1
= 1010. By converting 100, 110 and 1010 into In decimal no. 1111 = 15 & 11 = 3
15 3 = 5 Required answer = 1001 =
By converting 5 into binary no. 5. (b) 4 5 + 23 - 6 6 + 4
= 20 + 23 - 36 + 4 = 11
Now converting 11 into binary no.

Required answer = 101


4. (a) 14 + 2 - 7 = 9
By converting 9 into binary no.
Required answer = 1011 =

Self Evaluation Test


Directions (Q. 1 - 5 ): Study the following 2. Which of the following will represent HCF of
information carefully and answer the question given 10, 15 and 18?
below: (a) Q (b) P
In a certain code, the symbol for 0 (zero) is P and that (c) QQ
for 1 is Q . There are no other symbols for all other
(d) Can’t be determined
number greater than one. The number greater than one
are to be written only by using the tow symbols given 3. Which of the following will represent the value
above. The value of the symbol for 1 doubles itself ev- of 2 + 10 2?
ery time it shifts one place to the left. Study the follow- (a) QPP (b) QPQ
ing examples: (c) QQQ (d) QPQP
0 is written as P 4. Which of the following will be represented by
1 is written as Q QQPQQ?
2 is written as Q P (a) 30 (b) 25
3 is written as Q Q (c) 27 (d) 23
4 is written as Q P P and so on. 5. Which of the following will represent the LCM
1. Which of the following will represent the aver- of 10, 15 and 18?
age of QPQPP and QPQP? (a) QPQQQPP (b) QPQQPQP
(a) PQP (b) QPPP (c) QQQPPQP (d) QPPQQPQ
(c) QQQQ (d) QQPP

Answer
1. c 2. a 3. c 4. c 5. b
20. STRONG & WEAK ARGUMENT
Strong Argument
(b) if only argument II is strong
It touches the practical and real aspect of the situ-
(c) if either I or II is strong
ation as described in the statement.
(d) if neither I nor II is strong
Weak Argument
(e) if both I and II strong
It is less important and superfluous.
1. Statement:
Identification of strong arguments
Should new big industries be started in Kolkata?
(i) They give realistic diagonosis of the situation de-
Arguments:
scribed in the statement.
I. Yes. It will create a new job opportunities.
(ii) They give the deep analysis of the topic dealt
within the statement. II. No. It will further add to the pollution of the
city.
(iii) They relate with the statement and be supported
up by facts/established notions. Explanation: Option (e) is correct because creation
of new jobs as well as addition to pollution both
(iv) They are not mere reiteration of the situation given
are truth after begining of new big industries.
in the statement.
2. Statement:
(v) They are not based on imagination.
Is paying ransom or agreeing to the conditions of
(vi) They are not like a question.
kidnappers of political figures, a proper course of
Formats of the questions
action?
Directions (Q. 1 - 2): Each question given below
Arguments:
is followed by two arguments numbered I and II.
I. Yes. The victims must be saved at all cost.
You have to decide which of the arguments is a
‘strong’ argument and which is a ‘weak’ argument. II. No. It encourages the kidnappers to continue
with their sinister activities.
Give answer:
Explanation: Option (e) is correct because well
(a) if only argument I is strong
connected to the statement and very substantial.

Exercise
Directions (Q. 1 - 5): In making decision (c) if either I or II is strong
about important questions, it is desirable to be (d) if neither I nor II is strong
able to distinguish between ‘strong’ arguments
(e) if both I and II strong
and ‘weak’ arguments. ‘Strong’ arguments are
those which are both important and directly 1. Statement:
related to the question. ‘Weak’ arguments are Should the parents in India in future be forced
those which are of minor importance and also to opt for only child as against two or many
may not be directly related to the question or at present?
may be rel at ed t o a t ri vial aspect of t he Arguments:
question. I. Yes, this is the only way to check the ever-
Each question below is followed by two increasing population of India.
arguments numbered I and II. You have to decide II. No, this type of pressure tactic is not
which of the arguments is a ‘strong’ argument and adopted by any other country in the world.
which is a ‘weak’ argument.
2. Statement:
Give answer:
Should the sex determination test during
(a) if only argument I is strong pregnancy be completely banned?
(b) if only argument II is strong
Arguments: question.
I. Yes, this leads to indiscriminate female Each question below is followed by two
foeticide and eventually will lead to social arguments numbered I and II. You have to decide
imbalance. which of the arguments is a ‘strong’ argument and
II. No, people have a right to know about their which is a ‘weak’ argument.
unborn child. Give answer:
3. Statement: (a) if only argument I is strong
S h o u l d a l l t h e s l u ms i n b i g c i t i e s b e (b) if only argument II is strong
demolished and the people living in such (c) if either I or II is strong
slums be relocated outside the city limits? (d) if neither I nor II is strong
Arguments: (e) if both I and II strong
I. No, all these people will lose their home 6. Statement:
and livelihood and hence they should not
Should the sale of all the toys made in China
be relocated.
be banned in India?
II.Yes, the big cities need more and more
Arguments:
spaces to carry out development activities
and hence these slums should be removed. I. Yes, these are very cheap and hence will
put the local toy manufacturers out of
4. Statement:
business.
Should there be a complete ban on mining coal
II. No, Indian toys are of much better quality
in India?
and their sale will not be affected.
Arguments:
7. Statement:
I. Yes, the present stock of coal will not last
Should there be no examination upto Std. IX
long if we continue mining at the present
in all the schools in India?
rate.
Arguments:
II. No, we do not have alternate energy source
of sufficient quantity. I. No, students need to go through the process
of giving examinations right from young
5. Statement:
age.
Should there be uniforms for students in the
II. Yes, this will hel p st udent s to t hi nk
colleges in India as in the schools?
laterally and achieve then creative pursuits.
Arguments:
8. Statement:
I. Yes, this will improve the ambience of the
Should there be only a uniform rate of income
colleges as all the students will be decently
tax irrespective of the level of income?
dressed.
Arguments:
I I . N o , c o l l e ge s t u d e n t s s h o u l d n o t b e
regimented and they should be left to I. Yes, this will substantially reduce the work
choose their clothes for coming to college. o f t h e o f f i c i a l s o f t h e i n c o me t a x
department.
Directions (Q. 6 - 10): In making decision
about important questions, it is desirable to be II. No, this will reduce Govt tax collection to
able to distinguish between ‘strong’ arguments a large extent.
and ‘weak’ arguments. ‘Strong’ arguments are 9. Statement:
those which are both important and directly Should there be only two political parties in
related to the question. ‘Weak’ arguments are India?
those which are of minor importance and also Arguments:
may not be directly related to the question or
I. Yes, in many developed countries there are
may be rel at ed t o a t ri vial aspect of t he
only two political parties. completion of graduation and are capable
II. No, Indian electorate is not mature to select of performing euqally well as the graduate
between only two political parties. candidates.
10. Statement: 12. Statement:
Shoul d t he sal e of t obacco pr od uct s be S h o u l d t h e e x a mi n a t i o n b o d i e s f o r a l l
restricted to only a few outlets in each city/ university examinations permit the use of
town? calculators?
Arguments: Arguments:
I. Ye s , t h i s w i l l s u b s t a t i a l l y r e d u c e I. No, it is necessary for the students to know
consumption of tobacco products. the methods of manual calcualtion to make
II. No, those who want to purchase tobacco their concepts clear.
products should get them at convenient II. Yes, manual calculations are no more
locations. required with extensive use of computers
Directions (Q. 11 - 15): In making decision in all fields.
about important questions, it is desirable to be 13. Statement:
able to distinguish between ‘strong’ arguments Should the knowledge of Hindi language be
and ‘weak’ arguments. ‘Strong’ arguments are made compulsory for all the employees of
those which are both important and directly public sector organisations?
related to the question. ‘Weak’ arguments are Arguments:
those which are of minor importance and also I. Yes, it is necessary for dealing with people
may not be directly related to the question or from the educationally backward strata of
may be rel at ed t o a t ri vial aspect of t he the society.
question.
II. No, it is not necessary for every employee
Each question below is followed by two to have the knowledge of Hindi language.
arguments numbered I and II. You have to decide
14. Statement:
which of the arguments is a ‘strong’ argument and
which is a ‘weak’ argument. Should it be made compulsory for all the
private sector organisations to reserve quota
Give answer:
for socially backward classes?
(a) if only argument I is strong
Arguments:
(b) if only argument II is strong
I. N o, t he pr i vat e s ec t o r s ho ul d n ot b e
(c) if either I or II is strong governed by the Government rules.
(d) if neither I nor II is strong II. Yes, private sector organisation should also
(e) if both I and II strong contribute in upliftment of socially
11. Statement: backward classes.
S h o u l d gr a d u a t i o n b e ma d e mi n i mu m 15. Statement:
educational qualification for entry-level jobs Should the women be advised not to travel
in any public sector organisation? alone at night in vi ew of the increasing
Arguments: incidences of rape and sexual abuse?
I. Yes, graduates always perform better than Arguments:
non-graduates by virtue of their higher I. No, instead the Government should take
level of education. measures to control such incidences.
II. No, there are quite a few people who II. Yes, it is difficult even for the police
cannot afford to remain unemployed till the department to control such cases.
Answer
1. d 2. a 3. e 4. b 5. e 6. c
7. e 8. b 9. d 10. a 11. d 12. c
13. a 14. b 15. d

Answers with Explanation


1. (d) I is weak because it is superfluous; it does not officials is not too desirable a motive. II is
go into the reason for population control. II is strong because reduced tax collection will have
an argument by example and hence it is weak. a bad impact on state activities.
2. (a) I is strong because female foeticide is undesir- 9. (d) I is weak as it is an argument by example. II is
able. II is weak: which right are we talking weak because it is absurd. Infact, if maturity is
about? Right to know the sex of the unborn really seen as a criteria, lesser maturity would
child? No. Parents can wait till the child’s birth. be required when the choice is limited.
3. (e) I is strong on humanitarian grounds. II is 10. (a) I is strong because reduced tobacco consump-
strong on economic grounds. tion is desirable. II is weak because such con-
4. (b) I is weak because it is not relevant to “com- venience is not desirable.
plete ban”. II is a strong argument because ban- 11. (d) I is not necessarily true. Hence I is not strong
ning mining in such a scenario will lead us into II deviates from the core issue. Hence II is not
great trouble. strong.
5. (e) I is strong as improved ambience is desirable. 12. (c) Methods of mannual calculation are necessary
II is strong because regimentation of adults is for individuals. Hence they should be encour-
undesirable. aged. Thus, I is strong. II is also strong in its
6. (c) Both the arguments are strong but both can’t way. But since I and II contradict each other,
be true at the same time. The sale will either we go for either.
be affected (as I says) or not be affected (II 13. (a) I is strong as it is true to a large extent. But II
says). Hence, either is strong. is simplistic and hence weak.
7. (e) I is strong as school is the ground where we 14. (b) I is not right way of arguing. Hence I is not
prepare for the future battles of life. II is strong strong. Upliftment of socially backward classes
because examinations kill our creativity, turn- is necessary. Hence II is strong.
ing us all into mere clerks. 15. (d)
8. (b) I is weak because reudction of workload of IT
Self Evaluation Test
Directions (Q. 1 - 5): In making decision which of the arguments is a ‘strong’ argument and
about important questions, it is desirable to be which is a ‘weak’ argument.
able to distinguish between ‘strong’ arguments Give answer:
and ‘weak’ arguments. ‘Strong’ arguments are
(a) if only argument I is strong
those which are both important and directly
related to the question. ‘Weak’ arguments are (b) if only argument II is strong
those which are of minor importance and also (c) if either I or II is strong
may not be directly related to the question or (d) if neither I nor II is strong
may be rel at ed t o a t ri vial aspect of t he (e) if both I and II strong
question. 1. Statement:
Each question below is followed by two Should the major part of school examinations
arguments numbered I and II. You have to decide be made objective-type?
Arguments: problems like water pollution and water-
I. No, objective-type, examination does not logging and hence it is necessary to ban it.
test the students ability to express. Directions (Q. 6 - 10): In making decision
II. Yes, this is the best method of assessing about important questions, it is desirable to be
one’s ability and knowledge. able to distinguish between ‘strong’ arguments
2. Statement: and ‘weak’ arguments. ‘Strong’ arguments are
those which are both important and directly
Should the Government service in rural areas
related to the question. ‘Weak’ arguments are
at least for two years after completion of
those which are of minor importance and also
gr ad uat i on be made compul s ar y f or t he
may not be directly related to the question or
students of medicine?
may be rel at ed t o a t ri vial aspect of t he
Arguments: question.
I. Yes, it is everyone’s duty to serve the Each question below is followed by three
people in rural areas and contribute to their arguments numbered I, II and III. You have to
upliftment. decide which of the arguments is a ‘strong’
II. No, it cannot be applied onl y to the argument and which is a ‘weak’ argument.
medicine students since anyways they are 6. Statement:
cont r i but i n g dur i ng t hei r st udi es and
Shoul d smoking ci garat tes and dr i nking
particularly in the period of internship.
alcohol by the actors be completely banned
3. Statement: in the movies in India?
Should all the factories in the cities be shifted Arguments:
to the outskirts, far away from the main city?
I. Yes, this will significantly reduce the trend
Arguments: of smoking cigarattes and drinking alcohol
I. Yes, this is an essential step for controlling among the youth in India.
pollution in the city. II. No, there should be no such ban on the
II.N o , s u c h a s t e p w i l l l e a d t o l o t o f creative pursuits of the filmmaker.
inconvenience to the employees of the III. No, the films portray the society and
factories and their families as well. hence such scenes should be an integral
4. Statement: part of the movie if the storyline demands
Should the practice of rewarding high scores so.
be stopped to handle frustration among the (a) None is strong
moderate scoreres? (b) Only I and II are strong
Arguments: (c) Only II and III are strong
I. No, it is necessary to motivate the high (d) Only I and III are strong
scorers and reward is one of the best ways
7. Statement:
of motivating.
Should sale of vital human organs be made
II. Yes, too much appreciation for high scores
legal in India?
affects the moderate students adversely at
times leading to extreme situations. Arguments:
5. Statement: I. No, it goes against our culture.
Should there be a total ban on use of plastic II. No, this will lead to unhealthy practices.
bags? III. Yes, this will bring an end to the illegal
Arguments: trading of human organs.
I. No, instead the thickness of plastic bags, (a) None is strong
which can be used without much damage (b) Only I and II are strong
to the environment, should be specified. (c) Only III is strong
II. Yes, use of plastic bags causes various (d) Only II and III are strong
8. Statement: afford to pay salaries to the level of private
Should the conscription of citizens for defence sector.
services be made compulsory in India? I I I . Ye s , o t h e r w i s e t h e p u b l i c s e c t o r
Arguments: undertakings will not be able to compete
I. Yes, this is the only way to tackle the with the private sector organisations.
serious shortage of manpower in defence (a) None is strong
servies. (b) Only III is strong
II. No, instead the compensation package be (c) Only I is strong
made comparable to other job sectors to (d) Only I and III are strong
attract people to join defence services. 10. Statement:
III. Yes, many other countries have made this Should there be a complete ban on registration
compulsory. of new cars for a few months in the big cities
(a) Only I is strong. in India?
(b) Only II is strong Arguments:
(c) Only I and II are strong I. Yes, this will significantly reduce the
(d) None of these number of cars on the already overcrowded
9. Statement: roads of the big cities in India.
Should the salary and perquisites of public II. Yes, the existing car owners will be very
s e c t o r u n d e r t a ki n g e mp l o y e e s b e ma d e happy as they will face less traffic snarls
equivalent to those in the private sector? in peak hours.
Arguments: III. No, this is highly discriminatory against
I. Ye s, t hi s w i l l he l p t h e pu b l i c s e ct o r those who decide to buy cars now and
u n d e r t a ki n gs t o a t t r a c t a n d r e t a i n hence should not be enforced.
competent workforce. (a) Only I is strong
II. No, public sector undertakings cannot (b) Only I and III are strong
(c) Only III is strong
(d) All are strong

Answer
1. a 2. d 3. a 4. a 5. e 6. d
7. a 8. d 9. d 10. c

You might also like